Apostila de Filosofia Unesp

Propaganda
Capítulo I
A FILOSOFIA
BROWNE, Dik. Hagar, o terrível.
P
roveniente do grego clássico φιλοσοφία (philosofia: philos, que ama; sophia, sabedoria), a palavra “filosofia”
tem, como ocorre com os assuntos complexos, várias definições possíveis. Vejamos, pois, como o dicionário a
explica:
Filosofia: 1. amor pela sabedoria, experimentado apenas pelo ser humano consciente de sua própria ignorância [...] 5. no âmbito da relação entre teoria e prática, pensamento inicialmente contemplativo, em que o ser humano busca compreender a si
mesmo e a realidade circundante, e que irá determinar, em seguida, o seu caráter prescritivo ou prático, voltado para a ação
concreta e suas consequências éticas, políticas ou psicológicas. [HOUAISS, Antonio. Dicionário eletrônico Houaiss da língua
portuguesa.]
Seja qual for a definição que se faça, é interessante notar que, como notou o filósofo Martin Heidegger (18891976), quando se indaga “o que é Filosofia?” parte-se de uma questão que, para ser respondida, necessita da própria
resposta que se procura. Isso significa que questionar o que é Filosofia é, a seu modo, filosofar: se quero saber algo,
se procuro averiguar sua natureza, sua essência, já estou filosofando. E mesmo quando quero ignorar algo sou levado
a filosofar para que, consciente de minhas vontades, de minha visão de mundo, de minha liberdade de escolha, ignore
alguma coisa (que geralmente é a própria Filosofia! Mas não nos condenemos por isso). Tal situação ocorre porque
filosofar é buscar racionalmente o conhecimento verdadeiro; o “querer saber”, próprio dos homens, está na raiz do
pensar filosófico:
Se me disponho a filosofar, é porque busco compreender as coisas e os fatos que me envolvem, a Realidade em que estou imerso. É porque quero saber o que posso saber e como devo ordenar minha visão do Mundo, como situar-me diante do Mundo físico e do Mundo humano e de tudo quanto se oferece à minha experiência. Como entender os discursos dos homens e meu próprio discurso. Como julgar os produtos das artes, das religiões e das ciências. [PORCHAT, Oswaldo. A Filosofia e a visão comum do mundo.]
Com isso, vale salientar que a Filosofia pode ser encontrada (ou mesmo inserida) em quaisquer campos do conhecimento; onde existirem dúvidas, onde existirem interrogações fecundas, a Filosofia se coloca como atividade que
se presta a responder as questões que surgem aos homens para que, por fim, o saber se construa e, com ele, uma
melhor perspectiva de se encarar o mundo, a existência e o próprio homem.
Filosofar é olhar-se no espelho e retirar, da imagem que se vê, a silhueta mais precisa, menos distorcida daquilo
que é objeto de estudo: conhecer-se, entender-se, analisar-se, verificar o mundo, compreendê-lo, situá-lo e situarmonos enquanto peças fundamentais de um processo de conhecimento é a atitude que, à Filosofia, resta para que possamos fugir, nós, enquanto seres racionais, de tudo o que nos prende e não nos deixa prevalecer ante o equívoco, a
mentira, o engano.
E por não se tratar de um mero sistema de opiniões, de vontades individuais, de “achismos”, é importante salientar que a Filosofia é arranjada de maneira sistemática, o que quer dizer que ela, para “funcionar”, busca
[...] enunciados precisos e rigorosos, busca encadeamentos lógicos entre os enunciados, opera com conceitos ou ideias obtidos
por procedimentos de demonstração e prova, exige a fundamentação racional do que é enunciado e pensado. Somente assim a
reflexão filosófica pode fazer com que nossa experiência cotidiana, nossas crenças e opiniões alcancem uma visão crítica de si
mesmas. Não se trata de dizer “eu acho que”, mas de poder afirmar “eu penso que”. O conhecimento filosófico é um trabalho
intelectual. É sistemático porque não se contenta em obter respostas para as questões colocadas, mas exige que as próprias
questões sejam válidas e, em segundo lugar, que as respostas sejam verdadeiras, estejam relacionadas entre si, esclareçam
umas às outras, formem conjuntos coerentes de ideias e significações, sejam provadas e demonstradas racionalmente. [Pode-se
dizer também que] a Filosofia opera sistematicamente, com coerência e lógica, que a Filosofia tem uma vocação para formar
um todo daquilo que aparece de modo fragmentado em nossa experiência cotidiana. [CHAUÍ, Marilena. Convite à filosofia.]
Mas vejamos um pouco mais a fundo a natureza do assunto deste capítulo.
A Filosofia, nascida do espanto perante a própria ignorância do homem (como pensou Aristóteles, e bem evidenciada na famosa frase socrática “sei que nada sei”), deu-lhe uma ferramenta para que pudesse, diante daquilo que lhe
assombrava, lhe admirava, lhe surpreendia (a vida, a morte, o nada, o tudo etc.), encontrar razões que explicassem o
que lhe perturbava e, assim, conseguir sair do estado originário em que se encontrava, qual seja, a ignorância. Para
definirmos com maior precisão, tal espanto seria a situação em que
1
[...] tomamos distância do nosso mundo costumeiro, através de nosso pensamento, olhando-o como se nunca o tivéssemos visto
antes, como se não tivéssemos tido família, amigos, professores, livros e outros meios de comunicação que nos tivessem dito o
que o mundo é; como se estivéssemos acabando de nascer para o mundo e para nós mesmos e precisássemos perguntar o que
é, por que é e como é o mundo, e precisássemos perguntar também o que somos, por que somos e como somos. [Idem.]
Desse modo, aquele que age filosoficamente possui o que podemos chamar de atitude filosófica. E o que seria isto, a atitude filosófica? Seria, pois, quando diante de uma questão, um fato, uma evidência, tomarmos conscientemente
a decisão de não aceitar como óbvias e evidentes as coisas, as ideias, os fatos, as situações, os valores, os comportamentos de
nossa existência cotidiana; jamais aceitá-los sem antes havê-los investigado e compreendido. [Idem.]
As indagações que surgem, essência fundamental da Filosofia, fazem com que os homens busquem um saber que
lhes é, em um primeiro momento, impossível, para depois se transformar em algo improvável, depois em algo provável e, enfim, em algo possível. E o que seria esse algo que se faz possível? Seria, pois, o conhecimento humano e a
tentativa de encontrar respostas às questões que mais intrigam os homens.
A Filosofia pergunta qual é a realidade ou natureza e qual é a significação de alguma coisa, não importa qual; — perguntar
como a coisa, a ideia ou o valor, é. A Filosofia indaga qual é a estrutura e quais são as relações que constituem uma coisa,
uma ideia ou um valor; — perguntar por que a coisa, a ideia ou o valor, existe e é como é. A Filosofia pergunta pela origem ou
pela causa de uma coisa, de uma ideia, de um valor. A atitude filosófica inicia-se dirigindo essas indagações ao mundo que nos
rodeia e às relações que mantemos com ele. Pouco a pouco, porém, descobre que essas questões se referem, afinal, à nossa capacidade de conhecer, à nossa capacidade de pensar. Por isso, pouco a pouco, as perguntas da Filosofia se dirigem ao próprio
pensamento: o que é pensar, como é pensar, por que há o pensar? A Filosofia torna-se, então, o pensamento interrogando-se a
si mesmo. Por ser uma volta que o pensamento realiza sobre si mesmo, a Filosofia se realiza como reflexão. [Idem.]
Autor desconhecido.
Pode-se dizer que a atitude filosófica tem, em sua essência, outras duas atitudes: a atitude crítica e o pensamento
crítico:
A primeira característica da atitude filosófica é negativa, isto é, um dizer não ao senso comum, aos pré-conceitos, aos préjuízos, aos fatos e às ideias da experiência cotidiana, ao que “todo mundo diz e pensa”, ao estabelecido. A segunda característica da atitude filosófica é positiva, isto é, uma interrogação sobre o que são as coisas, as ideias, os fatos, as situações, os
comportamentos, os valores, nós mesmos. É também uma interrogação sobre o porquê disso tudo e de nós, e uma interrogação
sobre como tudo isso é assim e não de outra maneira. O que é? Por que é? Como é? Essas são as indagações fundamentais da
atitude filosófica. A face negativa e a face positiva da atitude filosófica constituem o que chamamos de atitude crítica e pensamento crítico. A Filosofia começa dizendo não às crenças e aos preconceitos do senso comum e, portanto, começa dizendo que
não sabemos o que imaginávamos saber. [CHAUÍ, Marilena. Convite à filosofia.]
Diante disso, pode-se perguntar qual é, afinal, o alcance de nosso pensamento: o que podemos conhecer? Tudo,
nada, muito, pouco? A princípio, quando se filosofa deve-se partir da liberdade que o homem tem de pensar, de fazer
significar o mundo e a si mesmo, sem algemas que o prendam e que lhe neguem a possibilidade de raciocinar sobre
aquilo que, por algum motivo, é válido que raciocine. Nesse caso, surge outra questão:
Há algum conhecimento tão certo que nenhum homem razoável possa dele duvidar? Esta questão, que à primeira vista parece
fácil, é na realidade uma das mais difíceis que se podem fazer. Quando tivermos compreendido as dificuldades com que se defronta uma resposta clara e segura, estaremos bem lançados no estudo da filosofia — uma vez que a filosofia é apenas a tentativa de responder a estas questões fundamentais, não descuidadamente e dogmaticamente, como fazemos na vida quotidiana e
mesmo nas ciências, mas criticamente, após termos explorado tudo o que torna estas questões embaraçosas e termos compreendido toda a vagueza e confusão que subjazem às nossas ideias vulgares. [RUSSELL, Bertrand. Aparência e realidade.]
Ao ser que pensa, a Filosofia é uma oportunidade para que se liberte das opiniões provenientes do senso comum,
das respostas prontas, do “por que sim e ponto final”. Saber separar o conhecimento verdadeiro (resultado que é de
uma reflexão crítica, livre, coerente e sensata) do conhecimento comum é uma das consequências de se pensar filosoficamente. Em um mundo repleto de informações, dúvidas, angústias, posicionamentos e respostas, se não se separar
o joio do trigo não há como se avaliar adequadamente quaisquer questões: um jardim floresce com beleza apenas se
2
alguém cultivá-lo, excluir do campo as ervas daninhas e outras pragas, cuidando para que as flores possam crescer e
florescer, proporcionando aquilo que o jardim tem como função: dar ao homem a possibilidade de melhorar, aperfeiçoar de alguma maneira seu mundo. Se quisermos, podemos compreender que o jardineiro não é ninguém mais que o
filósofo, alguém que metodicamente atenta-se para algo e retira, desse algo, resultados que melhorem a compreensão
que tem de sua existência enquanto ser que está no mundo.
Nascida do espanto, da admiração genuína, a palavra do filósofo tem o dom de apreender, elaborar e formular o que caracteriza essencialmente um tempo ou figura do mundo. [GIACOIA JR., Oswaldo. “Sofrimento cordial”. In: Folha de S. Paulo (14 de
junho de 2009).]
E assim, quando temos um ser humano em busca de respostas para as questões que se colocam à sua frente, é válido ressaltar que, em se tratando de um ser humano, as explicações naturalmente tendem a possuir algo de subjetivo,
ou seja, algo não apenas racional, mas também sentimental, com algum resquício de individualidade que é extremamente nocivo ao conhecimento por ser capaz de desvirtuá-lo, deixá-lo impuro, fazendo com que uma resposta, qualquer que seja, ao invés de se basear apenas nos traços objetivos que o mundo evidencia, mescla-se com “achismos” e
traços que, muitas vezes, são válidos apenas à pessoa que a pensou. E, sendo a Filosofia uma atitude crítica que busca
um conhecimento universal, fica evidente que qualquer resposta que sirva apenas para uma pessoa, e não a qualquer
ser humano, não é uma resposta filosoficamente válida.
Mesmo os mais belos sentimentos não pertencem à filosofia. Diz-se que os sentimentos são algo de irracional. A filosofia, pelo
contrário, não é apenas algo racional, mas a própria guarda da ratio [razão]. [...] Todavia, tão logo pomos em suspeição a
caracterização da filosofia como um comportamento racional, torna-se, da mesma maneira, também duvidoso se a filosofia
pertence à esfera do irracional. Pois quem quiser determinar a filosofia como irracional, toma como padrão para a determinação o racional, e isto de um tal modo que novamente pressupõe como óbvio o que seja a razão. Se, por outro lado, apontamos
para a possibilidade de que aquilo a que a filosofia se refere concerne a nós homens em nosso ser e nos toca, então poderia ser
que esta maneira de ser afetado não tem absolutamente nada a ver com aquilo que comumente se designa como afetos e sentimentos, em resumo, o irracional. [HEIDEGGER, Martin. Qu’est-ce que la philosophie?]
Portanto, aliar ao pensar filosófico aquilo que é subjetivo faz com que o conhecimento puro, real, livre de perspectivas individuais fique cada vez mais difícil (ou mesmo impossível) de ser alcançado. Deve-se, pois, ao se filosofar, despir o pensamento de tudo aquilo que nos é íntimo e pessoal: nossas crenças, nossas esperanças, nossos desgostos, nossas alegrias, nossas dores, nosso querer. Se acaso não filtrarmos tais “nossos”, teremos, então, um conhecimento inválido porque não será passível de ser estendido a todos (a não ser, claro, que todos os homens fossem iguais
e pensassem de maneira igual — o que é, vale dizer, uma ilusão —).
A filosofia origina-se de uma tentativa obstinada de atingir o conhecimento real. Aquilo que passa por conhecimento, na vida
comum, padece de três defeitos: é convencido, incerto e, em si mesmo, contraditório. O primeiro passo rumo à filosofia consiste em nos tornarmos conscientes de tais defeitos, não a fim de repousar, satisfeitos, no ceticismo indolente, mas para substituílo por uma aperfeiçoada espécie de conhecimento que será experimental, precisa e autoconsistente. Naturalmente, desejamos
atribuir outra qualidade ao nosso conhecimento: a compreensão. [RUSSELL, Bertrand. Dúvidas filosóficas.]
Mas, afinal, onde e por que motivo surgiu a Filosofia? Para respondermos tal questão é necessário termos em
mente o palco onde, pela primeira vez, o homem sentiu a necessidade de construir um edifício seguro do conhecimento: a Grécia Clássica.
HARDIN. The funny times. (“Comer. Sobreviver. Reproduzir. / Comer. Sobreviver. Reproduzir. / Comer. Sobreviver. Reproduzir. /
Comer. Sobreviver. Reproduzir. / O que é isso tudo, afinal?”)
A Filosofia nasce quando alguns homens, espantados e admirados com a realidade, notaram-se insatisfeitos com
as explicações que a tradição estabelecia, começando, assim, a buscarem respostas a todas as intrigantes questões que
tinham em mente, fossem elas antigas ou novas. Para tal, confiavam na hipótese de que a razão era capaz de eviden-
3
ciar, captar, esclarecer e responder as interrogações que, antes, não eram tratadas com o mesmo cuidado formal,
sistemático.
Em suma, a Filosofia surge quando se descobriu que a verdade do mundo e dos humanos não era algo secreto e misterioso,
que precisasse ser revelado por divindades a alguns escolhidos, mas que, ao contrário, podia ser conhecida por todos, através
da razão, que é a mesma em todos; quando se descobriu que tal conhecimento depende do uso correto da razão ou do pensamento e que, além da verdade poder ser conhecida por todos, podia, pelo mesmo motivo, ser ensinada ou transmitida a todos.
[CHAUÍ, Marilena. Convite à filosofia.]
Mas como era antes? Vale esclarecer que, em um momento anterior ao nascimento da Filosofia, a civilização
grega tinha não a razão como ferramenta racional para explicar o mundo; em seu lugar existiam os mitos, isto é,
explicações fantasiosas sobre a origem e o porquê das coisas, interpretações do mundo que não levavam em conta se
eram ou não passíveis de serem provadas, se eram ou não pertinentes com a realidade: não importava se, para explicar determinado fenômeno, fosse necessário utilizar alguma ideia impossível; importava, pois, dar uma explicação,
qualquer que fosse, aos homens.
Por que os seres nascem e morrem? Por que os semelhantes dão origem aos semelhantes, de uma árvore nasce outra árvore,
de um cão nasce outro cão, de uma mulher nasce uma criança? [...] Por que tudo muda? A criança se torna adulta, amadurece, envelhece e desaparece. [...] Por que a doença invade os corpos, rouba-lhes a cor, a força? Foram perguntas como essas
que os primeiros filósofos fizeram e para elas buscaram respostas. Sem dúvida, a religião, as tradições e os mitos explicavam
todas essas coisas, mas suas explicações já não satisfaziam aos que interrogavam sobre as causas da mudança, da permanência, da repetição, da desaparição e do ressurgimento de todos os seres. Haviam perdido força explicativa, não convenciam
nem satisfaziam a quem desejava conhecer a verdade sobre o mundo. [Idem.]
Narrativas imaginárias sobre a origem de algo (do ser humano, da Terra, dos animais, do poder etc.), os mitos resumiam o mundo e o homem a partir de uma perspectiva que não levava em conta a razão. Nesse contexto, a Filosofia originou-se e destacou-se como um modo de explicar o mundo, assim como faziam os mitos, mas com a diferença
de explicá-lo levando em conta um ponto de vista racional, a coerência das respostas, a ligação possível entre uma
interpretação do mundo e aquilo que o mundo poderia oferecer para confirmar a explicação dada. Com isso, podemos
esboçar uma comparação entre o mito e a Filosofia:
1. O mito pretendia narrar como as coisas eram ou tinham sido no passado imemorial, longínquo e fabuloso, voltando-se para
o que era antes que tudo existisse tal como existe no presente. A Filosofia, ao contrário, se preocupa em explicar como e por
que, no passado, no presente e no futuro (isto é, na totalidade do tempo), as coisas são como são;
2. O mito narrava a origem através de genealogias e rivalidades ou alianças entre forças divinas sobrenaturais e personalizadas, enquanto a Filosofia, ao contrário, explica a produção natural das coisas por elementos e causas naturais e impessoais.
[...]
3. O mito não se importava com contradições, com o fabuloso e o incompreensível, não só porque esses eram traços próprios
da narrativa mítica, como também porque a confiança e a crença no mito vinham da autoridade religiosa do narrador. A Filosofia, ao contrário, não admite contradições, fabulação e coisas incompreensíveis, mas exige que a explicação seja coerente,
lógica e racional; além disso, a autoridade da explicação não vem da pessoa do filósofo, mas da razão, que é a mesma em todos os seres humanos. [Idem.]
Tenhamos em mente que, diariamente, temos contato com situações, coisas, pessoas e atitudes que, diante das
quais, aceitamos, negamos, desejamos, evitamos, escolhemos. Questionamos (Aquela aula de Filosofia é hoje?),
opinamos (Mas o professor é um chato!), afirmamos (Terei que ir.), avaliamos (Mas que a aula é chata, é!). Mas tais
atitudes, por mais simples e corriqueiras que possam parecer, contêm alguns pressupostos que não prestamos atenção;
tornam-se, pois, naturalidades. Se questiono “Aquela aula de filosofia é hoje?” tenho em mente que estou matriculado(a) em um cursinho e que há, queira eu ou não, aulas de Filosofia aos sábados; isso com a finalidade de estudar
para uma prova e garantir um lugar na faculdade, assim conquistando, em longo prazo, um diploma e assim poder, no
mercado de trabalho, ter liberdade econômica e viver o melhor que puder. Em outro exemplo, se opino que “O professor de filosofia é um chato!” tenho que ter em mente alguma avaliação sobre o que é um professor chato, ter em
mente que tais professores me aborrecem e que aborrecer-me não é algo que eu queira numa tarde de sábado.
Como se vê, diariamente temos contato com pensamentos que não examinamos porque nos parecem corriqueiros,
naturais; pode-se notar, assim, que simples perguntas apresentam, ocultas, várias ideias que não ousamos questionar.
A Filosofia surge nesse momento, para mergulhar a fundo em nossos pensamentos e deixar de lado a superficialidade
das ideias, dos fatos, do mundo, de nós mesmos. E se, nesse momento, seguíssemos uma atitude filosófica? Não seria
possível, assim, conquistar algumas conclusões válidas que possam nos proporcionar algo de útil?
E se, mesmo depois de sabermos que temos a possibilidade de entender melhor as coisas, continuássemos a viver
e a pensar de maneira limitada, incompleta, duvidosa, engolindo, sem mastigar, as “verdades” que nos impõem,
estaríamos sendo honestos com nosso próprio bom senso, com nossa própria liberdade? Respondam, filósofos!
Leituras complementares
Ora, muitos fazem uma outra pergunta: afinal, para que Filosofia?
É uma pergunta interessante. Não vemos nem ouvimos ninguém perguntar, por exemplo, para que matemática ou
física? Para que geografia ou geologia? Para que história ou sociologia? Para que biologia ou psicologia? Para que
astronomia ou química? Para que pintura, literatura, música ou dança? Mas todo mundo acha muito natural perguntar:
Para que Filosofia?
4
Em geral, essa pergunta costuma receber uma resposta irônica, conhecida dos estudantes de Filosofia: “A Filosofia é uma ciência com a qual e sem a qual o mundo permanece tal e qual”. Ou seja, a Filosofia não serve para nada.
Por isso, se costuma chamar de “filósofo” alguém sempre distraído, com a cabeça no mundo da lua, pensando e dizendo coisas que ninguém entende e que são perfeitamente inúteis.
Essa pergunta, “Para que Filosofia?”, tem a sua razão de ser.
Em nossa cultura e em nossa sociedade, costumamos considerar que alguma coisa só tem o direito de existir se tiver alguma finalidade prática, muito visível e de utilidade imediata.
Por isso, ninguém pergunta para que as ciências, pois todo mundo imagina ver a utilidade das ciências nos produtos da técnica, isto é, na aplicação científica à realidade.
Todo mundo também imagina ver a utilidade das artes, tanto por causa da compra e venda das obras de arte,
quanto porque nossa cultura vê os artistas como gênios que merecem ser valorizados para o elogio da humanidade.
Ninguém, todavia, consegue ver para que serviria a Filosofia, donde dizer-se: não serve para coisa alguma.
Parece, porém, que o senso comum não enxerga algo que os cientistas sabem muito bem. As ciências pretendem
ser conhecimentos verdadeiros, obtidos graças a procedimentos rigorosos de pensamento; pretendem agir sobre a
realidade, através de instrumentos e objetos técnicos; pretendem fazer progressos nos conhecimentos, corrigindo-os e
aumentando-os.
Ora, todas essas pretensões das ciências pressupõem que elas acreditam na existência da verdade, de procedimentos corretos para bem usar o pensamento, na tecnologia como aplicação prática de teorias, na racionalidade dos conhecimentos, porque podem ser corrigidos e aperfeiçoados.
Verdade, pensamento, procedimentos especiais para conhecer fatos, relação entre teoria e prática, correção e acúmulo de saberes: tudo isso não é ciência, são questões filosóficas. O cientista parte delas como questões já respondidas, mas é a Filosofia quem as formula e busca respostas para elas.
Assim, o trabalho das ciências pressupõe, como condição, o trabalho da Filosofia, mesmo que o cientista não seja
filósofo. No entanto, como apenas os cientistas e filósofos sabem disso, o senso comum continua afirmando que a
Filosofia não serve para nada.
Para dar alguma utilidade à Filosofia, muitos consideram que, de fato, a Filosofia não serviria para nada, se “servir” fosse entendido como a possibilidade de fazer usos técnicos dos produtos filosóficos ou dar-lhes utilidade econômica, obtendo lucros com eles; consideram também que a Filosofia nada teria a ver com a ciência e a técnica.
Para quem pensa dessa forma, o principal para a Filosofia não seriam os conhecimentos (que ficam por conta da
ciência), nem as aplicações de teorias (que ficam por conta da tecnologia), mas o ensinamento moral ou ético. A
Filosofia seria a arte do bem viver. Estudando as paixões e os vícios humanos, a liberdade e a vontade, analisando a
capacidade de nossa razão para impor limites aos nossos desejos e paixões, ensinando-nos a viver de modo honesto e
justo na companhia dos outros seres humanos, a Filosofia teria como finalidade ensinar-nos a virtude, que é o princípio do bem-viver.
Essa definição da Filosofia, porém, não nos ajuda muito. De fato, mesmo para ser uma arte moral ou ética, ou
uma arte do bem-viver, a Filosofia continua fazendo suas perguntas desconcertantes e embaraçosas: O que é o homem? O que é a vontade? O que é a paixão? O que é a razão? O que é o vício? O que é a virtude? O que é a liberdade? Como nos tornamos livres, racionais e virtuosos? Por que a liberdade e a virtude são valores para os seres humanos? O que é um valor? Por que avaliamos os sentimentos e as ações humanas?
Assim, mesmo se disséssemos que o objeto da Filosofia não é o conhecimento da realidade, nem o conhecimento
da nossa capacidade para conhecer, mesmo se disséssemos que o objeto da Filosofia é apenas a vida moral ou ética,
ainda assim, o estilo filosófico e a atitude filosófica permaneceriam os mesmos, pois as perguntas filosóficas — o
que, por que e como — permanecem.
CHAUÍ, Marilena. “Para que Filosofia?”. In: Convite à filosofia.
φ
Reflexão significa movimento de volta sobre si mesmo ou movimento de retorno a si mesmo. A reflexão é o movimento pelo qual o pensamento volta-se para si mesmo, interrogando a si mesmo.
A reflexão filosófica é radical porque é um movimento de volta do pensamento sobre si mesmo para conhecer-se
a si mesmo, para indagar como é possível o próprio pensamento.
Não somos, porém, somente seres pensantes. Somos também seres que agem no mundo, que se relacionam com
os outros seres humanos, com os animais, as plantas, as coisas, os fatos e acontecimentos, e exprimimos essas relações tanto por meio da linguagem quanto por meio de gestos e ações.
A reflexão filosófica também se volta para essas relações que mantemos com a realidade circundante, para o que
dizemos e para as ações que realizamos nessas relações.
A reflexão filosófica organiza-se em torno de três grandes conjuntos de perguntas ou questões:
1. Por que pensamos o que pensamos, dizemos o que dizemos e fazemos o que fazemos? Isto é, quais os motivos,
as razões e as causas para pensarmos o que pensamos, dizermos o que dizemos, fazermos o que fazemos?
2. O que queremos pensar quando pensamos, o que queremos dizer quando falamos, o que queremos fazer quando agimos? Isto é, qual é o conteúdo ou o sentido do que pensamos, dizemos ou fazemos?
3. Para que pensamos o que pensamos, dizemos o que dizemos, fazemos o que fazemos? Isto é, qual é a intenção
ou a finalidade do que pensamos, dizemos e fazemos?
Essas três questões podem ser resumidas em: O que é pensar, falar e agir? E elas pressupõem a seguinte pergunta:
Nossas crenças cotidianas são ou não um saber verdadeiro, um conhecimento?
5
Como vimos, a atitude filosófica inicia-se indagando: O que é? Como é? Por que é?, dirigindo-se ao mundo que
nos rodeia e aos seres humanos que nele vivem e com ele se relacionam. São perguntas sobre a essência, a significação ou a estrutura e a origem de todas as coisas.
Já a reflexão filosófica indaga: Por quê?, O quê?, Para quê?, dirigindo-se ao pensamento, aos seres humanos no
ato da reflexão. São perguntas sobre a capacidade e a finalidade humanas para conhecer e agir.
CHAUÍ, Marilena. “A reflexão filosófica”. In: Convite à filosofia.
φ
O primeiro ensinamento filosófico é perguntar: O que é o útil? Para que e para quem algo é útil? O que é o inútil?
Por que e para quem algo é inútil?
O senso comum de nossa sociedade considera útil o que dá prestígio, poder, fama e riqueza. Julga o útil pelos resultados visíveis das coisas e das ações, identificando utilidade e a famosa expressão “levar vantagem em tudo”.
Desse ponto de vista, a Filosofia é inteiramente inútil e defende o direito de ser inútil.
Não poderíamos, porém, definir o útil de outra maneira?
Platão definia a Filosofia como um saber verdadeiro que deve ser usado em benefício dos seres humanos.
Descartes dizia que a Filosofia é o estudo da sabedoria, conhecimento perfeito de todas as coisas que os humanos
podem alcançar para o uso da vida, a conservação da saúde e a invenção das técnicas e das artes.
Kant afirmou que a Filosofia é o conhecimento que a razão adquire de si mesma para saber o que pode conhecer e
o que pode fazer, tendo como finalidade a felicidade humana.
Marx declarou que a Filosofia havia passado muito tempo apenas contemplando o mundo e que se tratava, agora,
de conhecê-lo para transformá-lo, transformação que traria justiça, abundância e felicidade para todos.
Merleau-Ponty escreveu que a Filosofia é um despertar para ver e mudar nosso mundo.
Espinosa afirmou que a Filosofia é um caminho árduo e difícil, mas que pode ser percorrido por todos, se desejarem a liberdade e a felicidade.
Qual seria, então, a utilidade da Filosofia?
Se abandonar a ingenuidade e os preconceitos do senso comum for útil; se não se deixar guiar pela submissão às
ideias dominantes e aos poderes estabelecidos for útil; se buscar compreender a significação do mundo, da cultura, da
história for útil; se conhecer o sentido das criações humanas nas artes, nas ciências e na política for útil; se dar a cada
um de nós e à nossa sociedade os meios para serem conscientes de si e de suas ações numa prática que deseja a liberdade e a felicidade para todos for útil, então podemos dizer que a Filosofia é o mais útil de todos os saberes de que os
seres humanos são capazes.
CHAUÍ, Marilena. “Inútil? Útil?”. In: Convite à filosofia.
φ
A atividade filosófica é sui generis. Parecemos viver muito bem sem ela. Aprendemos e ensinamos, trabalhamos,
ouvimos música, vamos à praia e podemos construir nossas vidas com planos de sucesso e estabilidade financeira
sem nos deixarmos envolver pelo discurso e pelos problemas filosóficos. Na verdade, os problemas filosóficos normalmente nos deixam incomodados, mal humorados, ansiosos. Isso porque, como normalmente ocorre, ao tentar
resolvê-los, deparamo-nos com outros problemas que até então não havíamos considerado. A filosofia parece ser não
apenas desnecessária para o bem viver; ela parece ser incompatível com a ideia de uma vida tranquila. Somando-se a
isso, devemos considerar o caráter abstrato da atividade filosófica. Por lidar com problemas distantes da vida comum,
o filósofo é considerado frequentemente uma pessoa destacada da realidade, perdido em especulações inúteis, alheio
aos problemas que a vida diária se lhe impõem.
Essa visão negativa do filósofo rondou-o desde os primórdios da filosofia. Como ilustração, é interessante recorrer a uma lenda acerca de Tales, o grande matemático e filósofo grego que revolucionou a geometria, aquele que
inventou o Teorema de Tales, estudado nas aulas de matemática do 2º grau. Em sua época, cerca de 580 a.C., não
havia a divisão do conhecimento que há hoje, de modo que o intelectual era tanto matemático, quanto político, astrônomo, geômetra, etc. Conta a lenda que Tales certa vez passeava à noite olhando para as estrelas, com o intuito de
estudar seus movimentos e regularidades. Com os olhos fixos no céu, ele não percebeu que caminhava em direção a
um poço. Depois de tropeçar e cair dentro dele, uma jovem trácia que testemunhara o fato observou em tom sarcástico: “tão preocupado com os assuntos celestes que acabou esquecendo da terra que o sustenta”. Essa lenda é utilizada
para caracterizar a visão que o senso comum tem do filósofo. “Filosofia”, diz o dito popular, “é aquilo sem o qual o
mundo seria tal e qual”. O filósofo é visto como um sonhador de sonhos inefáveis, ou ainda como uma pessoa que
está sempre envolvida com assuntos que a grande maioria das pessoas não dá o menor valor.
Essa visão caricatural da filosofia não se restringe ao senso comum. Guimarães Rosa certa vez definiu o filósofo
como “aquele que se encontra num quarto escuro, à procura de um gato preto que não está lá. E ele o encontra...”
Fernando Pessoa, em seu famoso poema Tabacaria, escreve que “a metafísica... é uma consequência de se estar mal
disposto...” Mas será que é assim mesmo, quer dizer, será que é tão simples descartar a filosofia como uma atividade
intelectual inútil? Para obtermos uma resposta satisfatória, é necessário que especifiquemos o ofício do filósofo. Qual
é a natureza do trabalho filosófico?
A leitura dos filósofos sugere que a primeira característica distintiva do filósofo é a de lidar com ideias ou conceitos e não com objetos palpáveis, como o lavrador e o ferreiro. É claro que estes últimos não dispensam (e não podem
dispensar) o uso de ideias, o ferreiro recorrendo sempre à ideia ou ao modelo do martelo a ser construído e o lavrador
à ideia do solo e da época de plantio. O filósofo, porém, lida com ideias que não são sempre traduzíveis em coisas
concretas, tais como o conceito de verdade ou de bem. Além disso, contrariamente ao psicólogo e ao sociólogo, por
6
exemplo, o filósofo não está preocupado em colocar em prática as suas ideias. Isso não quer dizer que ele se recuse a
fazê-lo; ele simplesmente não considera a concretização de suas ideias como fundamental para a sua atividade. Como
diz Platão: “o filósofo permanece totalmente alheio ao seu vizinho mais próximo; ele é ignorante..., ele mal sabe se é
um homem ou um animal; ele está investigando a essência do homem”. Embora ele prefira o convívio das cidades,
“sua mente, desdenhando da irrelevância e da nulidade das coisas humanas, está sobrevoando o estrangeiro”. [...]
Não há dúvida de que o homem comum possa passar a vida inteira sem se preocupar com os problemas que rondam os filósofos. Mas ele, conscientemente ou não, está se valendo de motivos para tomar as tantas decisões que a
vida o obriga a tomar. Se olharmos mais de perto, veremos que esses motivos estão calcados em princípios ou regras
morais, ou em informações às vezes genuínas (ou verdadeiras), às vezes equivocadas (falsas). Quer dizer, o homem
comum não pára de refletir, de especular. A reflexão, quer ele se dê conta disso ou não, faz parte de sua vida do
mesmo modo que faz parte da vida dos intelectuais, sejam eles cientistas ou filósofos.
Mas a filosofia é mais do que refletir. Ela é refletir sobre o refletir. A filosofia surge quando a própria capacidade
de refletir é posta em questão, quer dizer, refletimos sobre o refletir, quando queremos saber como adquirimos conhecimentos, ou se sabemos realmente aquilo que supomos saber. Por isso que, para Sócrates, o ponto de partida do
filosofar é o reconhecimento da própria ignorância. A afirmação “só sei que nada sei” só pode ser feita por alguém
que já exerceu uma autocrítica, que já se debruçou sobre as bases de seus conhecimentos e os avaliou de modo adequado. Muitas vezes, quando fazemos isso honestamente, quer dizer, quando olhamos para dentro de nós mesmos e
pesquisamos as razões daquilo que defendemos às vezes tão teimosamente, nada encontramos, e aí ficamos espantados, perturbados, incomodados. Platão chamava esse estado de espírito de thaumazéin, isto é, o espanto da própria
ignorância. Esse é o motor do filosofar. É o que nos leva a tentar preencher o vazio, a ausência do saber, a ignorância.
Para esclarecer esse ponto, é oportuno comparar a filosofia com a ciência. A atividade do cientista é marcadamente empírica. Ele tenta entender o mundo como ele é dado em sua experiência e, a partir daí, ele procura predizer e
explicar os eventos. O cientista via de regra pergunta: “O que causou isso?” Ao tentar responder a essa pergunta, ele
recorre a outros eventos que requerem eles mesmos mais explicações. Quando ele se vê às voltas com uma sequência
de eventos interligados, ele pode perguntar: “O que causou a existência das séries?”, ou ainda, “por que esta série e
não outra?” Estas perguntas, porém, levam-no para além dos limites da atividade científica, tendo em vista que uma
série como essa não é dada na experiência. Esse território, às vezes considerado como obscuro, é a filosofia. Certas
questões levam-nos a níveis de abstração que nenhuma investigação empírica pode proporcionar. Elas surgem, podese dizer, no final de todas as outras pesquisas, “quando problemas relativos aos fundamentos dos saberes particulares,
como a Física, a Matemática, a Geometria, etc., são detectados ou seus métodos de investigação passam a ser questionados. Assim sendo, os problemas filosóficos e os sistemas destinados a resolvê-los são formulados em termos que
tendem a se referir aos domínios da possibilidade e da necessidade e não aos da realidade, ou seja, ao que poderia e
ao que deveria ser e não ao que é”.
Isso quer dizer que nem toda pesquisa fronteiriça aos saberes especiais é filosófica. Quando se tenta resolver problemas filosóficos sem se questionar a validade dos procedimentos adotados, incentiva-se o dogmatismo e a superstição. Por exemplo, no caso da existência da série de eventos, se pressupusermos que Deus é a causa primeira e também a meta final de todas as coisas, acabamos recorrendo a um artigo de fé e não a um saber racional. Essa afirmação
tem o mérito de produzir uma dada resposta a quebra-cabeças metafísicos, mas ela possui uma grande desvantagem,
que é a de se basear numa suposição que não pode ser colocada em dúvida, e que é por isso mesmo dogmática. Daí
não se segue que o filósofo deva necessariamente ser um ateu. Muitos filósofos do passado (e mesmo vários do presente) acreditam em Deus e pertencem a diferentes religiões. Mas quando eles decidem discutir a existência ou não de
Deus, eles sabem que não podem simplesmente postulá-la sem maiores problemas. Eles sabem que toda discussão é
uma disputa, uma busca da melhor explicação ou da solução de um certo problema. Decidir discutir significa submeter-se ao tribunal final da razão, que não aceita a mera crença incontestável como base de argumentação.
Tal problemática remete-nos à relação da filosofia com a religião. Sem dúvida que há semelhanças entre o filósofo e o religioso. Ambos procuram refletir sobre questões abstratas, ambos procuram explicações gerais, ambos procuram um princípio ou um conjunto de princípios fundamentais a partir dos quais podemos responder às questões mais
importantes que nos afligem. mas há pelo menos uma diferença essencial entre os dois: o religioso encontra o seu
princípio fundamental em algo que, em última instância, requer uma crença não-justificável em um Ser Superior que
explica tudo. O filósofo, por seu turno, procura a verdade ou aquilo que pode ser estabelecido através de bases racionais.
Isso nos conduz a uma outra característica importante da atividade filosófica, a saber, a preocupação com a verdade. As questões filosóficas podem muito bem ficar sem respostas, ou podem mesmo propiciar polêmicas intermináveis (como geralmente ocorre). Mas elas são questões de qualquer modo e requerem, por isso mesmo, uma avaliação das razões sugeridas e propostas para que possamos caracterizá-las como verdadeiras ou falsas. Afinal, a filosofia
não pode ser um mero aglomerado de proposições retóricas, sem qualquer pretensão de estabelecer princípios sólidos.
Ela pode ser definida como uma atividade a partir da qual se estudam métodos e metas das nossas formas diferenciadas de reflexão, a fim de que possamos chegar a conclusões sobre os seus limites e a sua validade. A pesquisa filosófica se dá de uma maneira racional, quer dizer, sem qualquer remissão à fé, visando o estabelecimento de respostas
convincentes a questões as mais diversas que fogem ao âmbito das ciências particulares, mas que são comumente
trazidas à luz por elas.
Muito bem. Já disse que a filosofia tem por função, entre outras coisas, refletir sobre o refletir. Através do filosofar, podemos saber mais sobre a nossa capacidade reflexiva. Por quê? Porque, em assim o fazendo, podemos exercer
o poder de reflexão mais amplamente, mais efetivamente e com mais precisão. Mas por que é tão importante exercer
a capacidade reflexiva? A resposta é simples, mas essencial. Sem refletir, não poderíamos ser livres. Agir sem refletir
significa não ser dono das próprias ações, ou ser movido por causas outras que não a nossa própria razão. Essa é a
diferença entre nós e os robôs. Eles não possuem poder de reflexão e por isso mesmo eles não podem escolher por si
7
mesmos o curso de ação que irão adotar. Do mesmo modo, quando adotamos um certo curso de ação “sem refletir”,
mecanicamente, a gente se assemelha a um autômato, ou a um robô nas mãos do primeiro que passa.
É neste momento que fica claro o porquê do filosofar. A ponte entre a filosofia e as outras áreas não é imediata.
Mas ela existe. Quando digo que sem refletir seríamos apenas autômatos, eu quero dizer que a atividade reflexiva é
condição de possibilidade das decisões livres. Se assim é, então filosofia tem a ver com liberdade. Explico melhor: se
a atividade reflexiva leva-nos a ser livres, e se a filosofia permite-nos usar essa capacidade reflexiva com cada vez
mais profundidade, então a filosofia pode ser vista como uma ferramenta essencial para a nossa liberdade, levandonos a pensar mais claramente e, em consequência disso, a usar a capacidade de escolha em sua plenitude. O exercício
da filosofia é a expressão mais profunda e plena da nossa liberdade. É a liberdade do pensar, do refletir, que nos leva
a agir livremente. O exercício da liberdade pressupõe que reflitamos sobre as nossas vidas, as nossas ações, as pessoas que nos rodeiam, o país em que vivemos, as regras da comunidade à qual pertencemos, e as informações (verdadeiros ou falsas) que obtemos, etc.
Esse é um resultado fundamental. Se surgir então a pergunta sobre o porquê de se estudar filosofia, independentemente dos interesses intelectuais de cada um, essa é uma resposta possível. Além disso, a relação entre filosofia e
liberdade permite que a gente responda àqueles que dizem que o filósofo em nada contribui para o desenvolvimento
da humanidade ou para a mudança (para melhor) da realidade. Se procurarmos mudar a realidade sem liberdade, na
verdade estaremos mudando algo não segundo a nossa vontade, mas segundo a vontade dos outros.
Uma outra lição que se pode tirar da relação entre filosofia e liberdade é que ela nos ajuda a compreender o porquê da insatisfação constante do filósofo, aquela que Hume sente e que o leva a passear ao longo do rio e a jogar
gamão com os seus amigos. A insatisfação origina-se do fato de que a atividade filosófica, assim como a atividade
teórica em geral, não parece ter um ponto final. Mas isso é exatamente o que a torna tão essencial à liberdade. O
trabalho filosófico em particular e o teórico em geral não têm fim. Conceber um fim à atividade reflexiva é, de um
certo modo, conceber o fim do exercício da liberdade. A gente só pára de refletir sobre os princípios que atuam como
premissas de argumentos quando a gente se rende à superstição, à religião ou ao totalitarismo.
Finalmente, pode-se dizer que a atividade reflexiva é auto-referente. Isso quer dizer que, mesmo para combatê-la,
a gente tem que adotá-la. Esse é o erro de Sexto Empírico e de outros céticos que suspeitavam da atividade especulativa. Eles só podem combater a especulação de modo persuasivo se eles adotarem um procedimento especulativo.
Eles só podem condenar uma teoria adotando outra. O que resta então é adotar uma teoria que resista a ataques, e que
explique pelo menos alguns dos problemas que nos afligem. Mas como descobrir essa teoria, que não é mágica, como
queriam os dogmáticos, mas que inevitavelmente se encontra na atividade intelectual, como negavam os céticos? No
caso da filosofia, a gente tem que filosofar mesmo para negar a filosofia, como uma vez disse Aristóteles. A gente
tem que ser filósofo mesmo se a gente desejar jogar fora a filosofia.
FRANCIOTTI, Marco Antonio. A natureza da atividade filosófica.
φ
A filosofia, como todos os outros estudos, visa em primeiro lugar o conhecimento. O conhecimento que ela tem
em vista é o tipo de conhecimento que confere unidade sistemática ao corpo das ciências, bem como o que resulta de
um exame crítico dos fundamentos de nossas convicções, de nossos preconceitos e de nossas crenças. Mas não se
pode dizer, no entanto, que a filosofia tenha tido algum grande êxito na sua tentativa de fornecer respostas definitivas
a seus problemas. Se perguntarmos a um matemático, a um mineralogista, a um historiador ou a qualquer outro cientista, que definido corpo de verdades foi estabelecido pela sua ciência, sua resposta durará tanto tempo quanto estivermos dispostos a lhe dar ouvidos. Mas se fizermos essa mesma pergunta a um filósofo, ele terá que confessar, se
for sincero, que a filosofia não tem alcançado resultados positivos tais como tem sido alcançados por outras ciências.
É verdade que isso se explica, em parte, pelo fato de que, mal se torna possível um conhecimento preciso naquilo que
diz respeito a determinado assunto, este assunto deixa de ser chamado de filosofia, e torna-se uma ciência especial.
Todo o estudo dos corpos celestes, que hoje pertence à Astronomia, se incluía outrora na filosofia; a grande obra de
Newton tem por título: Princípios matemáticos da filosofia natural. De maneira semelhante, o estudo da mente humana, que era uma parte da filosofia, está hoje separado da filosofia e tornou-se a ciência da psicologia. Assim, em
grande medida, a incerteza da filosofia é mais aparente do que real: aquelas questões para as quais já se tem respostas
positivas vão sendo colocadas nas ciências, ao passo que aquelas para as quais não foi encontrada até o presente
nenhuma resposta exata, continuam a constituir esse resíduo, que é chamado de filosofia. [...]
O valor da filosofia, na realidade, deve ser buscado, em grande medida, na sua própria incerteza. O homem que
não tem algumas noções de filosofia caminha pela vida afora preso a preconceitos derivados do senso comum, das
crenças habituais de sua época e do seu país, e das convicções que cresceram no seu espírito sem a cooperação ou o
consentimento de uma razão deliberada. Para tal homem o mundo tende a tornar-se finito, definido, óbvio; para ele os
objetos habituais não levantam problemas e as possibilidades infamiliares são desdenhosamente rejeitadas. Quando
começamos a filosofar, pelo contrário, imediatamente nos damos conta (como vimos nos primeiros capítulos deste
livro) de que até as coisas mais ordinárias conduzem a problemas para os quais somente respostas muito incompletas
podem ser dadas. A filosofia, apesar de incapaz de nos dizer com certeza qual é a verdadeira resposta para as dúvidas
que ela própria levanta, é capaz de sugerir numerosas possibilidades que ampliam nossos pensamentos, livrando-os
da tirania do hábito. Desta maneira, embora diminua nosso sentimento de certeza com relação ao que as coisas são,
aumenta em muito nosso conhecimento a respeito do que as coisas podem ser; ela remove o dogmatismo um tanto
arrogante daqueles que nunca chegaram a empreender viagens nas regiões da dúvida libertadora; e vivifica nosso
sentimento de admiração, ao mostrar as coisas familiares num determinado aspecto não familiar.
RUSSELL, Bertrand. O valor da Filosofia.
8
φ
A Filosofia existe há 25 séculos. Durante uma história tão longa e de tantos períodos diferentes, surgiram temas,
disciplinas e campos de investigação filosóficos enquanto outros desapareceram. Desapareceu também a ideia de
Aristóteles de que a Filosofia era a totalidade dos conhecimentos teóricos e práticos da humanidade. Também desapareceu uma imagem, que durou muitos séculos, na qual a Filosofia era representada como uma grande árvore frondosa,
cujas raízes eram a metafísica e a teologia, cujo tronco era a lógica, cujos ramos principais eram a filosofia da Natureza, a ética e a política e cujos galhos extremos eram as técnicas, as artes e as invenções. A Filosofia, vista como
uma totalidade orgânica ou viva, era chamada de “rainha das ciências”. Isso desapareceu. Pouco a pouco, as várias
ciências particulares foram definindo seus objetivos, seus métodos e seus resultados próprios, e se desligaram da
grande árvore. Cada ciência, ao se desligar, levou consigo os conhecimentos práticos ou aplicados de seu campo de
investigação, isto é, as artes e as técnicas a ele ligadas. As últimas ciências a aparecer e a se desligar da árvore da
Filosofia foram as ciências humanas (psicologia, sociologia, antropologia, história, linguística, geografia, etc.). Outros campos de conhecimento e de ação abriram-se para a Filosofia, mas a ideia de uma totalidade de saberes que
conteria em si todos os conhecimentos nunca mais reapareceu. No século XX, a Filosofia foi submetida a uma grande
limitação quanto à esfera de seus conhecimentos. Isso pode ser atribuído a dois motivos principais:
1. Desde o final do século XVIII, com o filósofo alemão Immanuel Kant, passou-se a considerar que a Filosofia,
durante todos os séculos anteriores, tivera uma pretensão irrealizável. Que pretensão fora essa? A de que nossa razão
pode conhecer as coisas tais como são em si mesmas. Esse conhecimento da realidade em si, dos primeiros princípios
e das primeiras causas de todas as coisas chama-se metafísica. Kant negou que a razão humana tivesse tal poder de
conhecimento e afirmou que só conhecemos as coisas tais como são organizadas pela estrutura interna e universal de
nossa razão, mas nunca saberemos se tal organização corresponde ou não à organização em si da própria realidade.
Deixando de ser metafísica, a Filosofia se tornou o conhecimento das condições de possibilidade do conhecimento
verdadeiro enquanto conhecimento possível para os seres humanos racionais. A Filosofia tornou-se uma teoria do
conhecimento, ou uma teoria sobre a capacidade e a possibilidade humana de conhecer, e uma ética, ou estudo das
condições de possibilidade da ação moral enquanto realizada por liberdade e por dever. Com isso, a Filosofia deixava
de ser conhecimento do mundo em si e tornava-se apenas conhecimento do homem enquanto ser racional e moral.
2. Desde meados do século XIX, como consequência da filosofia de Augusto Comte — chamada de positivismo
—, foi feita uma separação entre Filosofia e ciências positivas (matemática, física, química, biologia, astronomia,
sociologia). As ciências, dizia Comte, estudam a realidade natural, social, psicológica e moral e são propriamente o
conhecimento. Para ele, a Filosofia seria apenas uma reflexão sobre o significado do trabalho científico, isto é, uma
análise e uma interpretação dos procedimentos ou das metodologias usadas pelas ciências e uma avaliação dos resultados científicos. A Filosofia tornou-se, assim, uma teoria das ciências ou epistemologia (episteme, em grego, quer
dizer ciência). A Filosofia reduziu-se, portanto, à teoria do conhecimento, à ética e à epistemologia. Como consequência dessa redução, os filósofos passaram a ter um interesse primordial pelo conhecimento das estruturas e formas
de nossa consciência e também pelo seu modo de expressão, isto é, a linguagem. O interesse pela consciência reflexiva ou pelo sujeito do conhecimento deu surgimento a uma corrente filosófica conhecida como fenomenologia, iniciada pelo filósofo alemão Edmund Husserl. Já o interesse pelas formas e pelos modos de funcionamento da linguagem
corresponde a uma corrente filosófica conhecida como filosofia analítica cujo início é atribuído ao filósofo austríaco
Ludwig Wittgenstein. No entanto, a atividade filosófica não se restringiu à teoria do conhecimento, à lógica, à epistemologia e à ética. Desde o início do século XX, a História da Filosofia tornou-se uma disciplina de grande prestígio
e, com ela, a história das ideias e a história das ciências. Desde a Segunda Guerra Mundial, com o fenômeno do
totalitarismo — fascismo, nazismo, stalinismo —, com as guerras de libertação nacional contra os impérios coloniais
e as revoluções socialistas em vários países; desde os anos 60, com as lutas contra ditaduras e com os movimentos
por direitos (negros, índios, mulheres, idosos, homossexuais, loucos, crianças, os excluídos econômica e politicamente); e desde os anos 70, com a luta pela democracia em países submetidos a regimes autoritários, um grande interesse
pela filosofia política ressurgiu e, com ele, as críticas de ideologias e uma nova discussão sobre as relações entre a
ética e a política, além das discussões em torno da filosofia da História. Atualmente, um movimento filosófico conhecido como desconstrutivismo ou pós-modernismo, vem ganhando preponderância. Seu alvo principal é a crítica de
todos os conceitos e valores que, até hoje, sustentaram a Filosofia e o pensamento dito ocidental: razão, saber, sujeito,
objeto, História, espaço, tempo, liberdade, necessidade, acaso, Natureza, homem, etc. Quais são os campos próprios
em que se desenvolve a reflexão filosófica nestes vinte e cinco séculos? São eles:
Ontologia ou metafísica: conhecimento dos princípios e fundamentos últimos de toda a realidade, de todos os seres;
Lógica: conhecimento das formas gerais e regras gerais do pensamento correto e verdadeiro, independentemente
dos conteúdos pensados; regras para a demonstração científica verdadeira; regras para pensamentos não-científicos;
regras sobre o modo de expor os conhecimentos; regras para a verificação da verdade ou falsidade de um pensamento, etc.;
Epistemologia: análise crítica das ciências, tanto as ciências exatas ou matemáticas, quanto as naturais e as humanas; avaliação dos métodos e dos resultados das ciências; compatibilidades e incompatibilidades entre as ciências;
formas de relações entre as ciências, etc.;
Teoria do conhecimento ou estudo das diferentes modalidades de conhecimento humano: o conhecimento sensorial ou sensação e percepção; a memória e a imaginação; o conhecimento intelectual; a ideia de verdade e falsidade; a
ideia de ilusão e realidade; formas de conhecer o espaço e o tempo; formas de conhecer relações; conhecimento
ingênuo e conhecimento científico; diferença entre conhecimento científico e filosófico, etc.;
Ética: estudo dos valores morais (as virtudes), da relação entre vontade e paixão, vontade e razão; finalidades e
valores da ação moral; ideias de liberdade, responsabilidade, dever, obrigação, etc.;
9
Filosofia política: estudo sobre a natureza do poder e da autoridade; ideia de direito, lei, justiça, dominação, violência; formas dos regimes políticos e suas fundamentações; nascimento e formas do Estado; ideias autoritárias,
conservadoras, revolucionárias e libertárias; teorias da revolução e da reforma; análise e crítica das ideologias;
Filosofia da História: estudo sobre a dimensão temporal da existência humana como existência sociopolítica e
cultural; teorias do progresso, da evolução e teorias da descontinuidade histórica; significado das diferenças culturais
e históricas, suas razões e consequências;
Filosofia da arte ou estética: estudo das formas de arte, do trabalho artístico; ideia de obra de arte e de criação; relação entre matéria e forma nas artes; relação entre arte e sociedade, arte e política, arte e ética;
Filosofia da linguagem: a linguagem como manifestação da humanidade do homem; signos, significações; a comunicação; passagem da linguagem oral à escrita, da linguagem cotidiana à filosófica, à literária, à científica; diferentes modalidades de linguagem como diferentes formas de expressão e de comunicação;
História da Filosofia: estudo dos diferentes períodos da Filosofia; de grupos de filósofos segundo os temas e problemas que abordam; de relações entre o pensamento filosófico e as condições econômicas, políticas, sociais e culturais de uma sociedade; mudanças ou transformações de conceitos filosóficos em diferentes épocas; mudanças na
concepção do que seja a Filosofia e de seu papel ou finalidade.
CHAUÍ, Marilena. “Temas, disciplinas e campos filosóficos”. In: Convite à filosofia.
Cogito, logo aprendo
1. (UFRJ 2008) Filosofia é uma palavra de origem grega. Ela é constituída pela reunião de duas outras palavras
gregas: philia e sophia. O termo grego philia pode ser traduzido por amizade, afeição, amor. Já o termo sophia costuma ser traduzido por sabedoria. A partir dessas considerações:
a) indique o significado da palavra filosofia;
b) comente o sentido da atividade que ela designa.
2. “A filosofia é diferente da ciência e da matemática. Ao contrário da ciência, não assenta em experimentações
nem na observação, mas apenas no pensamento. E ao contrário da matemática não tem métodos formais de prova. A
filosofia faz-se colocando questões, argumentando, ensaiando ideias e pensando em argumentos possíveis contra elas,
e procurando saber como funcionam realmente os nossos conceitos.” [NAGEL, Thomas. O que é a filosofia?]
a) A filosofia tem um método que é diferente do método das outras disciplinas. Explique o que é o método filosófico.
b) O objeto da filosofia é diferente do objeto das outras disciplinas. Cite três exemplos de objetos filosóficos.
3. “Uma vez mais, o melhor é dar exemplos e apontar algumas das características mais salientes dos problemas filosóficos típicos. Pensemos, por exemplo, em Deus. Os cristãos têm uma dada concepção de Deus, os muçulmanos
outra e os hindus outra ainda. E há muitas mais, tantas quantas as religiões. As religiões partem de certas verdades
reveladas pelos seus profetas e inscritas nos seus livros sagrados; procuram descobrir a verdadeira natureza de Deus e
encontrar o caminho da salvação. Mas nada disso são problemas filosóficos.” [MURCHO, Desidério. O caráter conceitual
da filosofia.]
O texto acima menciona uma abordagem não-filosófica ao problema de Deus. Quais as diferenças entre uma abordagem não-filosófica e uma abordagem filosófica a respeito de algum problema?
4. (UFMG 2005) “[...] a filosofia não é a revelação feita ao ignorante por quem sabe tudo, mas o diálogo entre iguais que se fazem cúmplices em sua mútua submissão à força da razão e não à razão da força.” [SAVATER, Fernando.
As perguntas da vida.]
A partir da leitura desse trecho e de outros conhecimentos sobre o assunto, REDIJA um texto destacando duas características da atitude filosófica.
5.
VERÍSSIMO, Luis Fernando. As cobras.
O problema “Deus existe?” pode ser abordado por meio da religião ou por meio da filosofia. Qual a diferença entre a abordagem religiosa e a abordagem filosófica a algum problema?
10
Capítulo II
A FILOSOFIA ANTIGA
SANZIO, Rafael. A escola de Atenas.
Filosofia é um saber específico com uma história que já dura mais de 25 séculos; seu surgimento ocorreu na
Grécia antiga por meio dos primeiros filósofos, denominados “pré-socráticos”. Ainda que seja, como já
salientamos, um saber específico, atualmente pode-se entender a Filosofia como uma atitude em relação ao
conhecimento: crítica, sistemática, objetiva. Com isso, e levando-se em conta que em cada época novos
valores são predominantes, novas perspectivas surgem, novas ideias vêm à tona e mais questionamentos são
(re)feitos, os temas abordados pela Filosofia, os pensamentos filosóficos e suas avaliações são constantemente retomados e repensados a fim de que as respostas possam se vincular ao mundo e às novas e sempre recorrentes interrogações que, com o passar do tempo, exigem novas respostas para se adequarem ao mundo em determinada época.
Por conseguinte a Filosofia terá, no correr dos séculos, um conjunto de inquietações, interesses e indagações que
lhe decorreram de seu nascimento na Grécia; é interessante notar que há pensadores que já afirmaram ser toda a
Filosofia ocidental pós-Grécia (o que inclui o pensamento filosófico atual) uma mera “nota de rodapé” que meramente pondera e aprofunda aquilo que os gregos originalmente já pensaram. Seja como for, a história que possui a Filosofia coloca em perspectiva o conhecimento filosófico e apresenta as ideias e os pensadores que, desde os primórdios
do pensamento filosófico, fundamentam o conhecimento que temos (até) hoje. E como toda história tem um início,
vejamos mais detidamente o primeiro período da Filosofia — período que, vale lembrar, faz parte de uma divisão
temporal meramente didática.
A Filosofia antiga, assim, é o período compreendido entre o século VI a.e.c. e o século VI e.c., situando-se entre o
surgimento da Filosofia propriamente dita e a queda do Império Romano. O período antigo se inicia quando alguns
pensadores gregos sentiram a necessidade de esclarecer o mundo por meio de explicações reais, coerentes, sem buscar explicação no pensar mitológico, no incompreensível; dessa feita, esses primeiros pensadores introduziram uma
nova forma de analisar e compreender o mundo e seus fenômenos, descartando as fantasias ilógicas dos mitos.
Com o início dessa indagação a respeito da racionalidade do mundo, os primeiros estudiosos partiram em busca
de uma compreensão racional de sua natureza. No entanto, não foi repentina a passagem da consciência mítica e
religiosa para a consciência racional e filosófica; houve um momento em que esses dois tipos de consciência coexistiram na sociedade grega. Ainda assim, é válido ressaltar que a busca pela verdade, pela qual os primeiros filósofos
trabalharam, refletiu
A
o processo social, político e cultural por que passavam as ilhas gregas nesse período, e a Filosofia ganhou força quando se
fortaleceu a superação da interpretação mítica rumo ao pensamento racional. [CHAUÍ, Marilena. Convite à filosofia.]
Como importante característica da Filosofia antiga, podemos apontar que seu conteúdo procurou compreender as
coisas a partir da totalidade, ou seja, abranger toda a realidade,
sem dividi-la em pormenores e sem abandonar alguma parte desse todo. Isso quer dizer que, diversamente às explicações religiosas ou míticas, os filósofos deveriam fundar suas observações e seus argumentos sobre uma base resistente, a saber, o raciocínio lógico: enquanto o mito e a religião buscam compreender o mundo através da crença e da narrativa, a Filosofia vai
fundamentar suas explicações na Razão (logos). Este é o seu método, que, aliás, foi herdado por quase todas as ciências que
conhecemos hoje. [Idem.]
11
Agora, vejamos mais a fundo quais foram as características dos quatro grandes períodos da Filosofia grega, nos
quais seu conteúdo passa por algumas variações e se enriquece. São eles: “período pré-socrático”, “período socrático”, “período sistemático” e “período helenístico”.
No período pré-socrático ou cosmológico (final do século VII a.e.c. ao final do século V a.e.c.; denominado “présocrático” porque tal divisão didática tem como referência a vida do filósofo ateniense Sócrates [469-399]), ainda que
recente a Filosofia se ocupa basicamente com a origem do mundo e as causas das transformações na Natureza (physis). Assim, como principais características da cosmologia podemos citar o fato de ter sido um período no qual prevaleceu a tentativa de explicar, racional e sistematicamente, a origem, a ordem e as transformações pelas quais passava
a Natureza.
Com isso, se tivermos em mente que os seres humanos fazem parte da natureza, fica claro que, ao explicá-la, a Filosofia também explica a origem e as transformações pelas quais passam os seres humanos; isso porque os primeiros
pensadores entendiam que todos os seres, além de serem originados e de serem mortais, são seres em ininterrupta
transformação (o saudável adoece, o doente se cura, o novo envelhece etc.). Dessa maneira, existem mudanças de
qualidade (o frio esquenta, o quente esfria, a noite se torna dia, o dia se torna noite etc.) e de quantidade (o pequeno
cresce e fica grande, o grande diminui e fica pequeno, o longe fica perto se eu for até ele etc.). Logo, o mundo estaria
em contínua mudança, sem por isso perder sua ordem, sua estabilidade e sua forma. Nesse sentido, cada filósofo
escolheu determinada physis, isto é, encontrou motivos válidos e razões possíveis para fundamentar
qual era o princípio eterno e imutável que está na origem da Natureza e de suas transformações: Tales de Mileto, por exemplo,
dizia que o princípio era a água ou o úmido; Anaximandro considerava que era o ilimitado sem qualidades definidas; Anaxímenes, que era o ar ou o frio; Heráclito afirmou que era o fogo; Leucipo e Demócrito disseram que eram os átomos etc. [Idem.]
Em seguida, no período socrático ou antropológico (final do século V a.e.c. e todo o século IV a.e.c.), a Filosofia
investigou as questões humanas, ou seja, a Política, a Ética e as técnicas. Isso se deveu porque, com o desenvolvimento das cidades, do comércio, dentre outros fatores, Atenas tornou-se o centro da vida cultural, social e política da
Grécia. Nesse período assistiu-se ao momento de maior florescimento da democracia; esta possuía, entre outras, duas
características de grande importância para o futuro da Filosofia:
[...] em primeiro lugar, a democracia afirmava a igualdade de todos os homens adultos perante as leis e o direito de todos de
participar diretamente do governo da cidade, da polis; em segundo lugar, e como consequência, a democracia, sendo direta e
não por eleição de representantes, garantia a todos a participação no governo, e os que dele participavam tinham o direito de
exprimir, discutir e defender em público suas opiniões sobre as decisões que a cidade deveria tomar. Surgia, assim, a figura
política do cidadão. Devemos observar, contudo, que estavam excluídos da cidadania o que os gregos chamavam de dependentes (mulheres, escravos, crianças e velhos), além dos estrangeiros. Assim, para conseguir que a sua opinião fosse aceita nas
assembleias, o cidadão precisava saber falar e ser capaz de persuadir. [Idem.]
Com isso, será vista uma mudança radical na educação grega. Se tivermos em mente que o ideal da educação dessa época era a formação de bons cidadãos, a nova educação, assim, estabeleceu como modelo ideal a formação do
bom orador, ou seja, aquele que soubesse falar, discorrer em público e persuadir os outros na política. Nesse meio,
com o objetivo de dar aos jovens esse tipo de educação, surgiram os denominados “sofistas”, considerados os primeiros filósofos do período socrático. Protágoras de Abdera (480-410), um dos mais importantes do período, é autor de
uma frase que identifica o ofício dos sofistas; disse ele que “o homem é a medida de todas as coisas”; com isso, fugiu-se daquela ideia inicial de neutralidade do conhecimento, pois quando adotamos o homem como medida de tudo,
é inevitável que os pensamentos ficarão mais distantes do ideal filosófico de imparcialidade.
Os sofistas, não obstante, afirmavam que os ensinamentos dos filósofos cosmologistas anteriores estavam abarrotados com contradições e erros que não tinham serventia para a vida da polis (a cidade no âmbito político); apresentavam-se, ainda, como mestres de retórica e oratória, garantindo ser possível ensinar aos jovens o procedimento para
que fossem bons cidadãos. Ensinado pelos sofistas, esse procedimento denominado “persuasão”, servia para que os
alunos pudessem defender uma opinião X, bem como a opinião contrária, não-X, de maneira que, numa discussão
política, tivessem fortes argumentos contra ou a favor de uma opinião, assim sendo possível que ganhassem a discussão. Os sofistas, assim,
aceitam a validade das opiniões e das percepções sensoriais e trabalham com elas para produzir argumentos de persuasão,
enquanto Sócrates e Platão consideram as opiniões e as percepções sensoriais, ou imagens das coisas, como fonte de erro,
mentira e falsidade, formas imperfeitas do conhecimento que nunca alcançam a verdade plena da realidade. [Idem.]
Para o já citado Sócrates, considerado o patrono da Filosofia, os sofistas corrompiam a mente dos jovens, já que
defendiam qualquer ideia, se isso fosse vantajoso, e faziam com que o erro e a mentira tivessem a mesma importância
que a verdade. Todavia, houve consenso em um ponto: Sócrates concordava com os sofistas em relação aos filósofos
cosmologistas; na época anterior, os filósofos defendiam ideias tão contrárias entre si (a partir de physis diferentes)
que não poderiam ser uma fonte segura para o conhecimento verdadeiro. Com isso, Sócrates propunha que, antes de
conhecer a Natureza e antes de persuadir os demais, cada homem deveria, antes de tudo, conhecer a si mesmo; o
autoconhecimento foi, assim, elevado ao patamar de necessidade filosófica principal.
Sócrates, assim, fazia perguntas sobre as ideias, sobre os valores nos quais os gregos acreditavam e que julgavam conhecer.
Suas perguntas deixavam os interlocutores embaraçados, irritados, curiosos, pois, quando tentavam responder ao célebre “o
que é?”, descobriam, surpresos, que não sabiam responder e que nunca tinham pensado em suas crenças, seus valores e suas
ideias. Mas o pior não era isso. O pior é que as pessoas esperavam que Sócrates respondesse por elas ou para elas, que sou-
12
besse as respostas às perguntas, como os sofistas pareciam saber, mas Sócrates, para desconcerto geral, dizia: “Eu também
não sei, por isso estou perguntando” — donde a famosa expressão atribuída a ele: “Sei que nada sei”. [Idem.]
Visto desse ângulo, podemos admitir que a consciência da própria ignorância é o ponto principal do período socrático. E o que buscava Sócrates?
Procurava a definição daquilo que uma coisa, uma ideia, um valor é verdadeiramente. Procurava a essência verdadeira da
coisa, da ideia, do valor. Procurava o conceito e não a mera opinião que temos de nós mesmos, das coisas, das ideias e dos valores. Qual a diferença entre uma opinião e um conceito? A opinião varia de pessoa para pessoa, de lugar para lugar, de época para época. É instável, mutável, depende de cada um, de seus gostos e preferências. O conceito, ao contrário, é uma verdade intemporal, universal e necessária que o pensamento descobre, mostrando que é a essência universal, intemporal e necessária de alguma coisa. Por isso, Sócrates não perguntava se tal ou qual coisa era bela — pois nossa opinião sobre ela pode variar — e sim: O que é a beleza? Qual é a essência ou o conceito do belo? Do justo? Do amor? Da amizade? Sócrates perguntava: Que razões rigorosas você possui para dizer o que diz e para pensar o que pensa? Qual é o fundamento racional daquilo
que você fala e pensa? Ora, as perguntas de Sócrates se referiam a ideias, valores, práticas e comportamentos que os atenienses julgavam certos e verdadeiros em si mesmos e por si mesmos. Ao fazer suas perguntas e suscitar dúvidas, Sócrates os fazia
pensar não só sobre si mesmos, mas também sobre a polis. Aquilo que parecia evidente acabava sendo percebido como duvidoso e incerto. [Idem.]
E se levarmos em consideração que aqueles que detêm o poder temem o pensamento (pois o poder é mais forte se
ninguém pensar, se todos aceitarem, sem reclames, as coisas como elas são, ou melhor, como nos dizem e nos fazem
acreditar que elas são), para os poderosos de Atenas Sócrates tornara-se um perigo, pois fazia a juventude pensar; por
isso, eles o acusaram e o condenaram a morte por “desrespeitar os deuses, corromper os jovens e violar as leis”.
DAVID, Jacques-Louis. A morte de Sócrates.
Ainda que não tenha escrito nada sobre suas ideias, o que sabemos a respeito de Sócrates e de seus pensamentos
encontra-se espalhado nas obras de seus vários discípulos, entre os quais Platão (428-347) foi o mais importante. Se
reunirmos o que esse filósofo escreveu sobre Sócrates, além da exposição de seus próprios pensamentos, poderemos
apresentar como características gerais do período socrático o que se segue:
Nesse período, a Filosofia se volta para as questões humanas no plano da ação, dos comportamentos, das ideias, das crenças,
dos valores e, portanto, se preocupa com as questões morais e políticas; o ponto de partida da Filosofia torna-se a confiança
no pensamento ou no homem como um ser racional, capaz de conhecer-se a si mesmo e, portanto, capaz de refletir; a preocupação dos filósofos se volta para estabelecer procedimentos que garantam que encontremos a verdade, isto é, o pensamento
deve oferecer a si mesmo caminhos próprios, critérios próprios e meios próprios para saber o que é o verdadeiro e como alcançá-lo em tudo o que investiguemos; é feita, pela primeira vez, uma separação radical entre, de um lado a opinião e as imagens das coisas, trazidas pelos nossos órgãos dos sentidos, nossos hábitos, pelas tradições, pelos interesses, e, de outro lado,
as ideias. [Idem.]
Platão, nesse contexto, desenvolveu a noção de que o ser humano está em contato permanente com duas espécies
de realidade: a inteligível e a sensível. A realidade inteligível é uma realidade imutável; já a realidade sensível aborda
todas as coisas que nos afetam os sentidos, sendo realidades mutáveis, dependentes, representações (ou cópias imperfeitas) da realidade inteligível. Tal compreensão é conhecida pelo nome de “teoria das Ideias”, constituindo-se em
uma maneira de assegurar a possibilidade do conhecimento pelo homem. Para Platão, o mundo percebido pelos sentidos é uma mera, imperfeita reprodução do mundo das Ideias: cada objeto que existe e é percebido pelos homens
participa, junto com todos os outros objetos de seu conjunto, de uma Ideia perfeita acerca desse objeto. Uma cadeira,
por exemplo, terá determinadas características (tamanho, formato, cor etc); outra cadeira terá outras características
diferentes, mas ainda será uma cadeira, tanto quanto a outra; assim, aquilo que faz com que as duas cadeiras sejam
cadeiras é, segundo Platão, a Ideia de cadeira, ou seja, uma abstração perfeita que esgota todas as possibilidades de
algo ser cadeira.
13
Uma das condições para a indagação ou investigação acerca das Ideias é que não estamos em estado de completa ignorância
sobre elas. Do contrário, não teríamos nem o desejo nem o poder de procurá-las. Em vista disso, é uma condição necessária,
para tal investigação, que tenhamos em nossa alma alguma espécie de conhecimento ou lembrança de nosso contato com as
Ideias (contato esse ocorrido antes do nosso próprio nascimento) e nos recordemos das Ideias ao vê-las reproduzidas palidamente nas coisas. [Idem.]
Vejamos agora o próximo período da Filosofia antiga, o período sistemático (final do século IV a.e.c. ao final do
século III a.e.c.). Nele, os filósofos tencionaram reunir e sistematizar tudo o que já havia sido pensado nos períodos
anteriores; ademais, os filósofos desse período interessaram-se, sobretudo, em mostrar que tudo poderia ser objeto de
estudo para o conhecimento filosófico, desde que o pensamento, as leis que o regem e as comprovações necessárias
estivessem estabelecidas com segurança, a fim de oferecer critérios adequados para se fundamentar a ciência e se
alcançar a verdade.
Discípulo de Platão e principal nome do período, Aristóteles (384-322) apresentou em sua obra um verdadeiro
apanhado de todo o saber produzido e acumulado pelos gregos, considerando essa totalidade de saberes como sendo a
Filosofia, que foi considerada, nesse período, não como um saber específico sobre algum assunto, mas como uma
maneira de conhecer todas as coisas. A Filosofia, para Aristóteles, pretendia não apenas reconsiderar, mas também
corrigir os erros e as imperfeições que os pensamentos anteriores trouxeram. Discordando de seu mestre, Aristóteles
defendeu a existência de um único mundo (e não de dois, como entendia Platão). Que mundo seria esse? Exatamente
este em que vivemos. Assim, Aristóteles rejeitou o mundo inteligível, pois argumentou que tal mundo estaria além de
nossa experiência sensível, de modo que não poderia ser conhecido pelos homens.
Por fim, no período helenístico ou greco-romano (final do século III a.e.c. até o século VI E.C.),
a amplidão do Império Romano, a presença crescente de religiões orientais no Império, os contatos comerciais e culturais entre ocidente e oriente fizeram aumentar os contatos dos filósofos helenistas com a sabedoria oriental. Podemos falar, nesse último período da Filosofia antiga, de uma “orientalização” da Filosofia, sobretudo nos aspectos místicos e religiosos. [Idem.]
Essa última subdivisão da Filosofia antiga é constituída por grandes e complexos sistemas ou doutrinas, ou seja,
amplas explicações sobre a Natureza, o homem, as relações entre ambos e a relação destes com a ideia metafísica de
divindade, de maneira que, aos poucos, e com o surgimento, crescimento e fortalecimento da religião cristã, a Filosofia passou a especular com maior atenção a natureza de Deus; essa especulação terá seu ápice, como veremos no
próximo capítulo, no decorrer do período subsequente (a saber, a época medieval).
Como vimos, o objetivo da Filosofia antiga em todos os seus períodos mostrou-se bastante pretensioso: os primeiros pensadores tinham por objetivo conhecer racionalmente o mundo, contemplar a verdade. É interessante notar
que tal ambição ainda se faz presente; como exemplo, temos as ciências atuais que acreditam na possibilidade de um
conhecimento verdadeiramente objetivo da realidade, de um conhecimento neutro e imparcial. Eis uma forte herança
da Filosofia grega como um todo, que se propôs um difícil desafio: observar, conhecer e distinguir a realidade a partir
de uma perspectiva abstrata, racional, da qual homem algum havia, antes, cogitado realizar. Essa é, pois, a principal
marca da Filosofia antiga — marca que, nunca é demais lembrar, influenciou fortemente o pensamento ocidental e a
organização política, social e intelectual daquela época até os dias atuais.
Leituras complementares
Imaginemos uma caverna subterrânea onde, desde a infância, geração após geração, seres humanos estão aprisionados. Suas pernas e seus pescoços estão algemados de tal modo que são forçados a permanecer sempre no mesmo
lugar e a olhar apenas para frente, não podendo girar a cabeça nem para trás nem para os lados. A entrada da caverna
permite que alguma luz exterior ali penetre, de modo que se possa, na semi-obscuridade, enxergar o que se passa no
interior.
A luz que ali entra provém de uma imensa e alta fogueira externa. Entre ela e os prisioneiros — no exterior, portanto — há um caminho ascendente ao longo do qual foi erguida uma mureta, como se fosse a parte fronteira de um
palco de marionetes. Ao longo dessa mureta-palco, homens transportam estatuetas de todo tipo, com figuras de seres
humanos, animais e todas as coisas.
Por causa da luz da fogueira e da posição ocupada por ela, os prisioneiros enxergam na parede do fundo da caverna as sombras das estatuetas transportadas, mas sem poderem ver as próprias estatuetas, nem os homens que as
transportam.
Como jamais viram outra coisa, os prisioneiros imaginam que as sombras vistas são as próprias coisas. Ou seja,
não podem saber que são sombras, nem podem saber que são imagens (estatuetas de coisas), nem que há outros seres
humanos reais fora da caverna. Também não podem saber que enxergam porque há a fogueira e a luz no exterior e
imaginam que toda luminosidade possível é a que reina na caverna.
Que aconteceria, indaga Platão, se alguém libertasse os prisioneiros? Que faria um prisioneiro libertado? Em primeiro lugar, olharia toda a caverna, veria os outros seres humanos, a mureta, as estatuetas e a fogueira. Embora dolorido pelos anos de imobilidade, começaria a caminhar, dirigindo-se à entrada da caverna e, deparando com o caminho
ascendente, nele adentraria.
Num primeiro momento, ficaria completamente cego, pois a fogueira na verdade é a luz do sol e ele ficaria inteiramente ofuscado por ela. Depois, acostumando-se com a claridade, veria os homens que transportam as estatuetas e,
prosseguindo no caminho, enxergaria as próprias coisas, descobrindo que, durante toda sua vida, não vira senão
14
sombras de imagens (as sombras das estatuetas projetadas no fundo da caverna) e que somente agora está contemplando a própria realidade.
Libertado e conhecedor do mundo, o prisioneiro regressaria à caverna, ficaria desnorteado pela escuridão, contaria aos outros o que viu e tentaria libertá-los.
Que lhe aconteceria nesse retorno? Os demais prisioneiros zombariam dele, não acreditariam em suas palavras e,
se não conseguissem silenciá-lo com suas caçoadas, tentariam fazê-lo espancando-o e, se mesmo assim, ele teimasse
em afirmar o que viu e os convidasse a sair da caverna, certamente acabariam por matá-lo. Mas, quem sabe, alguns
poderiam ouvi-lo e, contra a vontade dos demais, também decidissem sair da caverna rumo à realidade.
O que é a caverna? O mundo em que vivemos. Que são as sombras das estatuetas? As coisas materiais e sensoriais que percebemos. Quem é o prisioneiro que se liberta e sai da caverna? O filósofo. O que é a luz exterior do sol? A
luz da verdade. O que é o mundo exterior? O mundo das ideias verdadeiras ou da verdadeira realidade. Qual o instrumento que liberta o filósofo e com o qual ele deseja libertar os outros prisioneiros? A dialética. O que é a visão do
mundo real iluminado? A Filosofia. Por que os prisioneiros zombam, espancam e matam o filósofo (Platão está se
referindo à condenação de Sócrates à morte pela assembléia ateniense)? Porque imaginam que o mundo sensível é o
mundo real e o único verdadeiro.
CHAUÍ, Marilena. “O mito da caverna”. In: Convite à filosofia.
φ
O amor pela verdade que nos conduzirá a muitas perigosas aventuras, essa famosíssima veracidade de que todos
os filósofos sempre falaram respeitosamente — quantos problemas já nos colocou! E problemas singulares, malignos,
ambíguos! Apesar da velhice da estória, parece que acaba de acontecer. Se acabássemos, por esgotamento, sendo
desconfiados e impacientes, que haveria de estranho? É estranhável que essa esfinge nos tenha levado a nos formular
toda uma série de perguntas? Quem afinal vem aqui interrogar-nos? Que parte de nós tende “para a verdade?” Detivemo-nos ante o problema da origem dessa vontade, para ficar em suspenso diante de outro problema ainda mais
importante? Interrogamo-nos sobre o valor dessa vontade. Pode ser que desejamos a verdade, mas por que afastar o
não verdadeiro ou a incerteza e até a ignorância? Foi a problema da validade do verdadeiro que se colocou frente a
nós ou fomos nós que o procuramos? Quem é Édipo aqui? E quem é a Esfinge? Encontramo-nos frente a uma encruzilhada de questões e problemas. E parece, afinal de contas, que não foram colocados até agora, que fomos os primeiros a percebê-los, que nos atrevemos a confrontá-los, já que implicam um risco, talvez a maior dos riscos.
NIETZSCHE, Friedrich. §1. In: Além do bem e do mal.
φ
A sofística, ocorrida nos últimos cinquenta anos do século V, não se configura como uma doutrina, mas antes
como uma forma de ensinar. Os sofistas são professores que vão de cidade em cidade procurando audiência e que,
por um preço conveniente, ensina-a a se destacar, através de lições de ostentação e de vários cursos e métodos cujo
objetivo é tornar vencedora uma tese que eles querem que seja aceita. A pesquisa e promulgação da verdade é substituída pela busca do sucesso, baseando-se na arte de convencer, de persuadir e de seduzir. É a época onde a vida intelectual — cujo centro está na Grécia continental, toma a forma de competição e de jogo — uma forma polêmica,
muito familiar à vida grega. Tratam-se apenas de teses defendidas ou combatidas pelos antagonistas, os quais um juiz
soberano — quase sempre o publico — determina um preço. Aristófanes nos mostra este debate entre a tese justa e
injusta acontecendo. “Quem é?”, pergunta o justo. — Uma tese. — Certo, mas inferior à minha. — Você se pretende
superior a mim, mas eu tenho a vitória. — Que habilidade você tem, então? — Invento razões novas.” Eurípedes
descreve em Antíope este debate sobre o ideal de vida entre o amigo das musas e o homem político. Platão por outro
lado, nos mostra Sócrates se esquivando destas competições; no Protágoras Hipías tenta, em vão, promover um debate deste gênero entre Sócrates e Protágoras; no Górgias Cálicles, que mais adiante pronuncia um discurso em favor da
justiça natural, se queixa que Sócrates quebra as regras do jogo ao não respondê-lo e fazendo um outro discurso.
Havia ali uma preocupação da audiência, que conhecíamos mal até agora. O filósofo não revela mais a verdade, mas
a sugere e se submete de antemão ao veredicto do auditório. Este é um aspecto que permanece: depois da época dos
sofistas, tenta-se a missão de definir o filósofo em oposição ao orador, ao político, ao sofista — ou seja, a todos que
se dirigem ao público.
Nestas condições, o principal valor intelectual é a erudição que o homem de posse de todos os conhecimentos úteis a seu propósito reúne e a virtuosidade que lhe permite escolher seus temas com o propósito de apresentá-los de
maneira cativante. Daí duas características essenciais do sofista: por um lado são os técnicos que se vangloriam de
conhecer e ensinar todas as artes úteis ao homem; por outro são mestres da retórica que ensinam como capturar a
simpatia da platéia. [...]
Mas é sobretudo na política que os sofistas afirmam o poder e a autonomia do homem: a lei é uma invenção humana e, em certa medida, artificial e arbitrária. É o que indica o fato de legisladores da época, seja em Atenas, seja
nas colônias, retomam a todo instante o trabalho da constituição desde o início; Protágoras dá as leis à Turios, assim
como Parmênides havia feito com Eléia. A lei, como uma obra artificial, se opõe, então, à natureza. Existem, é claro,
leis não escritas, costumes tradicionais que tem um valor não-religioso, mas não pesam em nada na ponderada obra
do legislador. Este é o ponto de vista de Antífon, o sofista, nos fragmentos que foram recentemente descobertos. Ele
não se exime de opor a justiça artificial das leis à justiça natural. Por exemplo a lei, ao obrigar o homem a declarar a
verdade no tribunal, obriga-nos com frequência a prejudicar quem não nos fez mal algum, ou seja, contradizer o
primeiro preceito da justiça. Mas neste caráter convencional das leis, Antífon parece ver uma superioridade.
15
Este movimento de ideias tão importante teve um desenlace triste. Desemboca, no início do século IV, no cinismo
político ou na virtuosidade. De um lado o cinismo político dos aristocratas atenienses, Crítias e Alcíbiades, que é
relatado com frequência na História da Guerra do Peloponeso de Tucídides e que Platão imortalizou no Cálicles do
Górgias: é a depravação política e moral de Cálicles, para quem o poder é apenas uma maneira de satisfazer seus
apetites, que leva ao ensino de retórica por Górgias. A outra consequência é a pura virtuosidade, aquela que já se
encontrava no tratado do Górgias sobre o não-ser, onde, servindo-se dos métodos dialéticos do eleatismo, ele demonstra que nada existe; ou, se existe, é incognoscível; ou, se é cognoscível, é impossível de ser comunicado. Importância caracterizada pela importância pela importância atribuída à eloquência, ao ensinamento retórico de Górgias,
aos trabalhos de gramática geral de Protágoras e às pesquisas de Pródico sobre os sinônimos. Virtuosidade que encontra sua fonte em pequenas obras como os discursos duplos que resumem esquematicamente duas teses antilógicas
possíveis quando se aborda as questões morais. Enfim, virtuosidade que tem sua última manifestação na arte da disputa ou da erística, que Platão ridiculariza cruelmente no Eutidemo. A erística tem meios muito fáceis de submeter o
adversário por dois ou três princípios bastante simples tais como: o erro é impossível, qualquer refutação é impossível.
Apesar do talento intelectual dos sofistas, estes eram os resultados de uma vida intelectual dirigida unicamente
para o sucesso. No entanto este movimento, assim como seus precedentes, guarda algo de positivo: o naturalismo
iônico, o racionalismo da Grande Grécia [Megálé Hellás, Magna Grécia], o espírito religioso de Empédocles e dos
Pitagóricos, o humanismo dos sofistas, veremos tudo isso se reunir no mais respeitado dos filósofos gregos, Platão.
BRÉHIER, Émile. História da Filosofia.
φ
Deve-se levar a História a sério ou assisti-la como espectador? Ver nela um esforço na direção de uma meta ou o
jogo de uma luz que se aviva e empalidece sem necessidade nem razão? A resposta depende de nosso grau de ilusão
sobre o homem, de nossa curiosidade em adivinhar amaneira como se resolverá essa mistura de valsa e de matadouro
que compõe e estimula seu devir. Há um mal do século, que é apenas a doença de uma geração; há outro que resulta
de toda a experiência histórica e que se impõe como única conclusão para os tempos vindouros. Trata-se do “vago na
alma”, da melancolia do “fim do mundo”. Tudo muda de aspecto, até o sol, tudo envelhece, até a desgraça...
Incapazes da retórica, somos os românticos da decepção clara. Hoje, Werther, Manfredo, René conhecedores de
sua doença, a ostentariam sem pompa. Biologia, Fisiologia, Psicologia — nomes grotescos que, ao suprimir a ingenuidade de nosso desespero e introduzir a análise em nossos cantos, nos fazem desprezar a declamação! Filtradas pelos
tratados, nossas doutas amarguras explicam nossas vergonhas e classificam nossos frenesis. Quando a consciência
chegar a inclinar-se sobre todos os nossos segredos, quando for evacuado de nossa desgraça o último vestígio de
mistério, guardaremos ainda um resto de febre e de exaltação para contemplar a ruína da existência e da poesia?
Sentir o peso da história, o fardo do devir e esse abatimento sob o qual se dobra a consciência quando considera o
conjunto e a inanidade dos acontecimentos passados ou possíveis... A nostalgia, em vão, invoca um impulso ignorante das lições que se depreendem de tudo o que foi; há um cansaço, para o qual o próprio futuro é um cemitério, um
cemitério virtual como tudo o que espera chegar a ser. Os séculos tornaram-se onerosos e pesam sobre cada instante.
Estamos mais apodrecidos que todas as épocas, mais decompostos que todos os impérios. Nosso esgotamento interpreta a história, nossa prostração nos faz escutar os estertores das nações. Como atores cloróticos, nos preparamos
para interpretar os papéis de parte supérflua no tempo castigado: o pano de busca do universo está roído pelas traças
e, através de seus furos, só se vêem máscaras e fantasmas [...].
CIORAN, Emil Michel. “Rostos da decadência”. In: Breviário de decomposição.
Cogito, logo aprendo
1. (UEL 2003) “Zeus ocupa o trono do universo. Agora o mundo está ordenado. Os deuses disputaram entre si,
alguns triunfaram. Tudo o que havia de ruim no céu etéreo foi expulso, ou para a prisão do Tártaro ou para a Terra,
entre os mortais. E os homens, o que acontece com eles? Quem são eles?” [VERNANT, Jean-Pierre. O universo, os deuses,
os homens.]
O texto acima é parte de uma narrativa mítica. Considerando que o mito pode ser uma forma de conhecimento,
assinale a alternativa correta.
a) A verdade do mito obedece a critérios empíricos e científicos de comprovação.
b) O conhecimento mítico segue um rigoroso procedimento lógico-analítico para estabelecer suas verdades.
c) As explicações míticas constroem-se de maneira argumentativa e autocrítica.
d) O mito busca explicações definitivas acerca do homem e do mundo, e sua verdade independe de provas.
e) A verdade do mito obedece a regras universais do pensamento racional, tais como a lei de não-contradição.
2. (UEL 2004) “Mais que saber identificar a natureza das contribuições substantivas dos primeiros filósofos é
fundamental perceber a guinada de atitude que representam. A proliferação de óticas que deixam de ser endossadas
acriticamente, por força da tradição ou da imposição religiosa, é o que mais merece ser destacado entre as propriedades que definem a filosoficidade. [OLIVA, Alberto; GUERREIRO, Mario. Présocráticos: a invenção da filosofia.]
Assinale a alternativa que apresenta a guinada de atitude que o texto afirma ter sido promovida pelos primeiros filósofos.
a) A aceitação acrítica das explicações tradicionais relativas aos acontecimentos naturais.
b) A discussão crítica das ideias e posições, que podem ser modificadas ou reformuladas.
16
c) A busca por uma verdade única e inquestionável, que pudesse substituir a verdade imposta pela religião.
d) A confiança na tradição e na imposição religiosa como fundamentos para o conhecimento. oligarquia.
e) A desconfiança na capacidade da razão em virtude da proliferação de óticas conflitantes entre si.
3. (UEL 2004) “Entre os físicos da Jônia, o caráter positivo invadiu de chofre a totalidade do ser. Nada existe que
não seja natureza, physis. Os homens, a divindade, o mundo formam um universo unificado, homogêneo, todo ele no
mesmo plano: são as partes ou os aspectos de uma só e mesma physis que põem em jogo, por toda parte, as mesmas
forças, manifestam a mesma potência de vida. As vias pelas quais essa physis nasceu, diversificou-se e organizou-se
são perfeitamente acessíveis à inteligência humana: a natureza não operou no começo de maneira diferente de como o
faz ainda, cada dia, quando o fogo seca uma vestimenta molhada ou quando, num crivo agitado pela mão, as partes
mais grossas se isolam e se reúnem.” [VERNANT, Jean-Pierre. As origens do pensamento grego.]
Com base no texto, assinale a alternativa correta.
a) Para explicar o que acontece no presente é preciso compreender como a natureza agia no começo, ou seja, no
momento original.
b) A explicação para os fenômenos naturais pressupõe a aceitação de elementos sobrenaturais.
c) O nascimento, a diversidade e a organização dos seres naturais têm uma explicação natural e esta pode ser
compreendida racionalmente.
d) A razão é capaz de compreender parte dos fenômenos naturais, mas a explicação da totalidade dos mesmos está além da capacidade humana.
e) A diversidade de fenômenos naturais pressupõe uma multiplicidade de explicações e nem todas estas explicações podem ser racionalmente compreendidas.
4. (UFMG 2000) Leia o texto.
“Devemos submeter a experiência negativa a uma análise, com a finalidade de apurar o seu significado para o
comportamento filosófico. Através de sua caracterização chegaremos a compreender que, através dela, e só através
dela, pode o homem abandonar a postura dogmática, e aceder, consequentemente, à filosofia. Vale dizer que a experiência da negatividade é um momento do processo dialético que leva o homem a filosofar. Desta dialética, o primeiro momento é constituído pela afirmação dogmática do mundo; o segundo é a experiência da negatividade; e o terceiro, é o ato de assumir a filosofia como tarefa.” [BORNHEIM, G. Introdução ao filosofar.]
Esse texto refere-se a três momentos apontados pelo autor como constitutivos da descoberta do filosofar. EXPLIQUE um desses momentos.
5. (UFMG 2001) Leia estes trechos:
TRECHO 1. Terra primeiro gerou igual a si mesma
Céu constelado, a fim de cobri-la toda ao redor
e de que fosse aos deuses venturosos sede segura para sempre.
E gerou altas montanhas, belas moradas das deusas
Ninfas que habitam as montanhas frondosas.
E gerou também a infecunda planície impetuosa de ondas,
o Mar, sem desejoso amor.
TRECHO 2. “A água envolve a terra, tal como ao redor daquela encontra-se a esfera de ar e, ao redor desta, a esfera dita de fogo [...] por outro lado, o sol, movendo-se do modo como ele o faz, produz as mudanças da geração e da
corrupção e, por causa disto, a água mais leve e mais doce é aspirada todo dia e, uma vez dividida e vaporizada, é
transportada para a alta atmosfera; lá, ela é novamente condensada por causa do frio e desce então, mais uma vez,
para a terra. E isto, como dissemos anteriormente, a natureza sempre quer produzir deste modo.” [ARISTÓTELES. Meteorológica.]
Os dois trechos caracterizam formas distintas de conhecimento.
a. IDENTIFIQUE o tipo de conhecimento representado em cada um deles.
b. CARACTERIZE os dois tipos de conhecimento identificados.
17
Capítulo III
A FILOSOFIA MEDIEVAL
LAERTE. “Piratas do Tietê”. In: Folha de S. Paulo (28 de junho de 2009).
pós o período denominado “Filosofia antiga” sucedeu, na Europa, o predomínio dos valores religiosos (sobretudo em virtude do fortalecimento do Cristianismo em meio ao Império Romano). Nesse ínterim, o significativo monopólio intelectual praticado pela Igreja estabeleceu uma cultura de configuração fortemente teocêntrica; na Idade Média sobreveio uma intensa união entre as crenças religiosas e o conhecimento clássico que
surgiu na Grécia antiga. Não por acaso, os mais proeminentes pensadores da época medieval apresentaram
grande preocupação em evidenciar e discutir temas diretamente conectados ao desenvolvimento e à compreensão das
doutrinas cristãs.
Assim, uma das preocupações fundamentais dos filósofos medievais foi a de fornecer argumentos racionais para
justificar as denominadas “verdades reveladas” da Igreja Cristã, bem como da religião Islâmica, tais como a verdade
revelada da existência de Deus, da imortalidade da alma etc. Apoiando essa perspectiva, Tertuliano (155-222) apontava, já no século III, que o conhecimento não poderia ser válido e verdadeiro se não estivesse ligado de algum modo
aos valores cristãos admitidos como corretos; outros pensadores defendiam que as verdades contidas no pensamento
cristão não poderiam estar subordinadas ao pensamento racional; não obstante, outros pensadores medievais eram
contrários a essa completa oposição entre razão e fé. Nesse meio, um dos mais importantes representantes dessa
conciliação entre fé e razão foi o filósofo Santo Agostinho (354-430), que defendeu a busca por explicações de cunho
racional que justificassem os dogmas cristãos.
Contudo, as modificações vividas no período da baixa Idade Média promoveriam uma sensata revisão da teologia
de Agostinho: a denominada “Filosofia escolástica” surgiria com a finalidade de promover a harmonização entre o
campo da razão e o campo da fé. Entre seus representantes mais importantes estava o filósofo São Tomas de Aquino
(1227-1274) que, influenciado pela rigidez com a qual a Igreja era organizada, preocupou-se em criar modos de
conhecimento que não se “intimidassem” em relação a nenhum tipo de questionamento. Dessa maneira, o ser humano
teria um papel ativo na produção do conhecimento. Mas, apesar dessa nova perspectiva, a Filosofia escolástica não
conseguirá promover um distanciamento das questões religiosas: ainda que reconhecesse o valor positivo do livrearbítrio humano, a escolástica defendia o papel central que a Igreja, como instituição sagrada, teria na definição dos
caminhos e das atitudes que poderiam levar o homem até a sua salvação.
Vejamos, agora, as três subdivisões do período denominado “Filosofia medieval”: a “Filosofia patrística”, a “Filosofia medieval propriamente dita” e a “Filosofia da Renascença”.
O período da Filosofia patrística (do século I e.c. ao século VII e.c.) caracteriza-se em decorrência dos esforços
de, dentre outros, alguns apóstolos cristãos (principalmente Paulo e João), a fim de conciliar a nova religião (o Cristianismo) com o pensamento filosófico predominante entre os gregos e os romanos. Como se não bastasse, nesse primeiro período da Filosofia medieval os filósofos tomaram para si a tarefa de defenderem a fé cristã frente às inúmeras
e diversas críticas provenientes dos valores morais da Filosofia antiga.
A
Eles [os filósofos patrísticos] representam a primeira tentativa de harmonizar determinados princípios da Filosofia grega (particularmente do Epicurismo, do Estoicismo e do pensamento de Platão) com a doutrina cristã. [...]. Eles não só estavam envolvidos com a tradição cultural helênica como também conviviam com filósofos estóicos, epicuristas, peripatéticos (sofistas), pitagóricos e neoplatônicos. E não só conviviam, como também foram educados nesse ambiente multiforme da Filosofia grega
ainda antes de suas conversões. [SPINELLI, Miguel. Helenização e recriação de sentidos — A Filosofia na época da expansão
do Cristianismo.]
A Filosofia patrística liga-se, portanto, à tarefa da evangelização e da defesa do pensamento cristão; por isso, os
filósofos desse período foram obrigados a introduzir ideias que, aos filósofos greco-romanos, eram desconhecidas: a
ideia de “pecado original”, de “Deus como trindade”, de “criação do mundo em sete dias”, de “juízo final”, de “ressurreição dos mortos” etc. Precisou, sobretudo, explicar como o mal podia existir no mundo, já que, se tudo fora
criado por Deus — que é, em tese, pura perfeição e imaculada bondade — o mal não poderia advir dEle. Dentre tais
filósofos, Santo Agostinho e Boécio (475-524) introduziram, também, a ideia de “homem interior”, ou seja, do livrearbítrio e da consciência moral, pelos quais o homem se tornaria o único responsável pela existência do mal no mundo.
A fim de imporem as ideias cristãs, os filósofos vinculados à Igreja transformaram-nas em verdades reveladas por
Deus, seja por meio da Bíblia ou dos santos, e que, por serem leis divinas, seriam dogmas — ou, em outras palavras,
verdades indiscutíveis e irrefutáveis. Após isso, passa a existir uma distinção (até então desconhecida pelos antigos)
entre verdades reveladas (ou verdades da fé) e verdades da razão (ou verdades humanas), ou seja, entre verdades
18
sobrenaturais e verdades naturais. As verdades sobrenaturais introduziram a noção de conhecimento recebido por
uma graça divina, conhecimento este que seria inquestionável e superior ao simples conhecimento racional. Desse
modo, a grande temática de toda a Filosofia patrística vincula-se à possibilidade de harmonizar razão e fé. E,
a esse respeito, havia três posições principais: 1. Os que julgavam fé e razão irreconciliáveis e a fé superior à razão (diziam
eles: “Creio porque absurdo”). 2. Os que julgavam fé e razão conciliáveis, mas subordinavam a razão à fé (diziam eles:
“Creio para compreender”). 3. Os que julgavam razão e fé irreconciliáveis, mas afirmavam que cada uma delas tem seu campo próprio de conhecimento e não devem misturar-se (a razão se refere a tudo o que concerne à vida temporal dos homens no
mundo; a fé, a tudo o que se refere à salvação da alma e à vida eterna futura). [CHAUÍ, Marilena. Convite à filosofia.]
LAERTE. Piratas do Tietê.
Santo Agostinho, o filósofo mais expressivo do período patrístico, ainda que tenha vivido em uma época na qual
o Cristianismo estava consolidado (apesar de ter, na época, “apenas” quatrocentos anos de existência) e considerado a
verdade irrefutável, entendia que, sem o auxílio da fé, a razão não seria capaz de levar o homem a alcançar a felicidade; isso porque a razão, para Agostinho, serviria como instrumento auxiliar para a fé, elucidando e tornando inteligível aquilo que os homens intuíam mas eram incapazes de fundamentar. Conhecido por “cristianizar” o pensamento de
Platão, Agostinho chegou à conclusão de que o mal era consequência do mau uso do livre-arbítrio por parte dos seres
humanos, não sendo algo proveniente diretamente de Deus. Em suma, não existia mal, e sim ausência de Deus. E
Deus, por sua vez, seria a fonte dos conhecimentos perfeitos, não o homem.
Seria por meio da experiência mística que era possível alcançar a iluminação divina, assim chegando-se às verdades eternas, de modo que, consequentemente, o intelecto humano seria capaz de pensar corretamente acerca dos
assuntos divinos. Vale dizer ainda que Agostinho aliou a Filosofia à História, interpretando a História da humanidade
como o conflito entre a “cidade de Deus” (baseada no amor a Deus e nos valores pregados por Cristo) e a “cidade
humana” (fundada sobre os valores mundanos e imediatos); ambas as cidades estariam presentes na alma humana,
mas no final a Cidade de Deus prevaleceria.
Seguindo a linha do tempo chegamos, pois, no período da Filosofia medieval propriamente dita (de VIII e.c. até
XIV e.c.), período bastante influenciado pelas ideias socráticas e platônicas (conhecidas como “neoplatonismo”)
durante o qual os filósofos ocuparam-se em debater e problematizar as chamadas “questões universais”. É nesse
momento que o pensamento cristão firma-se como “Filosofia cristã”, e que, depois, irá introduzir o que conhecemos
por “teologia”. Mantendo e discutindo os problemas trabalhados pelos filósofos patrísticos, os filósofos medievais
acrescentaram outros e, além de serem influenciados por Platão e Aristóteles, sofreram grande influência da filosofia
de Santo Agostinho. Algo importante de se ter em mente é que, nesse período,
por causa de um método de disputa — teses, refutações, defesas, respostas, conclusões baseadas em escritos de outros autores
—, costuma-se dizer que, na Idade Média, o pensamento estava subordinado ao princípio da autoridade, isto é, uma ideia é
considerada verdadeira se for baseada nos argumentos de uma autoridade reconhecida (Bíblia, Platão, Aristóteles, um papa,
um santo). [CHAUÍ, Marilena. Convite à filosofia.]
Entre os filósofos mais importantes do período estão Santo Anselmo (1033-1109) e São Tomás de Aquino.
Santo Anselmo, em busca de um argumento para provar a existência de Deus e Sua bondade suprema, entendia
que a fé correspondia à verdade. Para ele, a alma humana era imortal, e nenhuma seria privada do bem proveniente do
Ser supremo, ainda que devesse buscá-lo por meio da fé.
ITURRUSGARAI, Adão. In: Folha de S. Paulo (11 de abril de 2009).
Por sua vez, São Tomás de Aquino ficou célebre por ter “cristianizado” Aristóteles (à semelhança do que fez Agostinho com Platão); transformou, assim, o pensamento do sábio grego em um modelo aceitável pela Igreja católica.
Apesar de Aristóteles ter vivido antes do surgimento do Cristianismo, e por sua obra ser original e independente de
19
dogmas, entendeu Tomás de Aquino que o pensador grego estaria em harmonia com o conhecimento contido na
Bíblia. Afirmou, além disso, que o homem poderia conhecer Deus pelos seus efeitos, sendo Ele a causa de todas as
coisas, a última e mais perfeita etapa da escala evolutiva. Mas, apesar de Deus ser a causa de tudo, Ele não agiria
diretamente em suas criações; daí que apenas pelos efeitos de Deus é que o ser humano poderia conhecê-Lo. Em
resumo, para Tomás de Aquino Deus seria a verdade pura, sem falsidade, que existe em si e para si.
Por fim, a Filosofia da Renascença (do século XIV e.c. até o século XVI e.c.) é marcada pela descoberta de algumas obras de Platão que, até então, eram desconhecidas na Idade Média, além de novas obras, também desconhecidas, de Aristóteles. Foi um período marcado por uma efervescência teórica sustentada por fatos como as crises
político-culturais que culminaram em intensas críticas à Igreja católica (que, vale lembrar, resultaram na Reforma
Protestante).
Podem-se citar três linhas de pensamento que predominaram na época. A linha proveniente de Platão e do neoplatonismo destacava a ideia da Natureza como um grande ser vivo; o homem faria parte da Natureza como um microcosmo, e poderia agir sobre ela por meio da alquimia e da astrologia, pois o mundo seria organizado a partir de conexões secretas entre as coisas; o homem poderia, além de distinguir esses vínculos, criar outros (como um deus). A
linha oriunda dos pensadores florentinos valorizava a vida ativa, ou seja, a política, defendendo os ideais republicanos
das cidades italianas contra o poderio dos papas e dos imperadores. Por fim, havia a linha que propunha o ideal do
homem como “artífice de seu próprio destino”, fosse por meio de seus conhecimentos (astrologia, alquimia etc.),
fosse por meio da política, das técnicas (engenharia, arquitetura, medicina, navegação etc.) e das artes (teatro, literatura, pintura, escultura etc.).
LAERTE. “Piratas do Tietê”. In: Folha de S. Paulo (3 de junho de 2009).
Enfim, nesse período assistiu-se a uma efervescência de novas ideias, alimentadas por importantes acontecimentos científicos, políticos, culturais e sociais; tal efervescência garantiu ao homem o conhecimento de novos mares,
novas terras e novos povos, permitindo-lhe uma visão crítica de sua própria sociedade. A Filosofia medieval, como
visto, se por um lado mostrou-se intensa e produtiva em se tratando de trabalhar teoricamente os assuntos metafísicos,
por outro lado descobriu, nesses mesmos assuntos (e no meio social), desordens e confusões para as quais era imprescindível uma transformação brusca de perspectiva. Inicia-se, com isso, a Filosofia moderna.
Leituras complementares
Dizer que a Idade Média vive de textos parece, um lugar comum, que arrasta consigo conotações eminentemente
negativas, já que assistimos hoje ao esquecimento desses mesmos textos, à perda da palavra como referência e à sua
substituição pela imediatez da imagem, integrada na vivência quase inconsciente de um pensamento de exterioridade.
E, no entanto, essa afirmação aponta para um outro modo de ver, decisivamente inserido na dimensão ontológica
e na afirmação de sua prioridade perante a instância cognitiva, que marcou a evolução da história do pensamento até
a modernidade e presidiu à abertura e ao fundamento de um Logos ocidental: grego, helenístico, romano, mas também cristão.
Nesta perspectiva, a Idade Média, na sua especificidade, na complexidade das suas matrizes culturais nunca totalmente unificadas é, a seu modo, a continuidade de um pensamento clássico, na amplitude abrangente de um movimento de translatio studiorum, que foi acolhendo decisivas transformações processuais, num percurso do sagrado ao
profano, da cosmologia à ética, da teologia à antropologia, da antropologia à soteriologia.
Quando o Cristianismo é apresentado ao mundo pagão, busca nele, natural e espontaneamente, os instrumentos e
os esquemas intelectuais para se constituir na sua dimensão metafísica.
Não pode deixar de ser significativo que já na versão grega dos Setenta, os tradutores escolham, para expressar o
hebraico dabar — palavra de Deus revelada aos homens, paralelamente conhecimento e força criadora — o complexo conceito de logos, da tradição filosófica grega. Em sequência, nos textos paulinos e no prólogo do Evangelho de
João, o mesmo conceito exprime, retomando a tradição complexa, o Verbo de Deus, consubstancial ao Pai, que encarnou e se fez homem, como mediador e revelador por excelência.
A Encarnação marca a erupção da eternidade no tempo e a instauração de uma nova ordem que engloba e completa a antiga: a escuta da Palavra divina que utiliza e se expressa em palavras humanas, desencadeia a dimensão
exegética e marca a permanente abertura do texto sagrado que a contém, numa pluralidade incessante de interpretações.
Deste modo, é ainda o conceito de logos que se transforma em instrumento de mediação, possibilitando o encontro entre o cristianismo como religião e a filosofia pagã, porque Cristo, fazendo convergir em si todos os aconteci-
20
mentos, se define como Verdade e Vida, conexionando, de modo definitivo, os fundamentos do conhecimento e da
norma da consciência moral. [...]
Se é certo que no seu longo percurso constituinte, o pensamento medieval tomou sempre como referência inequívoca a palavra revelada na sua dimensão escrita, o seu posicionamento meta-racional nunca limitou ou paralisou a
razão no seu percurso, antes se constituiu como fonte de dinamismo e de motivação. Assim se entende a correlacionação dessa leitura aberta com a busca de outros textos — sagrados ou profanos — com outras religiões — no diálogo ou na polêmica — e até com a natureza — entendida como um “livro” que se quer e pode decifrar.
PACHECO, Maria Cândida Monteiro. A Filosofia medieval e a questão da interpretação — a palavra e os textos — entre a letra e o
espírito.
Cogito, logo aprendo
1. (UFMG 2002) Leia estes trechos:
TRECHO 1. “Não tento, Senhor, penetrar na tua profundeza, porque de modo algum comparo a ela minha inteligência, mas desejo, ao menos, compreender tua verdade, em que meu coração crê e ama. Com efeito, não procuro
compreender para crer, mas creio para compreender.” [SANTO ANSELMO. Proslógion.]
TRECHO 2. “Há, com efeito, duas ordens de verdades que afirmamos de Deus. Algumas são verdades referentes
a Deus e que excedem toda capacidade da razão humana, como, por exemplo, Deus ser trino e uno. Outras são aquelas as quais a razão pode admitir, como, por exemplo, Deus ser, Deus ser uno, e outras semelhantes. Estas os filósofos, conduzidos pela luz da razão natural, provaram, por via demonstrativa, poderem ser realmente atribuídas a Deus.
[...] Embora a supracitada verdade da fé cristã exceda a capacidade da razão humana, os princípios que a razão tem
postos em si pela natureza não podem ser contrários àquela verdade. [AQUINO, Tomás de. Suma contra os gentios.]
A partir das ideias contidas nesses trechos,
a. IDENTIFIQUE as duas formas de conhecimento neles referidas.
b. EXPLIQUE que tipo de relação os autores medievais citados estabelecem entre essas duas formas de conhecimento.
21
Capítulo IV
A FILOSOFIA MODERNA
ITURRUSGARAI, Adão. “Mundo monstro”. In: Folha de S. Paulo (24 de julho de 2009).
urante o século XVII até meados do século XIX, o período da História da Filosofia que perdura é denominado de “Filosofia moderna”. Esse período é caracterizado por três importantes transformações intelectuais. A
primeira transformação diz respeito ao surgimento do “sujeito do conhecimento”; em outras palavras: em
lugar de iniciar seu trabalho conhecendo a Natureza e Deus para, depois, tratar do homem, a Filosofia começa investigar qual seria a capacidade do intelecto humano para distinguir e comprovar a verdade dos conhecimentos conhecidos.
D
Em outras palavras, a Filosofia começa pela reflexão, isto é, pela volta do pensamento sobre si mesmo para conhecer sua capacidade de conhecer. O ponto de partida é o sujeito do conhecimento como consciência de si reflexiva, isto é, como consciência que conhece sua capacidade de conhecer. O sujeito do conhecimento é um intelecto no interior de uma alma, cuja natureza
ou substância é completamente diferente da natureza ou substância de seu corpo e dos demais corpos exteriores. Por isso, a
segunda pergunta da Filosofia, depois de respondida a pergunta sobre a capacidade de conhecer, é: Como o espírito ou intelecto pode conhecer o que é diferente dele? Como pode conhecer os corpos da Natureza? [CHAUÍ, Marilena. Convite à filosofia.]
A segunda grande transformação intelectual do período moderno surge, então, da resposta à pergunta acima; essa
segunda mudança refere-se, pois, ao “objeto do conhecimento”: para os pensadores modernos, as coisas exteriores ao
homem (a Natureza, a política, a vida social etc.) podem ser conhecidas desde que sejam entendidas e consideradas
como representações, isto é, conceitos ou ideias estabelecidos pelo sujeito do conhecimento. Por um lado, isso quer
dizer que tudo aquilo que pode ser conhecido deve poder ser transformado em uma ideia, em um conceito claro,
distinto e demonstrável, formulado pelo intelecto humano; por outro lado, isso significa que a Natureza e a sociedade
podem ser totalmente conhecidas pelo sujeito, já que elas são inteligíveis em si mesmas, ou melhor, são racionais em
si mesmas, propensas a serem concebidas pelas ideias do sujeito do conhecimento.
Assim, essa compreensão da realidade como intrinsecamente racional e passível de ser plenamente apreendida
pelas ideias e conceitos organizou e preparou a terceira grande transformação intelectual moderna: a realidade, a
partir de Galileu Galilei (1564-1642), é concebida como um sistema racional de mecanismos físicos, cuja estrutura é
matemática. Nesse momento, em que a realidade, entendida como um sistema racional de mecanismos físicomatemáticos, dá-se origem à ciência clássica, ou seja, à mecânica, por meio da qual são apresentados, explicados e
interpretados os fenômenos da realidade; a realidade, por conseguinte, é entendida como um sistema de causalidades
racionais que podem ser conhecidas e transformadas pelo ser humano. Nasce, nesse momento, a ideia de experimentação e de tecnologia (ou seja, conhecimento teórico que dirige as intervenções práticas), bem como o ideal de que o
homem poderá dominar tecnicamente a sociedade e a Natureza — o que, vale lembrar, temos atualmente os resultados dessa dominação, sejam eles positivos ou negativos.
Predomina, nesse período, a ideia de conquista científica e técnica de toda a realidade, a partir da explicação mecânica e matemática do Universo e da invenção das máquinas, graças às experiências físicas e químicas. Existe também a convicção de
que a razão humana é capaz de conhecer a origem, as causas e os efeitos das paixões e das emoções e, pela vontade orientada
pelo intelecto, é capaz de governá-las e dominá-las, de sorte que a vida ética pode ser plenamente racional. [Idem.]
Vejamos, agora, as subdivisões didáticas desse período que se inicia pelo Renascimento e se estende até meados
do século XX: são eles a “Filosofia do século XVII”, a “Filosofia do século XVIII” e a “Filosofia do século XIX”.
O período da Filosofia do século XVII é, no mundo ocidental, considerada como o princípio da Filosofia moderna
e, ao mesmo tempo, como o distanciamento do pensamento medieval (em especial da escolástica). Considerado o
sucessor da Renascença e o predecessor do Iluminismo, o período é frequentemente denominado de “idade da razão”.
Nele, testemunhou-se o surgimento de pensamentos e teorias que marcaram fortemente a Filosofia; nesse meio, a
contribuição intelectual de René Descartes (1596-1650) exerceu grande influência.
Considerado o primeiro filósofo moderno, Descartes contribuiu em diversas áreas do conhecimento. Em se tratando de seu pensamento filosófico, o método que utilizou consistiu no chamado “ceticismo metodológico” (verificar
se existem evidências reais e indubitáveis a respeito do fenômeno ou coisa estudada; analisar ao máximo as coisas,
em suas unidades mais simples; agrupar novamente as unidades estudadas em um todo verdadeiro; e enumerar todas
as conclusões e princípios utilizados, a fim de manter a ordem do pensamento). Contrariamente aos gregos antigos e
aos escolásticos, que acreditavam que as coisas existiam simplesmente porque precisavam existir, Descartes instituiu
22
a dúvida: só se poderia dizer que existe aquilo que pudesse ser provado; o ato de duvidar seria, pois, indubitável. Com
isso, buscou provar a existência do próprio eu e de Deus.
A próxima subdivisão da Filosofia moderna é a Filosofia do século XVIII ou Iluminismo, quando, então, surgiu na
Europa o movimento intelectual denominado “Iluminismo”, que defendia a razão como ponto de partida da autoridade do conhecimento humano. Também conhecido como o “século das luzes” (luzes, iluminação, esclarecimento...),
foi um momento no qual, a partir da razão, o homem poderia conquistar a liberdade e a prosperidade social e política;
a razão seria hábil de dar ao homem a possibilidade de evoluir e progredir, bem como de libertar-se dos preconceitos
morais, sociais e religiosos, livrar-se das crenças irracionais, da superstição e do medo com o auxílio sempre possível
do conhecimento, das artes, das ciências.
Nesse período, diferenciou-se “Natureza” e “civilização”: a Natureza seria a esfera das relações necessárias de
causa e efeito, na qual prevaleciam leis naturais universais e imutáveis, enquanto que a civilização seria a esfera da
liberdade, das intenções sugeridas pela vontade livre dos homens. Surgem, nesse contexto, as ideias de proeminentes
filósofos, dois quais destacam-se David Hume (1711-1776) e Immanuel Kant (1724-1804).
Ainda relevante no pensamento atual, a filosofia de Hume trabalhou, dentre outras coisas, com a noção de causalidade: quando um evento provoca um outro evento, a maioria das pessoas pensa que estamos conscientes de uma
conexão entre os dois que faz com que o segundo siga o primeiro. Questionando essa crença Hume notou que, mesmo
que percebermos dois eventos, não temos necessariamente de perceber uma conexão entre os dois. Assim, não haveria como ter qualquer ideia de causalidade que não por meio do seguinte: quando vemos que dois eventos sempre
ocorrem conjuntamente, tendemos a criar uma expectativa de que quando o primeiro ocorre, o segundo seguirá. A
perspectiva de Hume parece ser a de que o homem tem uma crença na causalidade semelhante a um instinto, que se
baseia no desenvolvimento dos hábitos na nossa mente. Na visão de Hume, o comportamento humano, como tudo o
mais, é causado; com isso, se tomamos as pessoas como responsáveis pelos seus atos, devemos focar a recompensa
ou a punição de forma a que eles façam aquilo que é moralmente desejável e evitem aquilo que é moralmente repreensível. A razão não seria, pois, antagônica aos sentimentos.
Kant, por sua vez, inscreveu-se na linhagem dos pensadores que representaram uma etapa decisiva na História da
Filosofia. Com uma vasta produção filosófica (da qual a Crítica da razão pura é um dos livros mais importantes), seu
método foi a “crítica”, ou seja, uma análise reflexiva que consistia em remontar do conhecimento às condições que o
tornam eventualmente legítimo. Para ele, só se chegaria ao conhecimento por meio da análise crítica, que brotaria da
própria razão. Isso contraria, portanto, a concepção de que objetos externos poderiam ser percebidos como tal; o
conhecimento, para Kant, não seria mero reflexo dos objetos: partiria, na realidade, do sujeito, da mente humana que
determina a imagem das coisas e as organiza para explicar o universo.
GALHARDO, Caco. “Chico Bacon”. In: Folha de S. Paulo (11 de maio de 2006).
Influenciando uma nova geração de pensadores, os filósofos do século XVIII abriram caminho para o surgimento
da próxima subdivisão desse período, qual seja, a Filosofia do século XIX. Foi nessa época que se passou a delinear
com maior atenção os limites do estudo filosófico. Se os filósofos do início da época moderna ainda pareciam arquitetar suas teorias filosóficas ou fornecendo algum tipo de fundamento para uma determinada concepção científica,
bem como um trabalho de “faxina” necessário para preparar o terreno para a ciência tomar seu rumo, ou ainda como
competindo com determinada conclusão ou método científico, filósofos como Georg Wilhelm Friedrich Hegel (17701831) foram, aos poucos, deixando de se voltar ao objetivo de aumentar o conhecimento material, ou seja, de buscar a
descoberta de novas verdades, bem como de justificar as crenças religiosas racionalmente; na verdade, admitiram que
a Filosofia era, antes de tudo, uma atividade de clarificação das próprias condições do conhecimento humano.
Hegel, com uma filosofia fundamental ao pensamento filosófico, introduziu um sistema para compreender a História da Filosofia e do mundo, denominado de “dialética”: uma progressão em que cada movimento consecutivo
nasce como solução às contradições do movimento anterior. Assim, a História progrediria aprendendo com seus
erros, indicando que o homem poderia, nesse jogo dialético, fundamentar cada vez melhor suas perspectivas a fim de
aprimorar o conhecimento que possui do mundo e de si mesmo.
Leituras complementares
Decerto, a época moderna, por consequência da libertação do homem, conduz ao subjetivismo e ao individualismo. Mas é igualmente certo que nenhuma época antes dela produziu um objetivismo comparável, e que em nenhuma
outra época anterior o não-individual tornou-se legítimo, sob a configuração do coletivo. O essencial aqui é o jogo
23
necessário, recíproco e relevante entre subjetivismo e objetivismo. Este condicionamento recíproco repercute em
processos mais profundos. [...]
Quando refletimos sobre a época moderna, perguntamos sobre a imagem de mundo moderna. Nós a caracterizamos por meio do distanciamento frente às imagens de mundo antiga e medieval. Contudo, por que perguntamos sobre
uma imagem de mundo, de tal forma que conseguir para si mesma uma imagem de mundo é sua preocupação expressa? Ou será que perguntar sobre uma imagem de mundo já não é uma forma exclusivamente moderna da representação? [...]
Nenhuma época histórica pode ser descartada pela negação da sua pretensão a prevalecer. A negação só torna irrelevante o próprio negador. Todavia, para que no futuro a essência da época moderna seja suportada, a época moderna exige, em vista da referida essência, uma capacidade de alcançar a origem e um alcance da reflexão tais que os
contemporâneos só podem, hoje, preparar, mesmo assim parcialmente, embora sem poder dominá-los.
HEIDEGGER, Martin. Die Zeit des Weltbildes.
φ
Esclarecimento [Aufklärung] é a saída do homem de sua menoridade, da qual ele próprio é culpado. A menoridade é a incapacidade de fazer uso de seu entendimento sem a direção de outro indivíduo. O homem é o próprio culpado dessa menoridade se a causa dela não se encontra na falta de entendimento, mas na falta de decisão e coragem de
servir-se de si mesmo sem a direção de outrem. Sapere aude! Tem coragem de fazer uso de teu próprio entendimento,
tal é o lema do esclarecimento. [...]
Que porém um público se esclareça [aufkläre] a si mesmo é perfeitamente possível; mais que isso, se lhe for dada
a liberdade, é quase inevitável. Pois encontrar-se-ão sempre alguns indivíduos capazes de pensamento próprio, até
entre os tutores estabelecidos da grande massa, que, depois de terem sacudido de si mesmos o jugo da menoridade,
espalharão em redor de si o espírito de uma avaliação racional do próprio valor e da vocação de cada homem em
pensar por si mesmo. O interessante nesse caso é que o público, que anteriormente foi conduzido por eles a este jugo,
obriga-os daí em diante a permanecer sob ele, quando é levado a se rebelar por alguns de seus tutores que, eles mesmos, são incapazes de qualquer esclarecimento. Vê-se assim como é prejudicial plantar preconceitos, porque terminam por se vingar daqueles que foram seus autores ou predecessores destes. Por isso, um público só muito lentamente
pode chegar ao esclarecimento. Uma revolução poderá talvez realizar a queda do despotismo pessoal ou da opressão
ávida de lucros ou de domínios, porém nunca produzirá a verdadeira reforma do modo de pensar. Apenas novos
preconceitos, assim como os velhos, servirão como cintas para conduzir a grande massa destituída de pensamento
[...].
KANT, Immanuel. O que é Esclarecimento?
φ
Do lado do que denominamos Renascimento, encontramos os seguintes elementos definidores da vida intelectual:
1) surgimento de academias laicas e livres, paralelas às universidades confessionais, nas quais imperavam as versões
cristianizadas do pensamento de Platão, Aristóteles, Plotino e dos Estóicos e as discussões sobre as relações entre fé e
razão, formando clérigos e teólogos encarregados da defesa das ideias eclesiásticas; as academias redescobrem outras
fontes do pensamento antigo, se interessam pela elaboração de conhecimentos sem vínculos diretos com a teologia e
a religião, incentivam as ciências e as artes (primeiro, o classicismo e, depois da Contra-Reforma, o maneirismo); 2) a
preferência pelas discussões em torno da clara separação entre fé e razão, natureza e religião, política e Igreja. Considera-se que os fenômenos naturais podem e devem ser explicados por eles mesmos, sem recorrer à continua intervenção divina e sem submetê-los aos dogmas cristãos (como, por exemplo, o geocentrismo, com a Terra imóvel no centro do universo); defende-se a ideia de que a observação, a experimentação, as hipóteses lógico-racionais, os cálculos
matemáticos e os princípios geométricos são os instrumentos fundamentais para a compreensão dos fenômenos naturais (Bruno, Copérnico, Leonardo da Vinci sendo os expoentes dessa posição). Desenvolvem-se, assim, tendências
que a ortodoxia religiosa bloqueara durante a Idade Média, isto é, o naturalismo (coisas e homens, enquanto seres
naturais, operam segundo princípios naturais e não por decretos divinos providenciais e secretos); 3) interesse pela
ciência ativa ou prática em lugar do saber contemplativo, isto é, crença na capacidade do conhecimento racional para
transformar a realidade natural e política, donde o interesse pelo desenvolvimento das técnicas (respondendo a exigências intelectuais e econômicas da época, quando o capitalismo pede instrumentos que sejam aumentadores da
capacidade das forças produtivas); 4) alteração da perspectiva da fundamentação do saber, isto é, passagem da visão
teocêntrica (Deus como centro, principio, meio e fim do real) para a naturalista e para a humanista. Aqui, duas grandes linhas se desenvolvem: de um lado, a discussão sobre a essência da alma humana como racional e passional, de
sua força e de seus limites, conduzindo àquilo que, mais tarde, seria conhecido como o Sujeito do Conhecimento ou a
Subjetividade, que, no Renascimento, ainda se encontra mais próxima de uma “psicologia da alma” e de uma moral,
enquanto na filosofia moderna estará mais voltada pelo que seria chamado de Epistemologia (dessa preocupação com
o homem, Nicolau de Cusa, Ficino, Erasmo e Montaigne serão os grandes expoentes); e, de outro lado, a discussão
em torno dos fundamentos naturais e humanos da política. Nesta, três linhas principais se desenvolvem. A primeira,
vinda dos populistas e conciliaristas medievais e da história patriótica e republicana das cidades italianas, encontra
seu ponto mais alto e controvertido em Maquiavel que, além de desmontar as concepções clássicas e cristãs sobre o
“bom governante virtuoso” e de uma origem divina, ou natural ou racional do poder, funda o poder na divisão originária da sociedade entre os Grandes (que querem oprimir e comandar) e o Povo (que não quer ser oprimido nem
comandado), a Lei sendo a criação simbólica da unidade social pela ação política e pela lógica da ação (e não pela
força, como se costuma supor). Na segunda linha, a discussão se volta para a crítica do presente pela elaboração de
24
uma outra sociedade possível-impossível, justa, livre, igualitária, racional perfeita — a utopia, cujos expoentes são
Morus e Campanella. A terceira linha discute a política a partir dos conceitos de direito natural e direito civil (linha
que irá predominar entre os modernos), das causas das diferenças entre os regimes políticos e as formas da soberania,
sendo seus expoentes Pasquier, Bodin, Grócio. Nas três linhas, encontramos a preocupação com a história, seja como
prova de que outra sociedade é possível, seja como exame dos erros cometidos por outros regimes, seja como exemplo do que pode ser imitado ou conservado. [...]
A expressão “filosofia moderna ou filosofia do século XVII” é uma abstração, como já sugerimos ao mencionar a
questão da cronologia. Mas é também uma abstração se considerarmos as várias filosofias que polemizaram entre si
nesse período, os filósofos concebendo a metafísica, a ciência da Natureza, as técnicas, a moral e a política de maneiras muito diferenciadas. No entanto, para quem olha de longe, é impossível não reconhecer a existência de um campo
de pensamento e de um campo discursivo comuns a todos os pensadores modernos e no interior dos quais suas semelhanças e diferenças se configuram. É desse campo comum que falaremos aqui.
Convém não esquecermos que a distinção entre filosofia e ciência é muito recente (consolidou-se apenas nos meados do século XIX), de modo que os pensadores do século XVII são considerados sábios (e não intelectuais, noção
que também é muito recente) e não separam seus trabalhos científicos, técnicos, metafísicos, políticos. Para eles, tudo
isso constitui a filosofia e cada sábio costuma ser um pesquisador ou um conhecedor de todas as áreas de conhecimento, mesmo que se dedique preferencialmente mais a umas do que a outras. Essa relação entre as atividades levou
o filósofo Merleau-Ponty a designar a filosofia moderna como a época do Grande Racionalismo para o qual as relações entre ciência da Natureza, metafísica, ética, política, espírito e matéria, alma e corpo, consciência e mundo
exterior estavam articuladas porque fundadas num mesmo princípio que vinculava internamente todas as dimensões
da realidade: a Substância Infinita, isto é, o conceito do Ser Infinito ou Deus.
Das características gerais do campo de pensamento e de discursos da Filosofia Moderna, destacaremos os seguintes: o significado da nova ciência da Natureza, os conceitos de causalidade e de substância, a ideia de método ou de
mathesis universalis, e a ideia de razão, explícita ou implicitamente elaborada por tais pensadores.
A nova Ciência da Natureza ou Filosofia Natural. Num nível superficial, pode-se dizer que a nova Ciência da Natureza ou Filosofia Natural possui três características 1) passagem da ciência especulativa para a ativa, na continuidade do projeto renascentista de dominação da Natureza e cuja fórmula se encontra em Francis Bacon: “Saber é Poder”;
2) passagem da explicação qualitativa e finalística dos naturais para a explicação quantitativa e mecanicista; isto é,
abandono das concepções aristotélico-medievais sobre as diferenças qualitativas entre as coisas como fonte de explicação de suas operações (leve, pesado, natural, artificial, grande, pequeno, localizado no baixo ou no alto) e da ideia
de que os fenômenos naturais ocorrem porque causas finais ou finalidades os provocam a acontecer. Tais concepções
são substituídas por relações mecânicas de causa e efeito segundo leis necessárias e universais, válidas para todos os
fenômenos independentemente das qualidades que os diferenciam para nossos cinco sentidos (peso, cor, sabor, textura, odor, tamanho) e sem qualquer finalidade, oculta ou manifesta; 3) conservação da explicação finalística apenas no
plano da metafísica: a liberdade da vontade divina e humana e a inteligência divina e humana, embora incomensuráveis, se realizam tendo em vista fins (o filósofo Hobbes suprimirá boa parte das finalidades no campo da moral,
dando-lhe fisionomia mecanicista também, e o filósofo Espinosa suprimirá a finalidade na metafísica e na ética,
criticando-a como superstição e ignorância das verdadeiras causas das ações).
As ideias de substância e de causalidade. Enquanto o pensamento greco-romano e o cristão admitiam a existência
de uma pluralidade infinita (ou indefinida) de substâncias, os modernos irão simplificar enormemente tal conceito.
Substância é toda realidade capaz de existir (ou de subsistir) em si e por si mesma. Tudo que precisar de outro ser
para existir será um modo ou um acidente da substância. Na versão tradicional, mineral era uma substância, vegetal
era substância, animal, outra substância, espiritual, uma outra. Mas não só isto, dependendo das filosofias, cada mineral, cada vegetal, cada animal, cada espírito, era substância, de tal maneira que haveria tantas substâncias quantos
indivíduos. Simplificadamente: a substância podia ser pensada como um gênero, ou como uma espécie ou até como
um indivíduo. E cada qual teria seus modos ou acidentes e suas próprias causalidades.
Os modernos, especialmente após Descartes, admitem que há apenas três substâncias: a extensão (que é a matéria
dos corpos, regida pelo movimento e pelo repouso), o pensamento (que é a essência das ideias e constitui as almas) e
o infinito (isto é, a substância divina). Essa alteração significa apenas o seguinte: uma substância se define pelo seu
atributo principal que constitui sua essência (a extensão, isto é, a matéria como figura, grandeza, movimento e repouso; o pensamento, isto é, a ideia como inteligência e vontade; o infinito, isto é, Deus como causa infinita e incriada).
Na verdade, os modernos não concordarão com a tripartição de Descartes. Os materialistas, por exemplo, dirão
que há apenas extensão e infinito; os espiritualistas, que há apenas pensamento e infinito. E, nos dois extremos dessa
discussão, estarão Espinosa, de um lado, e Leibniz, de outro. Para Espinosa existe uma e apenas uma substância — a
infinitamente infinita, isto é, Deus, com infinitos atributos infinitos dos quais conhecemos dois, o pensamento e a
extensão (suprema heresia: Espinosa afirma que Deus é extenso), todo o restante do universo são os modos singulares
da única substância. Para Leibniz, existem infinitas substâncias, cada uma delas contendo em si mesma um dos dois
grandes atributos — pensamento (inteligência, vontade, desejo) ou extensão (figura, grandeza, movimento e repouso). Essas substâncias se chamam mônadas (unidade última e indivisível) e há apenas uma diferença entre as mônadas — isto é, há a Mônada Infinita, que é Deus, e há as mônadas criadas e finitas, isto é, os seres existentes no universo, e que podem ser extensas ou pensantes.
De qualquer maneira, o essencial na questão da Substância definida pelo seu atributo principal é que, de agora em
diante, conhecer é conhecer apenas três tipos de essências e suas operações fundamentais: a matéria (geometrizada), a
alma (intelecto, vontade e apetites) e o infinito.
Esse conhecimento se fará pelo conceito de causalidade. Conhecer é conhecer a causa da essência, da existência e
das ações e reações de um ser. Um conhecimento será verdadeiro apenas e somente quando oferecer essas causas.
Evidentemente, os filósofos discordarão quanto ao que entendem por causa e causalidade, discordarão quanto à de-
25
terminação de uma realidade como sendo causa ou como sendo efeito, discordarão quanto ao número de causas,
discordarão quanto aos procedimentos intelectuais que permitem conhecer as causas e, portanto, discordarão quanto à
definição da própria noção de verdade, uma vez que esta depende do que se entende por causa e por operação causal.
Mas todos, sem exceção, consideram que um conhecimento só pode aspirar à verdade se for conhecimento das causas, sejam elas quais forem e seja como for a maneira como operem. O importante é notar que fizeram a verdade, a
inteligibilidade e o pensamento dependerem da explicação causal e afastaram a explicação meramente descritiva ou
interpretativa. A síntese desse ideal encontra-se em Espinosa e em Leibniz. Afirma Espinosa que o conhecimento
verdadeiro é aquele que nos diz como uma realidade foi produzida, isto é, o conhecimento verdadeiro é o que alcança
a gênese necessária de uma realidade. Leibniz estabelece o chamado principio da Razão Suficiente, segundo o qual
nada existe que não tenha uma causa e que não possa ser conhecida, ou, como ficou conhecido: “Nihil sine ratione”,
nada é sem causa.
Com relação ao conceito de causalidade, é necessário fazermos três observações: 1) diferentemente dos gregos,
romanos e medievais (que admitiam quatro causas — material, formal, eficiente ou motriz e final), os modernos
admitem apenas duas: a eficiente (a causalidade propriamente dita como relação entre uma causa e seu efeito direto) e
a final, para os seres dotados de vontade livre, pois esta sempre age tendo em vista fins (Deus e homens). Apenas
Espinosa recusa a finalidade, considerando a causa final um produto da imaginação e uma ilusão; 2) a causa eficiente
exige que causa e efeito sejam de mesma natureza (de mesma substância; ou de mesmo modo, no caso de Espinosa),
de sorte que causas corporais não podem produzir efeitos anímicos e vice-versa. Ora, os humanos são criaturas mistas
(possuem corpo e alma) e é preciso explicar causalmente as relações entre ambos se se quiser conhecer o homem e
sobretudo o que os modernos chamam de ação e paixão. As soluções do problema serão variadas. Assim, por exemplo, Descartes imagina uma glândula — a glândula pineal, na base do pescoço — que faria a comunicação entre as
duas substâncias do composto humano; Espinosa e Leibniz consideram a posição cartesiana absurda, e para ambos a
relação entre alma e corpo não é “causal” no sentido de ação do corpo sobre a alma ou vice-versa, mas uma relação
de expressão, isto é, o que se passa num deles se exprime de maneira diferente no outro e vice-versa; os materialistas
resolvem o problema considerando que os efeitos anímicos são uma modalidade dos comportamentos corporais, pois
não haveria uma substância espiritual, a não ser Deus; os espiritualistas vão na direção contrária (como Malebranche), considerando os corpos e os acontecimentos corporais como aparência sensível de realidades puramente espirituais; 3) o conceito de causa possui três sentidos simultâneos e inseparáveis e não apenas um; esses três sentidos
simultâneos constituem a causalidade como princípio de plena inteligibilidade do real: a) a causa é algo real que
produz um efeito real (causa e efeito são entes, seres, coisas); b) a causa é a razão que explica a essência e a existência de alguma coisa, é sua explicação verdadeira e sua inteligibilidade; c) a causa é o nexo lógico que articula e vincula necessariamente uma realidade a uma outra, tornando possível não só sua existência, mas também seu conhecimento. Conhecer pela causa é, pois, conhecer entes, razões e vínculos necessários.
A ideia de método ou de mathesis universalis. Os filósofos modernos enfrentam três grandes problemas no tocante ao conhecimento verdadeiro:
1) tendo o Cosmos, sua ordem, sua hierarquia e seu centro desaparecido, o homem, como ser pensante, não encontra imediatamente nas coisas percebidas a verdade, a origem e o sentido do real, pois as coisas são percebidas em
suas qualidades sensoriais e o mundo parece ser finito e ordenado por valores e perfeições que a nova ciência da
Natureza revelou serem ilusórios;
2) o conceito de causalidade faz uma exigência teórica que, se não for respeitada, impede que a verdade seja conhecida. Essa exigência é de que as relações causais só se estabelecem entre coisas de mesma substância (a extensão,
ou a matéria, ou os corpos, dependendo da terminologia de cada sábio, só produz efeitos extensos, materiais, corporais; o pensamento, a alma, as ideias, também dependendo da terminologia de cada filósofo, só produzem efeitos
pensantes, anímicos, ideativos; o finito só produz efeitos finitos; o infinito, única exceção, produz efeitos finitos e
infinitos, mas não pode ser produzido por uma causa finita). Ora, como já o dissemos, os humanos são compostos de
duas substâncias (ou de modos diferentes da mesma substância, no caso de Espinosa) que, no plano causal, não podem causar-se um ao outro. Ora, conhecer é uma atividade da substância pensante ou do modo pensante, mas o conhecido pode tanto ser um aspecto do pensante quanto os corpos, as coisas ou os modos extensos. E, neste caso, a
causalidade não pode operar, pois o que se passa na extensão não pode causar efeitos no pensamento e vice-versa. A
solução encontrada por todos os filósofos (com variantes, novamente, e com exceção de Espinosa) consiste em considerar o conhecimento uma Representação, isto é, que a inteligência não afeta nem é afetada pelos corpos, mas pelas
ideias deles, havendo assim a homogeneidade exigida pela causalidade;
3) mas a representação cria um novo problema: como saber se as ideias representadas correspondem verdadeiramente às coisas representadas? Como saber se a ideia é adequada ao seu ideado? Para solucionar esta dificuldade
nasce o método.
A noção de representação significa que aquele que conhece — o Sujeito do Conhecimento — está sozinho, rodeado por coisas cuja verdade ele não pode encontrar imediatamente, pois percebe coisas, mas deve conhecer Objetos
do Conhecimento, isto é, as ideias verdadeiras ou os conceitos dessas coisas percebidas. Precisa de um instrumento
que lhe permita três atividades: 1) representar corretamente as coisas, isto é, alcançar suas causas sem risco de erro
(para os espiritualistas, os erros virão dos sentidos ou do corpo; para os materialistas, os erros virão das abstrações
indevidas feitas pela inteligência); 2) controlar cada um dos passos efetuados, pois a perda de controle de uma das
operações intelectuais pode provocar o erro no final do percurso, que, por isso, deve ser controlado passo por passo;
3) permitir que se possa deduzir ou inferir de algo já conhecido com certeza o conhecimento de algo ainda desconhecido, isto é, o instrumento deve permitir o progresso dos conhecimentos verdadeiros oferecendo recursos seguros para
que se possa passar do conhecido ao desconhecido. A função do método é de preencher esses três requisitos. Por essa
razão, nenhum dos filósofos modernos deixa de escrever um tratado sobre o método.
26
No século XVII, a palavra método (do grego: caminho certo, correto, seguro) tem um sentido vago e um sentido
preciso. Sentido vago, porque todos os filósofos possuem um método ou o seu método, havendo tantos métodos
quantos filósofos. Sentido preciso, porque o bom método é aquele que permite conhecer verdadeiramente o maior
número de coisas com o menor número e regras. Quanto maiores a generalidades e a simplicidade do método, quanto
mais puder ser aplicado aos mais diferentes setores do conhecimento, melhor será ele.
O método é sempre considerado matemático. Isto não quer dizer que se usa a aritmética, a álgebra, a geometria
para o conhecimento de todas as realidades, e sim que o método procura o ideal matemático, isto é, ser uma mathesis
universalis.
Isto significa duas coisas: 1) que a matemática é tomada no sentido grego da expressão ta mathema, isto é, conhecimento completo, perfeito e inteiramente dominado pela inteligência (aritmética, geometria, álgebra são matemáticas, por isso, isto é, porque dominam completa e intelectualmente seus objetos); 2) que o método possui dois elementos fundamentais de todo conhecimento matemático: a ordem e a medida.
Vimos que, no Renascimento, o conhecimento operava com a noção de Semelhança, era descritivo e interpretativo. A diferença entre os renascentistas e os modernos consiste no fato de que estes últimos criticam a Semelhança,
considerando-a causa dos erros e incapaz de alcançar a essência das coisas. Conhecer pela causa significa que a inteligência é capaz de discernir a identidade e a diferença no nível da essência invisível das coisas. A ordem e a medida
têm a função de produzir esse discernimento e por isso são o núcleo do método e da mathesis.
Conhecer é relacionar. Relacionar é estabelecer um nexo causal. Estabelecer um nexo causal é determinar quais
as identidades e quais as diferenças entre os seres (coisas, ideias, corpos, afetos, etc.). A medida oferece o critério
para essa identidade e essa diferença. Assim, por exemplo, a medida permitirá que não se estabeleça uma relação
causal entre realidades heterogêneas quanto à substância. Ela analisa, isto é, decompõe um todo em partes e estabelece qual o elemento que serve de unificador para essas partes (a “grandeza” comum a todas elas). A ordem é o conhecimento do encadeamento interno e necessário entre os termos que foram medidos, isto é, estabelece qual o termo que
se relaciona com outro e em qual sequência necessária, de sorte que ela estabelece uma série ordenada, sintetiza o que
foi analisado pela medida e permite passar do conhecido ao desconhecido.
A ordem é essencial ao método por três motivos: 1) porque os modernos consideram que a primeira verdade de
uma série é conhecida por uma intuição evidente, a partir da qual será colocada a medida e esta depende da seriação
dos termos feita pela ordem; 2) porque os conhecimentos de totalidades complexas são conhecimentos de séries
diferentes, cujas relações só podem ser estabelecidas se cada série estiver corretamente ordenada; 3) porque a ordem
permite a relação entre um primeiro termo e um último cuja medida pode não ser a mesma (são heterogêneos ou
incomensuráveis), mas a relação pode ser feita porque a ordenação foi fazendo aparecer entre um termo e outro uma
medida nova que encadeia o segundo ao terceiro, este ao quarto e assim por diante.
Um exemplo deste último e mais importante procedimento. Na filosofia de Descartes, não haveria como estabelecer relação causal entre a alma finita humana, Deus infinito e o mundo extenso, já que são três substâncias diferentes.
Aplicando a medida e a ordem, Descartes estabelece o que chama de cadeia de razões (nexos causais e lógicos) do
seguinte tipo: a alma pensa e ao pensar tem uma ideia de que ela própria não pode ser a causa, a ideia de Deus; isto é,
a alma finita não pode ser causa de uma ideia infinita. Sendo, porém, Deus uma ideia, pode perfeitamente estar em
nossa alma e pode causá-la em nós, porque o intelecto divino age sobre o nosso por meio das ideias verdadeiras. Ora,
a ideia de Deus é a ideia de um Ser Perfeito, que seria imperfeito se não existisse, portanto, a ideia presente em nossa
inteligência, causada pela inteligência de Deus, é a ideia de um ser que só será Deus se existir. Nós não podemos
fazer Deus existir, mas a ideia de Deus nos revela que ele existe. Passamos, assim, da ideia ao ser. Ora, esse ser é
perfeito, e se nos faz ter ideias das coisas exteriores através de nossos sentidos, é porque nos deu um corpo e criou
outros corpos que constituem o mundo extenso. Passamos, assim, do ser de Deus à ideia de nosso corpo e às ideias
dos corpos exteriores, o que não poderia ser feito sem a ordem, pois sem ela não poderíamos passar de nossa alma a
Deus e dele ao nosso corpo nem aos corpos exteriores. A medida é a ideia e a ordem da sequência causal dessas
ideias até chegar a corpos.
O método, ciência universal da ordem e da medida, pode ser analítico ou sintético. Na análise, vai-se das partes
ao todo ou do particular ao universal (é o método preferido por Descartes e Locke); na síntese, vai-se do todo às
partes ou do universal ao particular (é o método preferido por Espinosa); ou uma combinação de ambos, conforme as
necessidades próprias do objeto de estudo (como faz Leibniz). Em qualquer dos casos, realiza-se pela ordem e pela
medida, mas é considerado dedutivo pelos racionalistas intelectualistas (que partem das ideias para as sensações) e
indutivo pelos racionalistas empiristas (que partem das sensações para as ideias). Essa diferença repercute no conceito de intuição, que é considerado por todos como o ponto de partida da cadeia dedutiva ou da cadeia indutiva: no
primeiro caso, a intuição é uma visão puramente intelectual de uma ideia verdadeira; no segundo caso, a intuição é
sensível, isto é, visão ou sensação evidente de alguma coisa que levará à sua ideia.
A ideia moderna da Razão. Em seu livro História da Filosofia, Hegel declara que a filosofia moderna é o nascimento da Filosofia propriamente dita porque nela, pela primeira vez, os filósofos afirmam:
1) que a filosofia é independente e não se submete a nenhuma autoridade que não seja a própria razão como faculdade plena de conhecimento. Isto é, os modernos são os primeiros a demonstrar que o conhecimento verdadeiro só
pode nascer do trabalho interior realizado pela razão, graças a seu próprio esforço, sem aceitar dogmas religiosos,
preconceitos sociais, censuras políticas e os dados imediatos fornecidos pelos sentidos. Só a razão conhece e somente
ela pode julgar-se a si mesma;
2) que a filosofia moderna realiza a primeira descoberta da Subjetividade propriamente dita porque nela o primeiro ato de conhecimento, do qual dependerão todos os outros, é a Reflexão ou a Consciência de Si Reflexiva. Isto é, os
modernos partem da consciência da consciência, da consciência do ato de ser consciente, da volta da consciência
sobre si mesma para reconhecer-se como sujeito e objeto do conhecimento e como condição da verdade. A consciência é para si mesma o primeiro objeto do conhecimento, ou o conhecimento de que é capacidade de e para conhecer;
27
3) que a filosofia moderna é a primeira a reconhecer que, sendo todos os seres humanos seres conscientes e racionais, todos têm igualmente o direito ao pensamento e à verdade. Segundo Hegel, essa afirmação do direito ao pensamento, unida à ideia de liberdade da razão para julgar-se a si mesma, portanto, o igualitarismo intelectual e a recusa
de toda censura sobre o pensamento e a palavra, seria a realização filosófica de um principio nascido com o protestantismo e que este, enquanto mera religião, não poderia cumprir precisando da filosofia para realizar-se: o princípio
da individualidade como subjetividade livre que se relaciona livremente com o infinito e com a verdade.
A razão, o pensamento, a capacidade da consciência para conhecer por si mesma a realidade natural e espiritual, o
visível e o invisível, os seres humanos, a ação moral e política, chama-se Luz Natural. Embora os modernos se diferenciem quanto à Luz Natural (para alguns, por exemplo, a razão traz inatamente não só a possibilidade para o conhecimento verdadeiro, mas até mesmo as ideias, que seriam inatas; para outros, nossa consciência é como uma folha em
branco na qual tudo será impresso pelas sensações e pela experiência, nada possuindo de inato), o essencial é que a
Luz Natural significa a capacidade de auto-iluminação do pensamento, uma faculdade inteiramente natural de conhecimento que alcança a verdade sem necessidade da Revelação ou da Luz Sobrenatural (ainda que alguns filósofos,
como Pascal, Leibniz ou Malebranche, considerem que certas verdades só podem ser alcançadas pela Luz Natural se
esta for auxiliada pela luz da Graça Divina).
A primeira intuição evidente, verdade indubitável de onde partirá toda a filosofia moderna, concentra-se na célebre formulação de Descartes: “Penso, logo existo” (Cogito, ergo sum). O pensamento consciente de si como “Força
Nativa” (a expressão é de Espinosa), capaz de oferecer a si mesmo um método e de intervir na realidade natural e
política para modificá-la, eis o ponto fixo encontrado pelos modernos.
CHAUÍ, Marilena. Filosofia moderna.
Cogito, logo aprendo
1. (UFMG 2002) Leia este trecho: “Enquanto os homens se contentaram com suas cabanas rústicas, enquanto se
limitaram a costurar com espinhos ou com cerdas suas roupas de peles [...] — em uma palavra: enquanto só se dedicaram a obras que um único homem podia realizar, e a artes que não solicitavam o concurso de várias mãos, viveram
tão livres, sadios, bons e felizes quanto o poderiam ser por sua natureza, e continuaram a gozar entre si das doçuras
de um comércio independente; mas, desde o instante em que um homem sentiu necessidade da ajuda de outro, desde
que se percebeu ser útil a um só contar com provisões para dois, desapareceu a igualdade, introduziu-se a propriedade, o trabalho tornou-se necessário e as vastas florestas transformaram-se em campos aprazíveis que se impôs regar
com o suor dos homens e nos quais logo se viu a escravidão e a miséria germinarem e crescerem com as colheitas.”
[ROUSSEAU, Jean-Jacques. Discurso sobre a origem e os fundamentos da desigualdade entre os homens.]
A partir das ideias contidas nesse trecho e em outros conhecimentos presentes nessa obra de Rousseau, COMENTE a relação estabelecida pelo autor entre progresso e desigualdade.
2. (UFMG 2001) Leia este texto: “Da Filosofia nada direi, senão que, vendo que foi cultivada pelos mais excelsos
espíritos que viveram desde muitos séculos e que, no entanto, nela não se encontra ainda uma só coisa sobre a qual
não se dispute e, por conseguinte, que não seja duvidosa, eu não alimentava qualquer presunção de acertar mais que
os outros; e que, considerando quantas opiniões diversas, sustentadas por homens doutos, pode haver sobre uma
mesma matéria, sem que jamais possa existir mais de uma que seja verdadeira, reputava quase como falso tudo o que
era somente verossímil. [...] Eis por que, tão logo a idade me permitiu sair da sujeição de meus preceptores, deixei
inteiramente o estudo das letras. E, resolvendo-me a não mais procurar outra ciência, além daquela que poderia achar
em mim próprio, ou então no grande livro do mundo, empreguei o resto de minha mocidade em viajar, em ver cortes
e exércitos, em frequentar gente de diversos humores e condições, em recolher diversas experiências, em provar-me a
mim mesmo nos reencontros que a fortuna me propunha e, por toda parte, em fazer tal reflexão sobre as coisas que
me apresentavam que eu pudesse tirar delas algum proveito. [...] Mas, depois que empreguei alguns anos em estudar
assim no livro do mundo, e em procurar adquirir alguma experiência, tomei um dia a resolução de estudar também a
mim próprio e de empregar todas as forças de meu espírito na escolha dos caminhos que deveria seguir.” [DESCARTES,
René. Discurso do método.]
Na Primeira Parte do Discurso do método, o autor faz uma avaliação de todos os conhecimentos acumulados pela
tradição, inclusive da filosofia que aprendeu nos livros.
a. EXPLICITE por que Descartes defende o abandono dos caminhos traçados pela filosofia até então.
b. EXPLIQUE o novo ponto de partida da filosofia de Descartes.
3. (UFMG 2001) Leia estes trechos:
TRECHO 1. “Ora, nada é mais meigo do que o homem em seu estado primitivo, quando, colocado pela natureza a
igual distância da estupidez dos brutos e das luzes funestas do homem civil, e compelido tanto pelo instinto quanto
pela razão a defender-se do mal que o ameaça, é impedido pela piedade natural de fazer mal a alguém sem ser a isso
levado por alguma coisa ou mesmo depois de atingido por algum mal.” [ROUSSEAU, Jean-Jacques. Discurso sobre a origem e os fundamentos da desigualdade entre os homens.]
TRECHO 2. “O homem nasce livre, e por toda a parte encontra-se a ferros. O que se crê senhor dos demais não
deixa de ser mais escravo do que eles.” [ROUSSEAU, Jean-Jacques. Do contrato social.]
A partir das ideias contidas nesses trechos,
a. DEFINA a condição do homem em seu estado natural.
b. COMPARE a situação do homem primitivo, como descrita por Rousseau, com a situação vivida pelos homens
nas sociedades industriais contemporâneas.
28
4. (UFMG 2003) Leia esta afirmação:
“... as causas e os efeitos não podem ser descobertos pela razão, mas sim pela experiência...” [HUME, David. Investigação sobre o entendimento humano.]
REDIJA um texto justificando, do ponto de vista do autor, essa afirmação.
5. (UFMG 2004) Leia estes trechos:
TRECHO 1. “Afinal, que é o homem dentro da natureza? Um nada em relação ao infinito; um tudo em relação ao
nada; um meio entre nada e tudo. Infinitamente incapaz de compreender os extremos, tanto o fim das coisas como
seus princípios permanecem invencivelmente ocultos num segredo impenetrável, e é-lhe igualmente impossível ver o
nada de onde saiu e o infinito que o envolve. [...] Eis o nosso estado verdadeiro, que nos torna incapazes de saber com
segurança e de ignorar totalmente. Vagamos em um meio vasto, sempre incertos e flutuantes, empurrados de um
extremo ao outro. [...] É o estado que nos é natural e, no entanto, nenhum será mais contrário à nossa inclinação.”
TRECHO 2. “A grandeza do homem é grande na medida em que ele se conhece miserável. Uma árvore não se
conhece miserável. É, pois, ser miserável conhecer-se miserável, mas é ser grande conhecer que se é miserável.”
[PASCAL, Blaise. Pensamentos.]
Com base na leitura desses trechos e em outras ideias contidas nesta obra de Pascal, REDIJA um texto, desenvolvendo dois aspectos concernentes à ideia de condição humana.
6. (UFMG 2004) Leia este trecho: “É, pois, difícil para cada homem em particular conseguir livrar-se desta menoridade tornada quase uma natureza. [...] Mas que um público se esclareça a si mesmo, isso é bem mais possível e,
mais, se é deixado em liberdade, então é quase inevitável.” [KANT, Immanuel. Que é esclarecimento?]
A partir da leitura desses trechos e de outras ideias presentes nesta obra de Kant, REDIJA um texto, justificando
por que, para o autor, a saída da menoridade é difícil para os homens na esfera privada e bem mais possível para os
homens como membros de uma comunidade total.
29
Capítulo V
A FILOSOFIA CONTEMPORÂNEA
stamos, atualmente, imersos em um mundo, em meio a uma civilização que não para de trazer à tona novas
interrogações para a Filosofia, de maneira que, diante dessa enxurrada de novas questões, torna-se difícil
reconhecer o que é a Filosofia contemporânea. Em todo caso, esse período parte de alguns conceitos elaborados no século XIX; a Filosofia, com isso, relacionou-se com as ideias de progresso e de totalidade: o ser humano passou a ser entendido como um ser histórico, assim como a sociedade. Como consequência dessa
perspectiva, temos a ideia de progresso.
Assim, tanto a razão quanto o saber científico seguem na direção do desenvolvimento humano. Contudo, o que se
viu foi que o progresso mostrou-se descontínuo, ou seja, não se faz por etapas sucessivas, de modo que a história
universal não é um mero conjunto de diversas civilizações em etapas diversas de desenvolvimento: cada cultura tem
seus próprios valores, cada sociedade tem sua própria história. Tal visão de mundo permitiu o desenvolvimento de
várias ciências, como a antropologia, a etnologia e as ciências sociais. Mas, embora tenha se estendido em disciplinas
específicas, a Filosofia ainda possui (como nunca deixou de possuir) atitude filosófica.
Consideremos, portanto, como contemporâneo o período da Filosofia em que a atitude filosófica partiu daquilo
que a Filosofia moderna deixou como herança. Que herança foi essa? Dentre vários pontos importantes, temos a
busca pelo acordo entre a racionalidade formal do passado com um maior senso emocional e orgânico do mundo.
Temos, também, que a Filosofia no século XIX assegurava-se em uma confiança plena e total no saber científico e na
tecnologia para dominar e controlar a Natureza, a sociedade e os indivíduos. No entanto, com a Filosofia contemporânea o que se viu foi uma desconfiança no otimismo científico-tecnológico do século anterior em virtude de vários
acontecimentos: as duas guerras mundiais, o bombardeio de Hiroshima e Nagasaki, os campos de concentração nazistas, as guerras da Coreia, do Vietnã, do Oriente Médio, do Afeganistão, as ditaduras sangrentas da América Latina, a
devastação de mares, florestas e terras, os perigos cancerígenos de alimentos e remédios, o aumento de distúrbios e
sofrimentos mentais etc.
E
ANGELI. “Chiclete com banana”. In: Folha de S. Paulo (6 de abril de 2009).
Podemos notar, além disso, que se no século XIX a Filosofia descobre a cultura como o modo próprio e específico da existência dos seres humanos, uma criação coletiva de ideias, símbolos e valores pelos quais uma sociedade
define para si mesma o bom e o mau, o belo e o feio, o justo e o injusto, o verdadeiro e o falso, o possível e o impossível, o sagrado e o profano, veremos que no século XX surgirá a afirmação de que, por ser a História descontínua,
não há uma cultura, mas culturas diferentes, e que cada qual inventa seu modo de relacionar-se com o tempo, de criar
sua linguagem, de elaborar seus mitos e suas crenças, de organizar o trabalho e as relações sociais, de criar as obras
de pensamento e de arte.
Isso trouxe como resultado uma nova concepção da Filosofia e do filósofo. Friedrich Nietzsche (1844-1900), por
exemplo, assinalou que não se tratava mais de buscar um ideal de conhecimento verdadeiro, mas sim de interpretar e
avaliar a realidade. Por conseguinte, a interpretação teria como meta a definição do sentido dos fenômenos, sempre
parciais, fragmentários. A avaliação filosófica tentaria, assim, determinar o valor desses sentidos. O filósofo do futuro, para Nietzsche, deveria procurar uma unidade entre o pensamento e a vida, de modo que esta estimule o pensamento e o pensamento, por sua vez, afirme a vida. No entanto, o desenvolvimento da Filosofia teria ocasionado a
degeneração dessa característica e, em lugar de uma existência ativa e de um pensamento afirmativo, a Filosofia, ao
contrário, propôs como ocupação julgar a vida, opondo-a a valores supostamente superiores, impondo-lhe limites; em
suma, condenando-a.
Às margens de uma crise, a Filosofia contemporânea notou, não obstante, que em face da ciência o fazer filosófico percebeu silenciada sua razão de existir. Ainda assim, os filósofos mostraram que nem mesmo as ciências não
possuíam
princípios totalmente certos, seguros e rigorosos para as investigações, que os resultados podem ser duvidosos e precários, e
que, frequentemente, uma ciência desconhece até onde pode ir e quando está entrando no campo de investigação de uma outra.
Os princípios, os métodos, os conceitos e os resultados de uma ciência podem estar totalmente equivocados ou desprovidos de
fundamento. Com isso, a Filosofia voltou a afirmar seu papel de compreensão e interpretação crítica das ciências, discutindo a
validade de seus princípios, procedimentos de pesquisa, resultados, de suas formas de exposição dos dados e das conclusões
etc. [CHAUÍ, Marilena. Convite à filosofia.]
30
Nesse ínterim, se com o pensamento filosófico as certezas haviam sido derrubadas, a Filosofia do século XX tentou, pois, reformar, preservar, alterar os limites antes concebidos; trouxe, portanto, uma série de desenvolvimentos
teóricos relacionados à validade do conhecimento. Entretanto, essa nova perspectiva filosófica que se abria era demasiado diferente para que se pudesse estabelecer um padrão, que não fosse uma série de tentativas de reformar, preservar ou alterar os limites antes concebidos.
O mundo em que vivemos, das telecomunicações, da internet, dos programas espaciais, da física quântica, ou da medicina de
alta tecnologia parece não ter lugar para a filosofia. Onde está a filosofia? O filósofo Bertrand Russel pensou nessa questão:
“A filosofia, como todos os outros estudos, visa em primeiro lugar ao conhecimento. O conhecimento a que ela aspira é o
tipo de conhecimento que dá unidade e sistematiza o corpo das ciências, e que resulta de um exame crítico dos fundamentos de
nossas convicções, preconceitos e crenças.” (Bertrand Russell, Os problemas da filosofia.) [CHAUÍ, Marilena. Convite à filosofia.]
A Filosofia, em especial a contemporânea, apesar de se mostrar incapaz de nos dizer com certeza quais são as
respostas verdadeiras às dúvidas que ela provoca, mostra-se apropriada para apontar possibilidades capazes de ampliar os pensamentos do ser humano, libertando-o, com isso, da opressão que a tradição propicia. Embora enfraquecendo o sentimento humano de certeza a respeito do que as coisas são, o pensar filosófico aumenta, e muito, o conhecimento em relação ao que as coisas podem ser.
Talvez a característica externa mais saliente da filosofia contemporânea seja a disparidade de enfoques, sistemas e escolas, face ao desenvolvimento, de certo modo mais uniforme e linear, da filosofia moderna (racionalismo, empirismo, Kant, idealismo
hegeliano). Para esta proliferação de pontos de vista e de escolas, contribuíram, em grande medida, fatores sócio-culturais,
como: a crise contemporânea dos sistemas políticos, o avanço espetacular das ciências naturais e lógico-formais e o desenvolvimento das ciências humanas, cujos métodos e resultados tiveram repercussões e consequências de interesse no campo e nos
problemas da filosofia (psicanálise, estruturalismo). [CORDÓN, Juán Manuel Navarro; MARTÍNEZ, Tomás Calvo. História da filosofia.]
Vale dizer que o homem, também passando por uma crise existencial, mostrou-se carente de significados. A falta
de certezas colocou-o em uma situação na qual não mais consegue olhar para si e explicar-se, explicar sua existência,
explicar seu mundo. Isso, em grande parte e ironicamente, teve como causa o próprio pensamento humano que, antes
de desembocar nesse colapso ontológico, passou por humilhações que lhe feriram o ego:
[...] a humilhação cosmológica, infligida por Copérnico quando provou que a Terra não era o centro do Universo; a humilhação biológica, imposta por Darwin, cuja doutrina evolucionista reinseriu o homem no restante do mundo animal; e a humilhação psíquica, sofrida pelo homem quando a psicanálise revelou que o ego estava sujeito a determinismos internos cuja verdadeira natureza permanecia inconsciente. [ROUANET, Sérgio Paulo. Os choques da civilização.]
GALHARDO, Caco. “Piratas do Tietê”. In: Folha de S. Paulo (7 de abril de 2009).
Ainda assim, a Filosofia contemporânea não parou sua busca pelo conhecimento. Após a evidência das humilhações que implodiram o amor-próprio humano, no período contemporâneo filósofos como Jean-Paul Sartre (19051980) postularam que não haveria afirmações gerais verdadeiras sobre o que os homens deveriam ser. Sartre radicalmente negou que existia uma natureza humana, pois não existia nenhuma coisa como uma natureza que fosse comum
a todos os seres humanos, sem exceção; nada haveria, como uma essência específica, que poderia definir o que seja
“ser humano”. Se para Aristóteles, por exemplo, a essência do ser humano era ser racional, para Sartre cada indivíduo
deveria produzir sua própria essência, pois nenhum deus havia criado o homem a partir de um conceito previamente
definido; isso significa, em última análise, que o indivíduo é o que ele faz de si mesmo, sendo que viver nada mais é
que produzir significados. Ao afirmar que “a existência do homem precede sua essência”, Sartre quis dizer que primeiro o homem surgiu no mundo, sem qualquer projeto concebido previamente por um ente metafísico (Deus); desse
modo, não havendo tal essência, todos os seres humanos seriam iguais e igualmente livres para se fazerem.
Período de grande efervescência crítica, a Filosofia contemporânea mostrou-se, em um primeiro momento, estarrecida diante das interrogações e desafios que a civilização trouxe enquanto caminhava rumo a um desenvolvimento
tecnológico, científico e cultural nunca antes visto ou mesmo imaginado. A cada descoberta, a cada nova perspectiva,
a cada novo paradigma, o homem contemporâneo sentiu-se em desconforto. Em um segundo momento, o pensamento
filosófico contemporâneo tentou formular hipóteses, respostas, alternativas para que o conhecimento humano pudesse
ser um modo de real compreensão desse novo universo que se abria para a civilização. Ainda assim, a cada dia o ser
humano se depara com novos desafios, e a Filosofia, apesar de ter perdido parte de sua área de influência em virtude
do surgimento de novas ciências, ainda tem sua importância enquanto instrumento intelectual para o entendimento do
todo que se forma com a junção da existência humana e do meio natural.
31
Fazer da vida algo inteligível, coerente e bem-sucedido a um ser racional e sedento por respostas é a tarefa da Filosofia, que se hoje ainda sobrevive, passados mais de dois mil e quinhentos anos desde seu surgimento, é porque
teve e tem grande relevância para o homem e um papel proeminente em todo e qualquer momento que exija a necessidade de se pensar com liberdade, com objetividade e a fim de encontrar respostas sensatas para que, nós, seres
humanos, façamos por merecer a capacidade cognitiva que nos proporcionou a natureza: pensar. Negar essa possibilidade é condenarmo-nos à sombra, ao esquecimento, excluindo sem motivos coerentes a chance de sermos protagonistas de nossas próprias descobertas, desafios, intenções e razões.
Leituras complementares
No seu texto “A Serenidade” e a propósito uma celebração em memória de um compositor, Conradin Kreutzer,
Heidegger convida-nos a refletir sobre a essência daquilo a que chamamos “pensar”, começando por dizer que toda a
comemoração exige que pensemos. Mas o que pensar, o que dizer numa festa consagrada à recordação de um músico? Ora bem, a questão é então, até que ponto celebrar uma festa em nome da música não equivale a celebrar uma
festa onde pensamos? Mas Heidegger diz-nos que não tenhamos ilusões, porque acontece a todos nós sermos pobres
em pensamentos mesmo aos que fazem do ofício de pensar — dever profissional.
A carência de pensamentos é um hóspede inquietante que se insinua por todo o lado no mundo de hoje. Nos dias
de hoje tudo se aprende da maneira mais rápida e mais econômica e no momento a seguir é tudo rapidamente esquecido. Por conseguinte, dentro em breve, uma celebração é suplantada por outra celebração e, assim, as festas comemorativas tornam-se cada vez mais pobres em pensamentos.
No entanto, Heidegger acentua a dimensão pensante, meditativa, pois a compreensão é uma característica humana
de forma que, mesmos que estejamos privados de pensar não renunciamos ao poder que temos de o fazer, tal como se
nos podemos tornar surdos é precisamente porque ouvimos e se podemos envelhecer é porque já fomos jovens. Da
mesma forma, se nos podemos tornar pobres em pensamentos ou até destituídos dos mesmos, é porque a essência do
homem é, justamente, pensar; como diria Pascal “O homem é visivelmente feito para pensar. É essa toda a sua dignidade e todo o seu mérito”.
Para Heidegger, a falta crescente de pensamentos repousa no processo que ataca a substância mais íntima do homem contemporâneo: o homem contemporâneo foge diante do pensamento e isso explica a falta de pensamentos e
mais, o homem contemporâneo não quer sequer reconhecer esta fuga, muito pelo contrário, ele afirma o oposto remetendo para tudo o que o conhecimento científico tem produzido. Um tal pensamento tornou-se indispensável e reveste-se de um caráter particular: trata-se do pensamento calculador — parte-se de um projeto que se impõe, de um
pressuposto que se põe em causa. O pensamento que conta, calcula. Submete ao cálculo as possibilidades todos os
dias novas, cada vez mais ricas em perspectivas e ao mesmo tempo mais econômicas.
O pensamento que calcula não nos deixa nenhum prazo e impele-nos de uma possibilidade a outra. O pensamento
calculador não persegue, efetivamente, o sentido — ele antecipa, não se espanta, não medita.
Há, assim, dois tipos de pensamento por sua vez legítimos: o pensamento que calcula e o pensamento que medita
e é este último que Heidegger tem em vista quando diz que o homem está em fuga perante o pensamento. Mas podemos perguntar: não andará o pensamento meditativo longe da realidade? Efetivamente, ele parece não ajudar nada nas
realizações de ordem prática. Não será este pensamento demasiado “exigente”, “elevado” para o entendimento comum? Na realidade, podemos até dizer que o pensamento meditativo é menos espontâneo que o pensamento calculador pois o pensamento que medita requer um esforço significativo, reclama alguns cuidados...
Por outro lado, qualquer um de nós pode, dentro dos seus limites seguir os caminhos da meditação, e por quê?
Por que o homem é um ser pensante, isto é, meditativo e não é necessário que a meditação nos leve até “regiões
superiores”. É preciso que nos fixemos sobre aquilo que nos é próximo. Heidegger sublinha aqui a tônica no enraizamento. É da circunstância que devemos partir. É a partir do solo natal que uma obra de arte é criada e concluída.
Haverá uma terra natal onde o homem permaneça enraizado? Poderá o homem do futuro ser um ser meditativo?
Como diz Heidegger, poderá o homem do futuro se desenvolver, poderá a sua obra amadurecer a partir de uma terra
natal já constituída?, ou ficarão as coisas presas nas garras da planificação e do cálculo, da organização e do automatismo?
O enraizamento do homem está hoje ameaçado no seu mais íntimo e não só devido apenas às circunstâncias exteriores ou ao modo de vida superficial do homem, mas sim ao espírito da época em que o osso nascimento nos fixou.
O pensamento técnico fortaleceu-se, efetivamente, na Modernidade, de forma que se chega a afirmar que nasceu com
ela. Mas se o pensamento técnico é algo que já existe no mundo grego, é na Modernidade que o paradigma da teoria e
da praxis para ser o paradigma técnico.
A nossa época está ameaçada pelo desenraizamento. Esta época tem o nome de idade atômica e a sua característica mais evidente é a bomba atômica. Heidegger diz-nos que o poder escondido no seio da técnica contemporânea
determina a relação do homem com aquilo que ele é. Ela reina sobre a terra inteira. O homem começa já a afastar-se
da terra para penetrar no espaço cósmico. A terra é, pois, transformada num espaço cósmico. Esta revolução radical
na nossa visão do mundo realiza-se na filosofia moderna. O mundo aparece como um objeto sobre o qual o pensamento que calcula dirige os seus ataques e a esses ataques nada deve resistir! A natureza torna-se num único reservatório gigante, uma fonte de energia para a técnica e indústria modernas. Seja como ‘era atômica’, seja como civilização de consumo, a época moderna é (...) caracterizada pela maneira como a humanidade quer pôr ao seu alcance a
totalidade do ente e adquirir sobre esta totalidade a maior força possível graças ao domínio de todas as energias naturais, incluindo as da destruição. E portanto, a questão não é, então, a da técnica propriamente dita mas a da relação do
homem com a técnica. O perigo depende do uso que se faz da técnica. É nesta relação com o mundo que o pensamento meditativo se mostra diferente.
32
Se nós conseguirmos dominar a energia atômica e conseguiremos, diz-nos Heidegger, começará um novo desenvolvimento do mundo técnico. Todas as técnicas que conhecemos hoje, desde os filmes à TV, à informação, alimentação, etc, são apenas tentativas. Ninguém pode prever os transtornos que se seguirão. Não conseguimos/podemos
travar os progressos da técnica e um dos traços deste novo mundo técnico é a rapidez com a qual os êxitos são conhecidos e publicamente admirados.
Mas o que é realmente inquietante não é que o nosso mundo se torne um mundo completamente técnico, mas antes que o homem não esteja preparado para essa transformação, que não se consiga explicar pelos meios do pensamento meditativo.
A questão a que a filosofia deverá hoje responder é à falta de preparação do homem para esta transformação, logo, a filosofia como pensamento hermenêutico deverá ser capaz de responder a isto. A filosofia vai ser caracterizada
por Heidegger como dimensão originária do existir.
De fato, o homem da era atômica será “atirado” sem aviso nem defesa na onda crescente da técnica. E sê-lo-á efetivamente s e renunciar ao pensamento meditativo assumindo o pensamento simplesmente calculador. A questão é
agora: será que a era atômica é uma fatalidade ou permitirá ela um novo enraizamento?
Poder-se-ia pensar que Heidegger nega a técnica, mas é evidente que não se trata de negar a técnica... dependemos dos objetos tecnológicos, a questão é que não nos podemos tornar seus escravos. O homem moderno é o funcionário da técnica. É possível utilizarmos os objetos tecnológicos servindo-nos deles e, ao mesmo tempo, deles nos
libertarmos, ou seja, podemos dizer “sim” à utilização da técnica, mas também “não” ao fato de a técnica monopolizar, desunir e violar o nosso ser. A questão é a de o homem não deixar que a técnica atinja o que temos de mais íntimo e de mais próximo. Heidegger sempre apelou a que se pensasse a técnica a partir da sua essência.
Mas dizer “sim” e “não” não significará um relacionamento ambíguo com o mundo? Não, muito pelo contrário,
torna-se um relacionamento mais pacífico. A serenidade consiste em admitir a técnica, os objetos tecnológicos ao
mesmo tempo em que os deixamos repousar sobre eles próprios como algo que não tem nada de absoluto. Por que é
que a serenidade não é, então, ambígua? Simplesmente porque há uma dimensão conflitual no homem. Em tudo o que
ele constrói reina um sentido que ele não recebe, que não constrói. O homem não é só espontaneidade, é também
negatividade.
Apesar de, no limite, o homem ser votado ao nada, é também um ser que exige sentido e isto leva Heidegger da
Antropologia à Ontologia. Apesar da negatividade, o homem é afirmação originária, é postulado de sentido. A Modernidade descobriu o homem como exigência de sentido ao descobrir o “cogito”, mas esqueceu que é também sentido. Heidegger vai partir desta situação de negatividade que caracteriza o homem, pois ele é isso embora não coincida
com isso.
Esta exigência incondicional de sentido que o homem é, leva Heidegger a pensar que a raiz do sentido é qualquer
coisa que excede o próprio homem. Para Heidegger, o homem é definido como pré-compreensão do sentido. O que
constitui a sua essência é a exigência de sentido, a esperança, o desejo de ser na falta do próprio Ser. Heidegger toma
consciência desta experiência “de contraste”, desta dialética do existir. Na Modernidade a orientação para o sentido
aparece como coincidência. Para Heidegger, não. Apesar de o homem ser orientação para o sentido, ele ainda não é,
está a ser, ele é excesso que não coincide e é muito mais do que é. É só perante esta experiência de negatividade que
ele pode tomar consciência daquilo que é.
O sentido do mundo técnico oculta-se. Deixar-se entrever e ao mesmo tempo ocultar-se não é o traço fundamental
daquilo a que chamamos segredo? Para Heidegger, na raiz da técnica está a tomada de posição face à verdade. Subjaz, de fato, à ciência moderna uma noção de ente substancializada/sujeito puro e uma ideia de verdade como adequação/certeza. Mas a verdade não é adequação, mas desvelamento. Em que é que Heidegger fundamenta a ideia de que
a verdade é revelação? O fato de o dasein estar no mundo leva-o à revelação. O dasein começa por existir no mundo
segundo um modelo relacional, segundo o modelo da disponibilidade. A verdade como revelação tem como fundamento a verdade como pressuposição. Não existe verdade em si, mas verdade para o homem, porque ele acredita
nela. O homem é expectativa de verdade. A verdade é inerente ao homem, mas com a qual ele não coincide. É isto
que levará Gadamer, por exemplo, a recuperar o modelo da obra de arte.
A verdade é, então, desvelamento. A dimensão técnica perde a dimensão da luz. Quando a luz é considerada única, todo o segredo é confundido com ambiguidade negativa. Se virmos que a técnica tem também uma dimensão que
se oculta por detrás de si própria, estamos já no caminho de uma boa relação com ela. Trata-se, pois, de restaurar a
dimensão de segredo/oculto da verdade. É o restaurar desta dimensão que vai ser o objeto do pensamento meditativo
e só se pode restaurar essa dimensão quando o homem tomar consciência da sua historicidade.
O que levanta a questão da historicidade é uma questão muita mais radical, mais significativa que a questão ontológica.
Por que levanta Heidegger a questão ontológica, a questão do Ser? A experiência do sentido é a experiência fundamental. Qual é o pressuposto do sujeito transcendental? A sua condição “desencarnada”. O sentido escapa ao homem. Aparece-lhe como aquilo que escapa à representação. O Ser é agora a questão fundamental de tudo. A questão
é agora a questão do ser e esta é a questão fundamental do homem quando confrontado com a sua negatividade.
A experiência da historicidade como negatividade confronta o homem com aquilo que ele não é. Heidegger mostra em Ser e Tempo que é necessário repor a questão do Ser e analisá-la. Ela foi mal pensada pela tradição. Pensar a
historicidade é repensar a questão do Ser. A questão do ser do homem não é conforme com a ideia de uma ontologia
substancialista. Para compreender a historicidade é preciso tomar a questão do existir como dasein. É do dasein que
se parte para chegar à questão do seu sentido r da sua historicidade.
Por que motivo, então, se torna urgente recuperar a dimensão meditativa do pensamento?...
Heidegger sublinha que o que o grande perigo que nos ameaça é, de fato, a total falta de pensamentos, a robotização do homem. Somos seres finitos, mas ao mesmo tempo, abertos ao que nos transcende. É esta a condição finita do
homem. É necessário que o homem não rejeite aquilo que possui de mais próprio — o fato de ser um ser pensante.
33
Trata-se, então, de salvar essa essência do homem. Trata-se de manter acordado o pensamento. A Modernidade esqueceu o Ser, a realidade. Uma coisa é viver absorvido pela técnica, outra coisa é ler o mundo, habitar num mundo
lendo a outra dimensão do sentido literal ou técnico que essa dimensão tem.
Assim quando despertar em nós a identidade da alma perante as coisas, e o espírito se abrir ao outro, podemos esperar alcançar um novo caminho, uma nova terra, um novo solo. Nesse solo, a criação de obras perduráveis pode
enraizar-se de novo.
MAIA, Isabel. Sobre a “Serenidade” de Martin Heidegger.
φ
Algures na Índia. Uma fila de peças de artilharia em posição. Atado à boca de cada uma delas há um homem. No
primeiro plano da fotografia um oficial britânico ergue a espada e vai dar ordem de fogo. Não dispomos de imagens
do efeito dos disparos, mas até a mais obtusa das imaginações poderá “ver” cabeças e troncos dispersos pelo campo
de tiro, restos sanguinolentos, vísceras, membros amputados. Os homens eram rebeldes. Algures em Angola. Dois
soldados portugueses levantam pelos braços um negro que talvez não esteja morto, outro soldado empunha um machete e prepara-se para lhe separar a cabeça do corpo. Esta é a primeira fotografia. Na segunda, desta vez há uma
segunda fotografia, a cabeça já foi cortada, está espetada num pau, e os soldados riem. O negro era um guerrilheiro.
Algures em Israel. Enquanto alguns soldados israelitas imobilizam um palestino, outro militar parte-lhe à martelada
os ossos da mão direita. O palestino tinha atirado pedras. Estados Unidos da América do Norte, cidade de Nova York.
Dois aviões comerciais norte-americanos, sequestrados por terroristas relacionados com o integrismo islâmico, lançam-se contra as torres do World Trade Center e deitam-nas abaixo. Pelo mesmo processo um terceiro avião causa
danos enormes no edifício do Pentágono, sede do poder bélico dos States. Os mortos, soterrados nos escombros,
reduzidos a migalhas, volatilizados, contam-se por milhares.
As fotografias da Índia, de Angola e de Israel atiram-nos com o horror à cara, as vítimas são-nos mostradas no
próprio instante da tortura, da agônica expectativa, da morte ignóbil. Em Nova York tudo pareceu irreal ao princípio,
episódio repetido e sem novidade de mais uma catástrofe cinematográfica, realmente empolgante pelo grau de ilusão
conseguido pelo engenheiro de efeitos especiais, mas limpo de estertores, de jorros de sangue, de carnes esmagadas,
de ossos triturados, de merda. O horror, agachado como um animal imundo, esperou que saíssemos da estupefação
para nos saltar à garganta. O horror disse pela primeira vez “aqui estou” quando aquelas pessoas saltaram para o
vazio como se tivessem acabado de escolher uma morte que fosse sua. Agora o horror aparecerá a cada instante ao
remover-se uma pedra, um pedaço de parede, uma chapa de alumínio retorcida, e será uma cabeça irreconhecível, um
braço, uma perna, um abdômen desfeito, um tórax espalmado. Mas até mesmo isto é repetitivo e monótono, de certo
modo já conhecido pelas imagens que nos chegaram daquele Ruanda-de-um-milhão-de-mortos, daquele Vietnã cozido a napalme, daquelas execuções em estádios cheios de gente, daqueles linchamentos e espancamentos daqueles
soldados iraquianos sepultados vivos debaixo de toneladas de areia, daquelas bombas atômicas que arrasaram e calcinaram Hiroshima e Nagasaki, daqueles crematórios nazistas a vomitar cinzas, daqueles caminhões a despejar cadáveres como se de lixo se tratasse. De algo sempre haveremos de morrer, mas já se perdeu a conta aos seres humanos
mortos das piores maneiras que seres humanos foram capazes de inventar. Uma delas, a mais criminosa, a mais absurda, a que mais ofende a simples razão, é aquela que, desde o princípio dos tempos e das civilizações, tem mandado
matar em nome de Deus. Já foi dito que as religiões, todas elas, sem exceção, nunca serviram para aproximar e congraçar os homens, que, pelo contrário, foram e continuam a ser causa de sofrimentos inenarráveis, de morticínios, de
monstruosas violências físicas e espirituais que constituem um dos mais tenebrosos capítulos da miserável história
humana. Ao menos em sinal de respeito pela vida, deveríamos ter a coragem de proclamar em todas as circunstâncias
esta verdade evidente e demonstrável, mas a maioria dos crentes de qualquer religião não só fingem ignorá-lo, como
se levantam iracundos e intolerantes contra aqueles para quem Deus não é mais que um nome, nada mais que um
nome, o nome que, por medo de morrer, lhe pusemos um dia e que viria a travar-nos o passo para uma humanização
real. Em troca prometeram-nos paraísos e ameaçaram-nos com infernos, tão falsos uns como outros, insultos descarados a uma inteligência e a um sentido comum que tanto trabalho nos deram a criar. Disse Nietzsche que tudo seria
permitido se Deus não existisse, e eu respondo que precisamente por causa e em nome de Deus é que se tem permitido e justificado tudo, principalmente o pior, principalmente o mais horrendo e cruel. Durante séculos a Inquisição foi,
ela também, como hoje os talebanes, uma organização terrorista que se dedicou a interpretar perversamente textos
sagrados que deveriam merecer o respeito de quem neles dizia crer, um monstruoso conúbio pactuado entre a religião
e o Estado contra a liberdade de consciência e contra o mais humano dos direitos: o direito a dizer não, o direito à
heresia, o direito a escolher outra coisa, que isso só a palavra heresia significa.
E, contudo, Deus está inocente. Inocente como algo que não existe, que não existiu nem existirá nunca, inocente
de haver criado um universo inteiro para colocar nele seres capazes de cometer os maiores crimes para logo virem
justificar-se dizendo que são celebrações do seu poder e da sua glória, enquanto os mortos se vão acumulando, estes
das torres gêmeas de Nova York, e todos os outros que, em nome de um Deus tornado assassino pela vontade e pela
ação dos homens, cobriram e teimam em cobrir de terror e sangue as páginas da história. Os deuses, acho eu, só
existem no cérebro humano, prosperam ou definham dentro do mesmo universo que os inventou, mas o “fator Deus”,
esse, está presente na vida como se efetivamente fosse o dono e o senhor dela. Não é um deus, mas o “fator Deus” o
que se exibe nas notas de dólar e se mostra nos cartazes que pedem para a América (a dos Estados Unidos, não a
outra...) a bênção divina. E foi o “fator Deus” em que o deus islâmico se transformou, que atirou contra as torres do
World Trade Center os aviões da revolta contra os desprezos e da vingança contra as humilhações. Dir-se-á que um
deus andou a semear ventos e que outro deus responde agora com tempestades. É possível, é mesmo certo. Mas não
foram eles, pobres deuses sem culpa, foi o “fator Deus”, esse que é terrivelmente igual em todos os seres humanos
onde quer que estejam e seja qual for a religião que professem, esse que tem intoxicado o pensamento e aberto as
34
portas às intolerâncias mais sórdidas, esse que não respeita senão aquilo em que manda crer, esse que depois de presumir ter feito da besta um homem acabou por fazer do homem uma besta.
Ao leitor crente (de qualquer crença...) que tenha conseguido suportar a repugnância que estas palavras provavelmente lhe inspiraram, não peço que se passe ao ateísmo de quem as escreveu. Simplesmente lhe rogo que compreenda, pelo sentimento de não poder ser pela razão, que, se há Deus, há só um Deus, e que, na sua relação com ele, o
que menos importa é o nome que lhe ensinaram a dar. E que desconfie do “fator Deus”. Não faltam ao espírito humano inimigos, mas esse é um dos mais pertinazes e corrosivos. Como ficou demonstrado e desgraçadamente continuará
a demonstrar-se.
SARAMAGO, José. O fator Deus.
Cogito, logo aprendo
1. (UFMG 2002) “O homem é, antes de mais nada, um projeto que se vive subjetivamente, em vez de ser um
creme, qualquer coisa podre ou uma couve-flor; nada existe anteriormente a este projeto; nada há no céu inteligível, e
o homem será antes de mais o que tiver projetado ser. Não o que ele quiser ser. Porque o que entendemos vulgarmente por querer é uma decisão consciente e que, para a maior parte de nós, é posterior àquilo que ele próprio se fez.
Posso querer aderir a um partido, escrever um livro, casar-me; tudo isso não é mais do que a manifestação duma
escolha mais original, mais espontânea do que o que se chama de vontade. Mas se verdadeiramente a existência
precede a essência, o homem é responsável por aquilo que é. [...] Se, com efeito, a existência precede a essência, não
será nunca possível referir uma explicação a uma natureza dada e imutável; por outras palavras, não há determinismo,
o homem é livre, o homem é liberdade. [...] o homem, sem qualquer apoio e sem qualquer auxílio, está condenado a
cada instante a inventar o homem.” [SARTRE, Jean-Paul. O existencialismo é um humanismo.]
Com base na leitura desse trecho e em outros conhecimentos presentes nessa obra de Sartre,
a. EXPLIQUE com as suas próprias palavras o que o autor quer dizer ao afirmar que a existência “a existência
precede a essência.
b. ARGUMENTE a favor de ou contra a ideia do autor de que o “o homem está condenado a ser livre”.
2. (UFMG 2003) “O intelecto, como um meio para conservação do indivíduo, desdobra suas forças mestras no
disfarce; pois este é o meio pelo qual os indivíduos mais fracos, menos robustos, se conservam, aqueles aos quais está
vedado travar uma luta pela existência com chifres ou presas aguçadas. [...] O que é a verdade, portanto? Um batalhão móvel de metáforas, metonímias, antropomorfismos, enfim, uma soma de relações humanas, que foram enfatizadas poética e retoricamente, transpostas, enfeitadas, e que, após longo uso, parecem a um povo sólidas, canônicas e
obrigatórias: as verdades são ilusões das quais se esqueceu que o são, metáforas que se tornaram gastas e sem força
sensível, moedas que perderam sua efígie e agora só entram em consideração como metal, não mais como moedas.”
[NIETZSCHE, Friedrich. Sobre a verdade e a mentira no sentido extramoral.]
A partir da leitura desses trechos e de outras ideias presentes nesta obra de Nietzsche, JUSTIFIQUE por que, para
o autor, “as verdades são ilusões das quais se esqueceu que o são.”
3. (UFMG 2006) Leia este poema:
Vocês que vivem seguros
em suas cálidas casas,
vocês que, voltando à noite,
encontram comida quente e rostos amigos,
pensem bem se isto é um homem
que trabalha no meio do barro,
que não conhece a paz,
que luta por um pedaço de pão,
que morre por um sim por um não.
Pensem bem se isto é uma mulher,
sem cabelos e sem nome,
sem mais força para lembrar,
vazios os olhos, frio o ventre,
como um sapo no inverno.
Pensem que isto aconteceu:
eu lhes mando estas palavras.
Gravem-nas em seus corações,
Estando em casa, andando na rua,
ao deitar, ao levantar;
repitam-nas a seus filhos.
Ou, senão, desmorone-se a sua casa,
a doença os torne inválidos,
os seus filhos virem o rosto para não vê-los.
[LEVI, Primo*. É isto um homem?]
(* Primo Levi (1919-1987), escritor e químico italiano, sobrevivente de Auschwitz, campo de concentração nazista)
REDIJA um texto respondendo, do seu ponto de vista, à pergunta enunciada no título do poema: “É isto um homem?”
35
Capítulo VI
LÓGICA E EPISTEMOLOGIA
LAERTE. “Piratas do Tietê”. In: Folha de S. Paulo (17 de julho de 2009).
indos os cinco primeiros capítulos deste curso, nos quais vimos as épocas e os paradigmas pelos quais a Filosofia passou no decorrer de sua existência, vejamos agora, por outra perspectiva, alguns campos de estudo do
pensamento filosófico. Se levarmos em consideração que a matemática, por exemplo, é subdividida em diversas disciplinas (tais como a geometria, a aritmética etc.), veremos que a Filosofia, do mesmo modo, tem subdivisões. Algumas delas se destacam por englobar questões de extrema importância para o fazer filosófico e, por
isso, serão as que trataremos nos três próximos capítulos. São elas: a Lógica, a Epistemologia, a Metafísica e a Ética.
Vale lembrar, nesse ínterim, que assim como as ideias e os autores tratados nos primeiros capítulos não são os únicos que tiveram sua importância na História da Filosofia, essas também não são as únicas divisões filosóficas, de
modo que, por ser a Filosofia um campo de estudo extremamente vasto, daremos nossa atenção apenas às subdivisões
mais importantes. Elucidado isso, comecemos.
Neste capítulo trataremos da Lógica. Vejamos, pois, o que é e o que estuda esse ramo da Filosofia. Em síntese,
F
a Lógica estuda e sistematiza a validade ou invalidade da argumentação. Também se diz que estuda inferências ou raciocínios.
Podes considerar que argumentos, inferências e raciocínios são termos equivalentes. Muito bem, a Lógica estuda argumentos.
Mas qual é o interesse disso para a Filosofia? Bem, tenho de te lembrar que a argumentação é o coração da Filosofia. Em Filosofia temos a liberdade de defender as nossas ideias, mas temos de sustentar o que defendemos com bons argumentos e, é
claro, também temos de aceitar discutir os nossos argumentos. Os argumentos constituem um dos três elementos centrais da
Filosofia. Os outros dois são os problemas e as teorias. Com efeito, ao longo dos séculos, os filósofos têm procurado resolver
problemas, criando teorias que se apoiam em argumentos. Estás a ver por que é que o estudo dos argumentos é importante, isto é, por que é que a Lógica é importante. É importante, porque nos ajuda a distinguir os argumentos válidos dos inválidos,
permite-nos compreender por que razão uns são válidos e outros não e ensina-nos a argumentar correctamente. [PADRÃO, António Aníbal. Algumas noções de Lógica.]
Sabendo-se que uma argumentação adequada é fundamental para a Filosofia e que uma argumentação é constituída por argumentos, é inevitável que surja a questão: mas o que é um argumento? Um argumento nada mais é do que
um conjunto de proposições, de conjecturas que, quando as usamos, nos capacitam a explicar, a justificar, a dar suporte, a provar algo que queremos explicar, justificar, dar suporte, provar... Nesse meio, damos o nome de “conclusão” à proposição que queremos justificar, e denominamos “premissa” toda e qualquer proposição que usamos a fim
de apoiar e justificar a conclusão.
Lembremo-nos, pois, que no primeiro capítulo afirmamos ser a Filosofia um conhecimento metódico, objetivo, a
considerar que a razão atua a partir de determinados princípios que ela própria institui e que estão em concordância
com a própria realidade. O conhecimento racional, assim, obedece a certas leis fundamentais, princípios capitais.
Dentre tais princípios encontramos especialmente quatro, a saber, o “princípio da identidade”, o “princípio da nãocontradição”, o “princípio do terceiro excluído” e o “princípio da razão suficiente”.
O princípio da identidade pode ser enunciado do seguinte modo (inacreditavelmente óbvio em um primeiro momento): “X é X” ou “O que é, é”. Tal princípio, ainda que seja evidente ou mesmo “bobo”, é a condição do pensamento sem a qual não poderíamos pensar; por esse princípio, afirma-se que algo, seja o que for (um ser da Natureza,
uma figura geométrica, um ser humano, uma obra de arte, uma ação), pode ser conhecido e raciocinado somente se
for percebido e mantido com a identidade que possui. O princípio da identidade é, portanto, a condição básica para
que definamos as coisas e tenhamos a capacidade de conhecê-las a partir de suas definições.
Por exemplo, depois que um matemático definir o triângulo como figura de três lados e de três ângulos, não só nenhuma outra
figura que não tenha esse número de lados e de ângulos poderá ser chamada de triângulo como também todos os teoremas e
problemas que o matemático demonstrar sobre o triângulo, só poderão ser demonstrados se, a cada vez que ele disser “triângulo”, soubermos a qual ser ou a qual coisa ele está se referindo. [CHAUÍ, Marilena. Convite à filosofia.]
Já o princípio da não-contradição enuncia-se do seguinte modo: “X é X e é impossível que seja, ao mesmo tempo e na mesma relação, não-X”. Com isso, o princípio da não-contradição garante que uma coisa ou uma ideia que se
negam a si mesmas se autodestroem, deixam de existir; as coisas e as ideias contraditórias são, por isso, impossíveis e
impensáveis.
Assim, é impossível que a árvore que está diante de mim seja e não seja uma mangueira; que o cachorrinho de dona Filomena
seja e não seja branco; que o triângulo tenha e não tenha três lados e três ângulos; que o homem seja e não seja mortal; que o
36
vermelho seja e não seja vermelho, etc. Sem o princípio da não-contradição, o princípio da identidade não poderia funcionar.
[Idem.]
Por sua vez, o princípio do terceiro excluído pode ser exprimido como se segue: “ou A é x ou é y, e não existe
uma terceira possibilidade”. Por esse princípio, define-se a decisão de um dilema (“ou isto ou aquilo”) e determina-se
que apenas uma das alternativas seja verdadeira.
Por exemplo: “Ou este homem é Sócrates ou não é Sócrates”; “Ou faremos a guerra ou faremos a paz”. [...] Mesmo quando
temos, por exemplo, um teste de múltipla escolha, escolhemos na verdade apenas entre duas opções — “ou está certo ou está
errado” — e não há terceira possibilidade ou terceira alternativa, pois, entre várias escolhas possíveis, só há realmente duas,
a certa ou a errada. [Idem.]
Enfim, o princípio da razão suficiente é enunciado do seguinte modo: “dado X, necessariamente se dará Y” (e,
por conseguinte, “dado Y, necessariamente houve X”); tal princípio alega que tudo o que existe e tudo o que acontece
possui uma razão, uma causa, um motivo para existir ou mesmo para acontecer, de modo que tal motivo pode ser
conhecido pela razão humana. Também chamado de “princípio da causalidade”, o princípio da razão suficiente afirma, por meio da razão, a existência de relações ou conexões internas entre as coisas, entre os fatos, entre as ações e os
acontecimentos. Assim, podemos reunir algumas características importantes desse princípio:
- não possuem um conteúdo determinado, pois são formas: indicam como as coisas devem ser e como devemos pensar, mas não
nos dizem quais coisas são, nem quais os conteúdos que devemos ou vamos pensar;
- possuem validade universal, isto é, onde houver razão (nos seres humanos e nas coisas, nos fatos e nos acontecimentos), em
todo o tempo e em todo lugar, tais princípios são verdadeiros e empregados por todos (os humanos) e obedecidos por todos
(coisas, fatos, acontecimentos);
- são necessários, isto é, indispensáveis para o pensamento e para a vontade, indispensáveis para as coisas, os fatos e os acontecimentos. Indicam que algo é assim e não pode ser de outra maneira. Necessário significa: é impossível que não seja dessa
maneira e que pudesse ser de outra. [Idem.]
Visto isso, e tendo-se em mente que argumentar faz parte do que conhecemos por atividade racional, temos que a
Filosofia distingue duas grandes seções da atividade racional, desempenhadas que são pelo sujeito do conhecimento;
são elas: a “razão intuitiva” (ou simplesmente “intuição”) e a “razão discursiva” (ou apenas “raciocínio”).
A atividade racional discursiva, como a própria palavra indica, discorre, percorre uma realidade ou um objeto para chegar a
conhecê-lo, isto é, realiza vários atos de conhecimento até conseguir captá-lo. A razão discursiva ou o pensamento discursivo
chega ao objeto passando por etapas sucessivas de conhecimento, realizando esforços sucessivos de aproximação para chegar
ao conceito ou à definição do objeto. A razão intuitiva ou intuição, ao contrário, consiste num único ato do espírito, que, de
uma só vez, capta por inteiro e completamente o objeto. Em latim, intuitos significa: ver. A intuição é uma visão direta e imediata do objeto do conhecimento, um contato direto e imediato com ele, sem necessidade de provas ou demonstrações para saber
o que conhece. [Idem.]
Agora, vejamos mais detidamente as duas modalidades da atividade racional, começando pela razão intuitiva. E
para entendermos o que é isso, tenhamos em mente que “intuição” é uma compreensão imediata e completa de um
fato, de uma verdade, de um objeto; por meio da intuição, e de uma só vez,
a razão capta todas as relações que constituem a realidade e a verdade da coisa intuída. É um ato intelectual de discernimento
e compreensão, como, por exemplo, tem um médico quando faz um diagnóstico e apreende de uma só vez a doença, sua causa e
o modo de tratá-la. Os psicólogos se referem à intuição usando o termo insight, para referirem-se ao momento em que temos
uma compreensão total, direta e imediata de alguma coisa, ou o momento em que percebemos, num só lance, um caminho para
a solução de um problema científico, filosófico ou vital. [Idem.]
A razão intuitiva pode ser de duas espécies: “intuição sensível” (ou “empírica”) e “intuição intelectual”. Vejamos
esses dois tipos:
A intuição sensível ou empírica é o conhecimento direto e imediato das qualidades sensíveis de um objeto externo a nós (odores, cores, sabores, gostos, dimensões, texturas etc.); é, além disso, o conhecimento direto e imediato
dos estados internos ou mentais de nós mesmos (sentimentos, desejos, lembranças etc.). A intuição sensível é, portanto, um tipo de conhecimento que possuímos em todos os momentos de nossa vida: com um rápido olhar podemos
perceber uma sala de aula, suas paredes, o teto, alguém diante de um grupo de alunos, as carteiras, a lousa etc. Em
uma rápida ação visual qualquer um é capaz (desde que tenha a visão normal) de olhar para alguém e notar a cor dos
olhos, dos cabelos, o tamanho, a compleição física da pessoa, o que ela veste, o que ela calça etc., de modo que, com
essa intuição empírica, temos essa pessoa por inteiro e de uma só vez diante de nós.
Se levarmos em consideração o importante fato de que a intuição sensível ou empírica tem natureza psicológica
(ou seja, refere-se à situação do sujeito do conhecimento enquanto ser corporal, psíquico e individual [desejos, sensações, lembranças, sentimentos e percepções são exclusivamente pessoais]), teremos que a marca da intuição empírica
é sua singularidade:
por um lado, está ligada à singularidade do objeto intuído (ao “isto” oferecido à sensação e à percepção) e, por outro, está ligada à singularidade do sujeito que intui (aos “meus” estados psíquicos, às “minhas” experiências). A intuição empírica não
capta o objeto em sua universalidade e a experiência intuitiva não é transferível para um outro objeto. [Idem.]
37
Por sua vez, a intuição intelectual difere da intuição sensível exatamente por sua necessidade e universalidade.
Se, por exemplo, pensarmos que “algo não pode ser e não ser ao mesmo tempo”, sabemos, sem necessidade de exames, provas e demonstrações, que isso é verdade; já temos, pois, o conhecimento intuitivo do princípio da nãocontradição. Se, por outro exemplo, afirmamos que “o azul é diferente do verde”, sabemos, sem necessidade de exames, provas e demonstrações, que há diferenças entre as duas cores: vemos, com a intuição sensível, as cores amarela
e azul, mas vemos, com a intuição intelectual, a diferença que há entre as cores.
Em resumo, a intuição intelectual é o conhecimento direto e imediato dos princípios da razão (identidade, contradição, terceiro excluído, razão suficiente), das relações necessárias entre os seres ou entre as ideias, da verdade de uma ideia ou de um ser.
Na história da Filosofia, o exemplo mais célebre de intuição intelectual é conhecido como o cogito cartesiano, isto é, a afirmação de Descartes: “Penso (cogito), logo existo”. De fato, quando penso, sei que estou pensando e não é preciso provar ou demonstrar isso, mesmo porque provar e demonstrar é pensar e para demonstrar e provar é preciso, primeiro, pensar e saber que
se pensa. Quando digo: “Penso, logo existo”, estou simplesmente afirmando racionalmente que sei que sou um ser pensante ou
que existo pensando, sem necessidade de provas e demonstrações. A intuição capta, num único ato intelectual, a verdade do
pensamento pensando em si mesmo. [Idem.]
Podemos, se quisermos, também, apontar outro tipo de intuição, qual seja, a intuição emotiva (ou valorativa), que
seria aquela intuição com a qual, em conjunto com o sentido ou significação de algo, apreendemos também seu valor;
ou seja, com a ideia compreendemos, intuímos igualmente se tal algo é verdadeiro ou falso, justo ou injusto, coerente
ou incoerente, feio ou belo, mau ou bom, possível ou impossível etc.
A intuição intelectual capta, portanto, a essência do objeto (o que ele é) e a intuição emotiva ou valorativa capta essa essência
pelo que o objeto vale. [Idem.]
Agora, entendamos mais a fundo a razão discursiva ou raciocínio. Esta, ao contrário da intuição, é um conhecimento que exige provas, comprovações e demonstrações, realizando-se igualmente por meio de provas, comprovações e demonstrações das verdades que estão sendo conhecidas ou investigadas. O raciocínio não é, todavia, um ato
intelectual, mas vários atos intelectuais ligados internamente, desenvolvendo um processo de conhecimento.
Um caçador sai pela manhã em busca da caça. Entra no mato e vê rastros: choveu na véspera e há pegadas no chão; pequenos
galhos rasteiros estão quebrados; o capim está amassado em vários pontos; a carcaça de um bicho está à mostra, indicando
que foi devorado há poucas horas; há um grande silêncio no ar, não há canto de pássaros, não há ruídos de pequenos animais.
O caçador supõe que haja uma onça por perto. Ele pode, então, tomar duas atitudes. Se, por todas as experiências anteriores,
tiver certeza de que a onça está nas imediações, pode preparar-se para enfrentá-la: sabe que caminhos evitar, se não estiver
em condições de caçá-la; sabe que armadilhas armar, se estiver pronto para capturá-la; sabe como atraí-la, se quiser conservá-la viva e preservar a espécie. O caçador pode ainda estar sem muita certeza se há ou não uma onça nos arredores e, nesse
caso, tomará uma série de atitudes para verificar a presença ou ausência do felino: pode percorrer trilhas que sabem serem
próprias de onças; pode examinar melhor as pegadas e o tipo de animal que foi devorado; pode comparar, em sua memória,
outras situações nas quais esteve presente uma onça, etc. Assim, partindo de indícios, o caçador raciocina para chegar a uma
conclusão e tomar uma decisão. Temos aí um exercício de raciocínio empírico e prático (isto é, um pensamento que visa a uma
ação) e que se assemelha à intuição sensível ou empírica, isto é, caracteriza-se pela singularidade ou individualidade do sujeito e do objeto do conhecimento. [Idem.]
Entretanto, quando um raciocínio é realizado a partir de condições tais que a individualidade psicológica do sujeito do conhecimento e a singularidade do objeto são substituídas por critérios de universalidade e generalidade, temos
a “dedução”, a “indução” e a “abdução”. A dedução e a indução
são procedimentos racionais que nos levam do já conhecido ao ainda não conhecido, isto é, permitem que adquiramos conhecimentos novos graças a conhecimentos já adquiridos. Por isso, se costuma dizer que, no raciocínio, o intelecto opera seguindo
cadeias de razões ou os nexos e conexões internos e necessários entre as ideias ou entre os fatos. [Idem.]
GONSALES, Fernando. “Níquel Náusea”. In: Folha de S. Paulo (24 de julho de 2009).
Mais precisamente, a dedução consiste em partir de uma verdade já conhecida, seja por intuição, seja por uma
demonstração anterior, e que funciona como um princípio geral ao qual se subordinam todos e quaisquer casos a
serem demonstrados a partir dela. No processo dedutivo, parte-se de uma verdade já conhecida para demonstrar que
ela pode ser aplicada a quaisquer casos particulares, desde que sejam iguais. Seu ponto de partida é ou uma ideia
verdadeira ou uma teoria verdadeira; no caso de uma teoria, a dedução admitirá que cada caso particular descoberto
seja conhecido, evidenciando que a ele podem ser aplicadas todas as leis, regras e verdades da teoria.
38
Por exemplo, se definirmos o triângulo como uma figura geométrica cujos lados somados são iguais à soma de dois ângulos
retos, dela deduziremos todas as propriedades de todos os triângulos possíveis. Se tomarmos como ponto de partida as definições geométricas do ponto, da linha, da superfície e da figura, deduziremos todas as figuras geométricas possíveis. [Idem.]
A dedução, assim, é um procedimento pelo qual um fato ou objeto particulares são conhecidos a partir de sua inclusão numa teoria geral. Para representá-la, costuma-se utilizar a seguinte fórmula:
Todos os x são y (definição);
A é x (caso particular);
Logo, A é y (dedução).
Exemplificando, temos:
Todos os estudantes do Cursinho Metamorfose (x) passarão no vestibular (y);
Você (A) é um estudante do Cursinho Metamorfose (x);
Logo, você (A) passará no vestibular (y).
Já a indução realiza um caminho contrário ao da dedução: partimos, na indução, de casos particulares iguais ou
semelhantes e buscamos uma lei geral, uma definição geral ou uma teoria geral que seja capaz de explicar todos os
casos particulares. Assim como ocorre com a dedução, a razão também oferece um conjunto de leis precisas para
orientar a indução; se tais regras não forem seguidas, a indução será considerada falsa.
Por exemplo, colocamos água no fogo e observamos que ela ferve e se transforma em vapor; colocamos leite no fogo e vemos
também que ele se transforma em vapor; colocamos vários tipos de líquidos no fogo e vemos sempre sua transformação em vapor. Induzimos desses casos particulares que o fogo possui uma propriedade que produz a evaporação dos líquidos. Essa propriedade é o calor. Verificamos, porém, que os diferentes líquidos não evaporam sempre na mesma velocidade; cada um deles,
portanto, deve ter propriedades específicas que os fazem evaporar em velocidades diferentes. Descobrimos, porém, que a velocidade da evaporação não é o fato a ser observado e sim quanto de calor cada líquido precisa para começar a evaporar. Se
considerarmos a água nosso padrão de medida, diremos que ela ferve e começa a evaporar a partir de uma certa quantidade
de calor e que é essa quantidade de calor que precisa ser conhecida. Podemos, a seguir, verificar um fenômeno diferente. Vemos que água e outros líquidos, colocados num refrigerador, endurecem e se congelam, mas que, como no caso do vapor, cada
líquido se congela ou se solidifica em velocidades diferentes. Procuramos, novamente, a causa dessa diferença de velocidade e
descobrimos que depende tanto de certas propriedades de cada líquido quanto da quantidade de frio que há no refrigerador.
Percebemos, finalmente, que é essa quantidade que devemos procurar. Com essas duas séries de fatos (vapor e congelamento),
descobrimos que os estados dos líquidos variam (evaporação e solidificação) em decorrência da temperatura ambiente (calor e
frio) e que cada líquido atinge o ponto de evaporação ou de solidificação em temperaturas diferentes. Com esses dados podemos formular uma teoria da relação entre os estados da matéria — sólido, líquido e gasoso — e as variações de temperatura,
estabelecendo uma relação necessária entre o estado de um corpo e a temperatura ambiente. Chegamos, por indução, a uma
teoria. [Idem.]
Conhecidas também pelo nome de “inferência” (ou seja, concluir alguma coisa a partir de outra já conhecida), a
dedução e a indução podem ser assim resumidas: no processo dedutivo, “se X, concluo a, b, c, d”; no processo indutivo, “se a, b, c, d, concluo X”. No entanto, alguns filósofos (como Charles Sanders Pierce [1839-1914]) consideram
que, além da dedução e da indução, a razão discursiva também se realiza por meio de uma terceira modalidade de
inferência, ainda que esta não seja exatamente demonstrativa: a “abdução”.
A abdução é uma espécie de intuição, mas que não se dá de uma só vez, indo passo a passo para chegar a uma conclusão. A
abdução é a busca de uma conclusão pela interpretação racional de sinais, de indícios, de signos. O exemplo mais simples oferecido por Peirce para explicar o que seja a abdução são os contos policiais, o modo como os detetives vão coletando indícios
ou sinais e formando uma teoria para o caso que investigam. Segundo Peirce, a abdução é a forma que a razão possui quando
inicia o estudo de um novo campo científico que ainda não havia sido abordado. Ela se aproxima da intuição do artista e da
adivinhação do detetive, que, antes de iniciarem seus trabalhos, só contam com alguns sinais que indicam pistas a seguir. Os
historiadores costumam usar a abdução. De modo geral, diz-se que a indução e a abdução são procedimentos racionais que
empregamos para a aquisição de conhecimentos, enquanto a dedução é o procedimento racional que empregamos para verificar ou comprovar a verdade de um conhecimento já adquirido. [Idem.]
Como vimos, a Lógica é o campo da Filosofia que fundamenta e sustenta as regras da argumentação correta, sendo, por isso, um instrumento do pensar. Ainda que a Lógica não tenha um fim em si, ela conquista grande importância e sentido enquanto meio de assegurar que o pensamento decorra de maneira correta, adequada, com o objetivo de
se chegar a conhecimentos verdadeiros.
Vejamos, agora, outra grande e importante subdivisão da Filosofia: a Epistemologia.
Conhecida também por “teoria do conhecimento”, a Epistemologia é o campo da Filosofia que explora e estuda
problemas que têm relação com o conhecimento em geral. Por tratar dos problemas filosóficos relacionados à crença
e ao conhecimento, a Epistemologia estuda a origem, os métodos, a composição estrutural e a validade do conhecimento; relaciona-se, portanto, com a Lógica, uma vez que analisa a coerência lógica de uma teoria e a coesão dos
fatos que a fundamentam. Sua problematização também abrange a questão sobre a possibilidade do conhecimento
humano: seria possível ao homem, algum dia, alcançar de fato o conhecimento total e puro? Outros questionamentos
dizem respeito aos limites do conhecimento e, também, à sua origem.
39
GONSALES, Fernando. “Níquel Náusea”. In: Folha de S. Paulo (10 de maio de 2009).
A Epistemologia surge com a filosofia de Platão, quando o filósofo grego opôs “crença” (ou opinião) a “conhecimento”, separando-os por serem coisas distintas: a crença seria um ponto de vista subjetivo; o conhecimento, uma
crença verdadeira e justificada. Com isso, podemos notar que a Filosofia, a Ciência e, sobretudo, o senso comum em
determinados momentos promovem um caráter universal ao que é incerto, subjetivo, tornando-o dogmático. É nesse
meio em que a Epistemologia age, apontando as falhas do conhecimento admitido e, mostrando as incoerências e
contradições dos pensamentos, das ideias e das teorias, faz com que o conhecimento se policie.
As questões epistemológicas são, pode-se dizer, tão velhas quanto a própria Filosofia, apesar de que seu primeiro
estudo seja o encontrado nas obras de Platão. Foi, contudo, no período moderno (do século XVII em diante), com o
trabalho de René Descartes (1596-1650) e John Locke (1632-1704), em associação com o surgimento da ciência
moderna, que a Epistemologia ocupou um lugar central na Filosofia. Entre os questionamentos epistemológicos que
surgem dessa época há um em especial que passou por extensos debates. A questão relaciona-se com a razão, e busca
compreender se ela é “inata” ou “adquirida”.
De onde vieram os princípios racionais (identidade, não-contradição, terceiro excluído e razão suficiente)? De onde veio a capacidade para a intuição (razão intuitiva) e para o raciocínio (razão discursiva)? Nascemos com eles? Ou nos seriam dados
pela educação e pelo costume? Seriam algo próprio dos seres humanos, constituindo a natureza deles, ou seriam adquiridos
através da experiência? [Idem.]
Para tal questão, duas respostas surgiram: o “inatismo” (ou “racionalismo”) e o “empirismo”. O inatismo alega
que o ser humano racional traz, desde seu nascimento, não apenas os princípios racionais citados acima, mas também
algumas ideias verdadeiras (que, por acompanharem o homem desde seu nascimento, são ideias inatas). Já o empirismo alega que a razão, com seus princípios, com seus procedimentos e com suas ideias, é adquirida pelos homens
por meio do conhecimento empírico, isto é, por meio da experiência. Vejamos, pois, mais de perto cada uma dessas
respostas.
O inatismo teve como principais defensores os filósofos Platão (século IV a.e.c.) e Descartes (século XVII). Para
o pensador grego, a validade do inatismo é legitimada por sua teoria das ideias (ou “teoria da reminiscência”), que
afirma nascer o homem com a razão e as ideias verdadeiras, de modo que a Filosofia, enquanto atitude de conhecimento, nada mais faz do que relembrar o homem sobre essas ideias.
Conhecer, diz Platão, é recordar a verdade que já existe em nós; é despertar a razão para que ela se exerça por si mesma. Por
isso, Sócrates fazia perguntas, pois, através delas, as pessoas poderiam lembrar-se da verdade e do uso da razão. Se não nascêssemos com a razão e com a verdade, indaga Platão, como saberíamos que temos uma ideia verdadeira ao encontrá-la?
Como poderíamos distinguir o verdadeiro do falso, se não nascêssemos conhecendo essa diferença? [Idem.]
René Descartes, séculos depois, discute a teoria das ideias inatas, afirmando que tais ideias são inteiramente racionais e podem existir apenas porque o ser humano já nasce com elas. São exemplos a ideia do infinito e as ideias
matemáticas, posto que ninguém tem qualquer experiência do infinito, e a matemática, por poder trabalhar teoricamente com ideias tais como a de uma figura de mil lados (a qual jamais poderemos ter a percepção). Para Descartes,
então, essas ideias são “a assinatura do Criador” na mente das criaturas racionais, de modo que a razão é ferramenta
natural inata que permite ao homem conhecer a verdade.
Como as ideias inatas são colocadas em nosso espírito por Deus, serão sempre verdadeiras, isto é, sempre corresponderão integralmente às coisas a que se referem, e, graças a elas, podemos julgar quando uma ideia adventícia é verdadeira ou falsa e saber
que as ideias fictícias são sempre falsas (não correspondem a nada fora de nós). Ainda segundo Descartes, as ideias inatas são as
mais simples que possuímos (simples não quer dizer “fáceis”, e sim não-compostas de outras ideias). A mais famosa das ideias
inatas cartesianas é o “Penso, logo existo”. Por serem simples, as ideias inatas são conhecidas por intuição e são elas o ponto de
partida da dedução racional e da indução, que conhecem as ideias complexas ou compostas. A tese central dos inatistas é a seguinte: se não possuirmos em nosso espírito a razão e a verdade, nunca teremos como saber se um conhecimento é verdadeiro ou falso,
isto é, nunca saberemos se uma ideia corresponde ou não à realidade a que ela se refere. Não teremos um critério seguro para
avaliar nossos conhecimentos. [Idem.]
Por sua vez, o empirismo alega que a razão, a verdade e as ideias racionais são adquiridas pelos homens por meio
da experiência (conhecimento a posteriori). Antes dela (conhecimento a priori), nossa razão é como uma “folha em
branco”, na qual nada foi escrito; uma “tábua rasa” (nas palavras de John Locke, um dos mais importantes empiristas), na qual nada foi gravado. Assim, os conhecimentos humanos começam com a experiência proveniente dos sentidos, ou seja, com as sensações captadas por nossos órgãos dos sentidos, excitados pelos objetos exteriores que nos
possibilitam ver cores, sentir sabores e odores, ouvir sons etc. Tais sensações se reúnem para formar a percepção:
40
percebemos um único objeto que nos chegou através de várias e diferentes sensações; as percepções, então, se combinam. Se, por um lado, a experiência escreve e grava em nossa mente as ideias, a razão associa-as, combina-as ou
separa-as, constituindo todos os nossos pensamentos.
Leituras complementares
O que é a epistemologia? A resposta é: o ramo da filosofia que se ocupa do conhecimento humano, pelo que também é designada de “teoria do conhecimento”. Só que isto diz-nos quase nada. Por que temos necessidade de uma
teoria do conhecimento? E ela é uma teoria acerca de quê, e como é que a defendemos (ou contestamos)? Aliás, o que
implica dizer que a epistemologia é um ramo da filosofia? O que há de especial nas investigações filosóficas do
conhecimento? Em que diferem da discussão psicológica ou sociológica acerca do “conhecimento” ou da “cognição”?
Muitos filósofos nos dias de hoje negam que as questões filosóficas acerca do conhecimento tenham um carácter
especial. Defendem que a epistemologia precisa de ser “naturalizada”: quer dizer, aproximá-la de uma ou mais ciências, talvez da psicologia cognitiva. Outros filósofos defendem que a epistemologia está morta. Estas perspectivas são
dificilmente separáveis: a distinção entre a transformação radical e a abolição imediata não é nítida. Contudo, penso
que o naturalismo está enganado e que os obituários da epistemologia são prematuros.
Cinco problemas. Para perceber o que há de diferente numa determinada área teórica, a melhor forma de começar
é perguntar que problema (ou problemas) aborda. No que diz respeito à epistemologia, sugiro que se distinga, cinco
tipos de problemas [...]. São eles:
1. O problema analítico: O que é o conhecimento? (Ou se preferirmos, o que entendemos ou devemos entender
por “conhecimento”? Por exemplo, como se distingue (ou se deve distinguir) o conhecimento da simples crença ou
opinião? O que aqui se pretende, idealmente, é uma explicação precisa ou “análise” do “conceito” de conhecimento.
2. O problema da demarcação: Este divide-se em dois problemas: a) O problema “externo” pergunta: sabendo-se
de algum modo o que é o conhecimento, poderemos determinar à partida que coisas podemos razoavelmente esperar
conhecer? Ou como se refere amiúde, poderemos determinar o âmbito e os limites do conhecimento humano? Será
que há assuntos acerca dos quais podemos ter conhecimento, enquanto há outros acerca dos quais não podemos ter
mais do que opinião (ou fé)? Será que há uma quantidade significativa de formas de discurso que ficam simultaneamente fora do domínio do “factual” ou do que “tem sentido”? O objectivo é traçar uma fronteira que separe a província do conhecimento de outros domínios cognitivos (ou talvez o cognitivo do não cognitivo). b) O problema “interno”
pergunta se há fronteiras significativas no interior do domínio do conhecimento. Por exemplo, muitos filósofos têm
defendido que há uma distinção fundamental entre o conhecimento a posteriori ou “empírico” e o conhecimento a
priori ou “não empírico”. O conhecimento empírico depende (de uma forma ou de outra) da experiência ou observação, ao passo que o conhecimento a priori é independente da experiência, fornecendo a matemática o exemplo mais
claro. Contudo, outros filósofos negam que se possa fazer tal distinção.
3. O problema do método: Este relaciona-se com o modo como obtemos ou procuramos conhecimento. Sugiro
que distingamos três subproblemas. a) O problema da “unidade” coloca a questão seguinte: Há só uma forma para
adquirir conhecimento, ou há várias, dependendo do tipo de conhecimento em questão? Por exemplo, alguns filósofos
têm defendido que há diferenças fundamentais entre as ciências naturais e as sociais ou humanas. b) O problema do
desenvolvimento (progresso) coloca a questão seguinte: Podemos melhorar as nossas formas de investigação? No séc.
XVII este era um problema de importância capital para os filósofos que defendiam os avanços científicos contra o
que consideravam ser o dogmatismo estéril da escolástica (a versão semi-oficial das posições filosóficas e científicas
de Aristóteles ensinada nas universidade e “escolas”). c) Finalmente, o problema da “razão” ou da “racionalidade”. A
preocupação aqui é saber se há métodos de investigação, ou de fixação de crenças, que sejam claramente racionais e,
se há, quais são.
4. O problema do cepticismo: Será de facto possível obter algum conhecimento? Este problema é difícil porque
há argumentos poderosos, alguns bastante antigos, a favor da resposta negativa. Por exemplo, embora o conhecimento não possa assentar em pressupostos brutos, todos os argumentos têm de acabar por chegar ao fim. Parece que, em
última análise, as opiniões das pessoas assentam em indícios que elas não podem justificar e não podemos considerar
conhecimento genuíno. O problema que aqui se coloca, então, é o de conhecer os argumentos do cepticismo filosófico, a tese que defende a impossibilidade do conhecimento. Uma vez que há uma ligação forte entre conhecimento e
justificação, o problema do cepticismo está intimamente ligado ao problema da justificação.
5. O problema do valor: Os problemas esboçados são significativos somente se faz sentido possuir conhecimento.
Mas será faz, e se sim porquê? Supondo que sim, para que o queremos? Queremo-lo de qualquer forma, ou por causa
de determinados objectivos e em determinadas situações? O conhecimento é o único objectivo da investigação, ou há
outros com igual (ou maior) importância?
Obviamente que estes problemas não são independentes. O modo como lidamos com um impõe constrangimentos
ao modo como lidamos com os outros. Mas o modo como um dado filósofo ajuíza a sua importância relativa determinará o sentido que uma dada teoria do conhecimento necessita alcançar e como pode ser defendida. Isto é típico na
filosofia, porquanto se verificam profundos desacordos não apenas em relação à correcção das respostas a um conjunto determinado de perguntas mas também às próprias questões.
Muitos filósofos atribuem um estatuto privilegiado ao problema do cepticismo na teoria do conhecimento. Com
efeito, identificam praticamente o problema do conhecimento com este problema. The Problem of Knowledge, de
Ayer (1956), é um exemplo cabal disto.
41
Há muito a favor desta perspectiva. Há um consenso generalizado relativamente ao facto de a idade moderna da
filosofia começar com Descartes (1596-1650), e que o seu contributo fundamental foi ter induzido a filosofia a realizar uma viragem enfaticamente epistemológica.
Descartes escreveu durante um período de grande produção intelectual quando (entre outras coisas) a visão medieval do mundo, uma síntese de algum modo instável entre a filosofia aristotélica e a teologia cristã, começou a ser
crescentemente pressionada por novas ideias científicas emergentes. Insatisfeito com o ensino do seu tempo e sedento
de promoção da nova ciência, Descartes defende um corte radical com o passado. Pretende construir uma visão do
mundo e o nosso conhecimento dele a partir dos alicerces. Ao promover esta reconstrução, afirma que aceita como
princípios básicos apenas aqueles que, logicamente falando, não podem ser colocados em dúvida. Com efeito, utiliza
o argumento céptico como um filtro para eliminar todas as opiniões duvidosas: devemos aceitar apenas as proposições que resistam ao mais determinado assalto céptico. Por confiar no facto de ter encontrado tais proposições, Descartes não é realmente um céptico. Não obstante, a sua “dúvida metódica” coloca os problemas do cepticismo no
centro da reflexão.
Conjuntamente com estas considerações históricas, há razões teóricas fortíssimas a favor da posição que afirma
que os problemas cépticos são a força motriz por detrás das teorias filosóficas do conhecimento. Uma das formas
mais esclarecedoras para compreender a diferença entre as teorias tradicionais do conhecimento é considerá-las tentativas de descolagem de ideias concorrentes sobre os erros dos argumentos cépticos. [...]
Colocar as preocupações com o cepticismo no centro da epistemologia torna muito claro o que distingue a reflexão filosófica acerca do conhecimento. Tal reflexão responde a preocupações profundas sobre se de facto o conhecimento é possível. Isto não pode ser considerado uma matéria científica estrita na medida em que o cepticismo questiona todo o alegado conhecimento, incluindo o científico.
[...] A ameaça do cepticismo não foi nunca o verdadeiro motivo da reflexão filosófica sobre o conhecimento humano. Uma distinção útil aqui é a que se pode estabelecer entre o objectivo do filósofo e a sua tarefa: isto é, entre o
que ele espera alcançar e o modo como pensa que deve prosseguir (devo esta terminologia útil a Robert Fogelin). O
cepticismo tem sido o problema epistemológico dominante na idade moderna não porque “refutar o céptico” seja
sempre o objectivo da reflexão epistemológica mas porque eliminar a argumentação céptica é quase invariavelmente
uma das suas tarefas fundamentais. Por exemplo, se suspeitamos que certo tipo de afirmações são mais vulneráveis
aos ataques cépticos do que outras, explorar os limites do cepticismo oferecerá uma via para definir demarcações
significativas. Ou, dito de outro modo, se pudermos mostrar onde erram os argumentos cépticos, é de esperar aprender importantes lições sobre conhecimento e justificação. Não é necessário estar no espaço das conclusões cépticas
para nos interessarmos pelos argumentos cépticos.
[...] Embora a epistemologia moderna tenha mostrado uma tendência definitiva para seguir o paradigma cartesiano, colocando o cepticismo em primeiro lugar, a minha caracterização da epistemologia no que diz respeito à listagem dos problemas, deixa em aberto a possibilidade de desenvolver outras abordagens. Este aspecto da minha abordagem da epistemologia será importante quando discutirmos se o tema se esgotou.
Epistemologia e a “tradição ocidental”. Dos meus cinco problemas, o do valor é o menos discutido pelos filósofos contemporâneos. Mas todos os outros problemas dependem deste. Se o conhecimento não tivesse importância,
não perderíamos tempo a imaginar como o definir, como o obter, nem a traçar linhas à sua volta. Nem nos interessaria refutar o céptico. Se não víssemos valor no conhecimento, o cepticismo seria provavelmente ainda um puzzle mas
não um problema. Contudo, parece-me que o conhecimento tem importância (para a maioria de nós, pelo menos
algumas vezes); se não o conhecimento de acordo com alguns critérios muito estritos, pelo menos outros conceitos
epistemológicos, tais como justificação ou racionalidade. Porquê?
Uma resposta é que a preocupação com o conhecimento (ou com realidades afins) está de tal modo enraizada na
nossa tradição ocidental que não é opcional. Esta tradição, que nos seus aspectos filosóficos e científicos, tem as suas
origens na Grécia clássica, é globalmente e no seu sentido mais lato uma tradição racionalista e crítica. A ciência e a
filosofia começam quando as ideias acerca da origem e natureza do universo se separam do mito e da religião e são
tratadas como teorias que se podem discutir: isto é, comparadas com (e porventura superadas por) teorias concorrentes. Como observou Karl Popper, esta abordagem globalmente racionalista para compreender o mundo pode ser
considerada como um tipo de tradição de “segunda ordem”: o que conta não são crenças particulares — encaradas
como sagradas, ancestrais, e desse modo mais ou menos inquestionáveis — mas a prática do exame crítico das ideias
correntes para que se possa reter apenas o que fica depois da inspecção. Ter herdado esta tradição explica a nossa
tendência para contrastar conhecimento com preconceito ou com a (simples) tradição. A distinção é invejosa, o que é
uma outra forma de dizer que o conhecimento importa. E isto não é apenas um preconceito local. Uma vez desperto
para o facto de mesmo as nossas mais compartilhadas posições poderem ser desafiadas, não há retorno para um estádio pré-crítico, para uma perspectiva tradicionalista. É por isso que a preocupação com o conhecimento já não é
opcional.
A perspectiva racionalista pode aplicar-se a ela própria. Quando o é temos a epistemologia: um estudo de terceira
ordem, segundo uma tradição de reflexão metacrítica sobre os nossos objectivos e procedimentos epistemológicos.
Temos uma tradição de investigação centrada no tipo de questões que iniciamos.
Dada esta perspectiva de epistemologia, é fácil ver por que razão o cepticismo é especialmente difícil de ignorar.
O cepticismo é o gato com o rabo de fora do racionalismo ocidental: um ataque argumentativamente sofisticado à
própria argumentação racionalista. Representa o caso extremo da tradição da investigação crítica reflexivamente
aplicada. Desde os primórdios da filosofia ocidental, tem havido uma contra-tradição que defende que os limites da
razão são muito mais estreitos do que os epistemólogos optimistas gostam de pensar, que a própria ideia de razão é
uma armadilha e uma ilusão e que, mesmo que não o fosse, o conhecimento científico e filosófico acaba por não ser o
que se pensa que é. Se o cepticismo não pode ser refutado, a perspectiva racional destrói-se a si própria.
WILLIAMS, Michael. O que é a epistemologia?
42
φ
Haverá algum conhecimento no mundo que seja tão certo que nenhum homem razoável possa dele duvidar? Esta
pergunta, que à primeira vista pode não parecer difícil, é na realidade uma das mais difíceis que se pode fazer. Quando nos dermos conta dos obstáculos que se levantam a uma resposta fácil e confiante, estaremos já bem lançados no
estudo da filosofia — pois a filosofia é apenas a tentativa de responder a questões últimas deste género, não de modo
descuidado e dogmático, como fazemos na vida comum e até nas ciências, mas criticamente, depois de explorar tudo
o que gera perplexidade nessas perguntas, e depois de tomar consciência de toda a vagueza e confusão que subjaz às
nossas ideias comuns.
Na vida quotidiana pressupomos como certas muitas coisas que, num escrutínio mais atento, se revelam tão cheias de aparentes contradições que só uma grande quantidade de pensamento nos permite saber em que podemos realmente acreditar. Na procura da certeza, é natural começar pelas nossas experiências presentes e, num certo sentido,
sem dúvida que delas será derivado conhecimento. Mas qualquer afirmação sobre o que as nossas experiências imediatas nos fazem saber está muito provavelmente errada. Parece-me que estou agora sentado numa cadeira, a uma
secretária de uma certa forma, na qual vejo folhas de papel escritas ou impressas. Mas ao virar a cabeça vejo para lá
da janela edifícios e nuvens e o Sol. Acredito que o Sol está a cerca de cento e cinquenta milhões de quilómetros de
distância da Terra; que é um globo quente muito maior que a Terra; que, devido à rotação da Terra, nasce todas as
manhãs e continuará no futuro a fazê-lo por um período indeterminado de tempo. Acredito que se qualquer outra
pessoa normal entrar na minha sala, verá as mesmas cadeiras e secretárias e livros e papéis que eu vejo, e que a secretária que vejo é a mesma que a secretária cuja pressão sinto contra o meu braço. Tudo isto parece tão evidente que
nem parece valer a pena afirmá-lo, excepto para responder a um homem que duvide que eu saiba alguma coisa. Contudo, de tudo isto se pode razoavelmente duvidar e tudo exige muita discussão cuidadosa antes de podermos ter a
certeza de que o afirmámos de uma forma que seja inteiramente verdadeira.
Para tornar evidentes as nossas dificuldades, concentremos a atenção na mesa. Para a visão, é oblonga, castanha e
brilhante, para o tacto, é lisa e fria e dura; quando lhe bato, emite um som de madeira. Qualquer outra pessoa que veja
e sinta e oiça a mesa concordará com esta descrição, de modo que poderá parecer que nenhuma dificuldade se irá
levantar; mas assim que tentamos ser mais precisos começam os problemas. Apesar de eu acreditar que a mesa é
“realmente” toda da mesma cor, as partes que reflectem a luz parecem muito mais brilhantes que as outras, e algumas
partes parecem brancas por causa da luz reflectida. Sei que, se me deslocar, as partes que reflectem a luz serão diferentes, de modo que a distribuição manifesta de cores na mesa irá mudar. Segue-se que se várias pessoas estão a olhar
para a mesa no mesmo momento, nenhuma vê exactamente a mesma distribuição de cores, pois nenhuma pode vê-la
exactamente do mesmo ponto de vista, e qualquer mudança de ponto de vista provoca alguma mudança no modo
como a luz é reflectida.
Para a maior parte dos efeitos práticos, estas diferenças não são importantes, mas para o pintor são de máxima
importância: o pintor tem de desaprender o hábito de pensar que as coisas parecem ter a cor que o senso comum diz
que “realmente” têm, e tem de adquirir o hábito de ver as coisas como parecem. Temos já aqui o princípio de uma das
distinções que provocam mais problemas em filosofia — a distinção entre “aparência” e “realidade”, entre o que
parece que as coisas são e o que são. O pintor quer saber o que as coisas parecem, o homem prático e o filósofo querem saber o que são; mas o desejo do filósofo de saber isto é mais forte que o do homem prático, e é mais importunado pelo conhecimento das dificuldades em responder à questão.
Regressemos à mesa. É evidente pelo que descobrimos que não há qualquer cor que pareça proeminentemente a
cor da mesa, ou mesmo de uma qualquer parte particular da mesa — parece ter cores diferentes de diferentes pontos
de vista, e não há qualquer razão para considerar que algumas são mais realmente a sua cor do que outras. E sabemos
que mesmo de um dado ponto de vista a cor parecerá diferente à luz artificial, ou a um daltónico, ou a um homem
com óculos azuis, sendo que às escuras não haverá qualquer cor, apesar de ao tacto e ao ouvido a mesa se manter
inalterada. Esta cor não é algo que seja inerente à mesa, sendo antes algo que depende da mesa e do espectador e do
modo como a luz incide sobre a mesa. Quando, na vida comum, falamos de a cor da mesa, queremos apenas falar do
género de cor que parecerá ter a um espectador normal de um ponto de vista comum sob condições de luz habituais.
Mas as outras cores que a mesa parece ter noutras condições têm igual direito de serem consideradas reais; e portanto,
para evitar favoritismo, somos obrigados a negar que, em si, a mesa tenha uma cor particular qualquer.
O mesmo se aplica à textura. A olho nu consegue-se ver a textura mas, à parte isso, a mesa parece lisa e uniforme.
Se víssemos a mesa por um microscópio, veríamos rugosidade e montes e vales, e todo o tipo de diferenças, imperceptíveis a olho nu. Qual destas é a mesa “real”? Temos a tentação natural de dizer que o que vemos pelo microscópio é mais real, mas isso por sua vez mudaria com um microscópio ainda mais poderoso. Mas se não podemos confiar
no que vemos a olho nu, por que razão devemos confiar no que vemos pelo microscópio? Uma vez mais, pois, nos
abandona a confiança que tínhamos nos nossos sentidos.
A forma da mesa não é melhor. Todos temos o hábito de fazer juízos sobre a forma “real” das coisas, e fazemos
isto tão irreflectidamente que acabamos por pensar que vemos efectivamente as formas reais. Mas, de facto, como
temos de aprender se tentarmos desenhar, uma dada coisa parece ter uma forma diferente de diferentes pontos de
vista. Se a nossa mesa é “realmente” rectangular, parecerá, de quase todos os pontos de vista, como se tivesse dois
ângulos agudos e dois ângulos obtusos. Se os lados opostos forem paralelos, parecerá que convergem num ponto
afastado do espectador; se forem de comprimento igual, o lado mais próximo parecerá mais comprido. Não se repara
normalmente em todas estas coisas ao olhar para uma mesa, pois a experiência ensinou-nos a construir a forma “real”
a partir da forma aparente, e a forma “real” é o que nos interessa como homens práticos. Mas a forma “real” não é o
que vemos; é algo que se infere do que vemos. E o que vemos muda constantemente de forma à medida que nos
deslocamos na sala; de modo que também neste caso os sentidos não parecem dar-nos a verdade sobre a mesa em si,
mas apenas sobre a aparência da mesa.
43
Levantam-se dificuldades análogas quando consideramos o sentido do tacto. É verdade que a mesa nos dá sempre
uma sensação de dureza, e sentimos que resiste à pressão. Mas a sensação que obtemos depende da força com que
pressionamos a mesa e também da parte do corpo com que a pressionamos; assim, não se pode supor que as várias
sensações que resultam de diferentes pressões ou de diferentes partes do corpo revelam directamente qualquer propriedade definida da mesa, sendo no máximo sinais de uma propriedade que talvez cause todas as sensações, mas que
não é efectivamente manifesta em qualquer delas. E o mesmo acontece ainda mais obviamente aos sons que se podem
extrair batendo na mesa.
Assim, torna-se evidente que a mesa real, se existe, não é o mesmo de que temos experiência imediata pela visão
ou pelo tacto ou pela audição. A mesa real, se existe, não é de modo algum imediatamente conhecida por nós, tendo
antes de ser uma inferência do que é imediatamente conhecido. Assim, levantam-se desde já duas questões muito
difíceis; nomeadamente:
1. Haverá de todo em todo uma mesa real?
2. Se sim, que tipo de objecto poderá ser?
Ao considerar estas questões será vantajoso ter alguns termos simples cujo significado é definido e claro. Seja
dado o nome “dados dos sentidos” às coisas que são imediatamente conhecidas pela sensação: coisas como cores,
sons, cheiros, dureza, rugosidade, e assim por diante. Daremos o nome “sensação” à experiência de estar imediatamente ciente destas coisas. Assim, sempre que vemos uma cor, temos uma sensação da cor, mas a cor em si é um
dado dos sentidos e não uma sensação. A cor é aquilo do qual estamos imediatamente cientes, e o próprio estar ciente
é a sensação. É claro que para sabermos seja o que for sobre a mesa tem de ser por meio dos dados dos sentidos —
cor castanha, forma oblonga, lisura, etc. — que associamos à mesa; mas, pelas razões dadas, não podemos dizer que a
mesa é os dados dos sentidos, nem mesmo que os dados dos sentidos são directamente propriedades da mesa. Assim,
levanta-se um problema quanto à relação entre os dados dos sentidos e a mesa real, supondo que há tal coisa.
À mesa real, se existe, chamaremos “objecto físico”. Assim, temos de considerar a relação entre os dados dos
sentidos e os objectos. À colecção de todos os objectos físicos chama-se “matéria”. Assim, as duas questões anteriores podem ser reformuladas do seguinte modo:
1. Será que há matéria?
2. Se há, qual é a sua natureza?
O filósofo que primeiro deu destaque marcado às razões para considerar que os objectos imediatos dos nossos
sentidos não existem independentemente de nós foi o bispo Berkeley (1685-1753). O seu Três Diálogos entre Hylas e
Philonous, em Oposição aos Cépticos e Ateus, dedica-se a provar que a matéria é coisa que não existe de modo algum, e que o mundo não é mais do que mentes e as suas ideias. Hylas acreditou até essa altura na matéria, mas não
está à altura de Philonous, que o conduz implacavelmente a contradições e paradoxos, tornando a sua própria negação
da matéria, no final, como se fosse quase senso comum. Os argumentos usados são muito desiguais em valor: alguns
são importantes e sólidos, outros são confusos ou dúbios. Mas Berkeley tem o mérito de ter mostrado que a existência
da matéria pode ser negada sem absurdo e que se há algumas coisas que existem independentemente de nós, não
podem ser os objectos imediatos das nossas sensações.
Há duas questões diferentes envolvidas quando perguntamos se a matéria existe, e é importante que permaneçam
claras. Por “matéria” queremos habitualmente dizer algo que se opõe a “mente”, algo que pensamos que ocupa espaço e que é radicalmente incapaz de qualquer tipo de pensamento ou consciência. É principalmente neste sentido que
Berkeley nega a matéria; ou seja, não nega que os dados dos sentidos que comumente tomamos como sinais da existência da mesa são realmente sinais da existência de algo independente de nós, mas nega que este algo seja não mental, nega que não seja mente nem ideias concebidas numa mente. Admite que tem de haver algo que continua a existir
quando saímos da sala ou fechamos os olhos, e que o que chamamos “ver a mesa” nos dá realmente razão para acreditar em algo que persiste mesmo quando não a estamos a ver. Mas pensa que este algo não pode ter uma natureza
radicalmente diferente do que vemos, e não pode ser completamente independente do ver, apesar de ter de ser independente do nosso ver. Berkeley é assim levado a considerar que a mesa “real” é uma ideia na mente de Deus. Tal
ideia tem a permanência e independência de nós que se exige, sem ser — como a matéria seria — algo relativamente
incognoscível, no sentido de poder apenas ser inferida, não podendo nós estar cientes dela directa e imediatamente.
Depois de Berkeley, outros filósofos sustentaram também que, apesar de a existência da mesa não depender de
ser vista por mim, depende de ser vista (ou apreendida de outro modo na sensação) por alguma mente — não necessariamente a mente de Deus, mas mais frequentemente a totalidade da mente colectiva do universo. Sustentam isto
principalmente porque, como Berkeley, pensam que nada de real pode existir — ou em qualquer caso nada que se
saiba ser real — excepto mentes e os seus pensamentos e sentires. Podemos formular o argumento com que sustentam
a sua perspectiva de certo modo como se segue: “Seja o que for que pode ser pensado é uma ideia na mente da pessoa
que a pensa; logo, nada pode ser pensado excepto ideias em mentes; logo, qualquer outra coisa é inconcebível, e o
que é inconcebível não pode existir”.
Tal argumento, na minha opinião, é falacioso; e claro que quem o avança não o apresenta tão concisamente nem
tão grosseiramente. Mas, seja ou não válido, o argumento tem sido muitíssimo avançado numa ou noutra forma; e
muitíssimos filósofos, talvez a maioria, têm sustentado que nada de real existe excepto mentes e as suas ideias. Chama-se “idealistas” a tais filósofos. Quando se trata de explicar a matéria ou dizem, como Berkeley, que a matéria
nada é senão uma colecção de ideias, ou dizem, como Leibniz (1646-1716), que o que parece matéria é na realidade
uma colecção de mentes mais ou menos rudimentares.
Mas estes filósofos, apesar de negarem a matéria como algo que se opõe à mente, admitem contudo a matéria.
Recorde-se que fizemos duas perguntas; nomeadamente, 1) Haverá de todo em todo uma mesa real? 2) Se sim, que
tipo de objecto poderá ser? Ora, tanto Berkeley como Leibniz admitem que existe uma mesa real, mas Berkeley
afirma que é certas ideias na mente de Deus, e Leibniz diz que é uma colónia de almas. Assim, ambos respondem
afirmativamente à nossa primeira pergunta, divergindo apenas das perspectivas dos mortais comuns na sua resposta à
44
nossa segunda pergunta. De facto, quase todos os filósofos parecem concordar que há uma mesa real: quase todos
concordam que, por mais que os nossos dados dos sentidos — cor, forma, lisura, etc. — possam depender de nós, a
sua ocorrência é contudo um sinal de algo que existe independentemente de nós, algo que difere, talvez, completamente dos nossos dados dos sentidos, e que contudo se deve considerar que causa tais dados dos sentidos sempre que
estamos numa relação adequada com a mesa real.
Ora, este aspecto sobre o qual os filósofos concordam — a perspectiva de que há uma mesa real, seja qual for a
sua natureza — é de importância vital, e valerá a pena considerar que razões há para aceitar esta perspectiva antes de
avançarmos para a questão seguinte quanto à natureza da mesa real. O nosso próximo capítulo, consequentemente,
ocupar-se-á das razões para supor que há de todo em todo uma mesa real.
Antes de avançarmos será bom considerar por momentos o que descobrimos até agora. Tornou-se manifesto que,
se tomamos qualquer objecto comum do tipo que é supostamente conhecido pelos sentidos, o que os sentidos nos
dizem imediatamente não é a verdade sobre o objecto tal como este é além de nós, mas apenas a verdade sobre certos
dados dos sentidos que, tanto quanto podemos ver, dependem das relações entre nós e o objecto. Assim, o que vemos
e sentimos directamente é apenas “aparência”, que acreditamos ser um sinal de uma “realidade” que está por detrás.
Mas se a realidade não é o que aparece, teremos algum meio de saber se há alguma realidade de todo em todo? E em
caso afirmativo, teremos algum meio de descobrir como é ela?
Estas perguntas são desconcertantes, e é difícil saber se mesmo as hipóteses mais estranhas não poderão ser verdadeiras. Assim, a nossa mesa familiar, que até agora não deu origem senão aos mais insignificantes pensamentos em
nós, tornou-se um problema cheio de possibilidades surpreendentes. O que sabemos dela é que não é o que parece.
Além deste resultado modesto, até agora, temos a mais completa liberdade de conjectura. Leibniz diz-nos que é uma
comunidade de almas; Berkeley diz-nos que é uma ideia na mente de Deus; a ciência sóbria, dificilmente menos
maravilhosa, diz-nos que é uma vasta colecção de cargas eléctricas em movimento violento.
Entre estas possibilidades surpreendentes, a dúvida sugere que talvez não haja qualquer mesa. A filosofia, se não
pode responder a tantas perguntas como gostaríamos, tem pelo menos o poder de fazer perguntas que aumentam o
interesse do mundo, e mostram a estranheza e a maravilha repousando imediatamente sob a superfície mesmo nas
coisas mais comuns da vida quotidiana.
RUSSELL, Bertrand. Aparência e realidade.
φ
Epistemologia: tentar dizer a verdade acerca da verdade. A investigação reflexiva última acerca da investigação
ocorre no ramo da filosofia conhecido como epistemologia, a teoria do conhecimento. Também aqui as controvérsias
existentes nas margens criaram um efeito nocivo, uma distorção que muitas vezes conduziu a interpretações erradas.
Ao concordar que a verdade é um conceito muito importante, os epistemólogos tentaram dizer exactamente o que é a
verdade — sem se despistarem. Perceber o que é verdade acerca da verdade, contudo, acabou por se revelar uma
tarefa difícil, tecnicamente difícil, uma tarefa na qual as definições e as teorias que parecem à primeira vista inocentes
conduzem a complicações que rapidamente fazem o teórico enredar-se em doutrinas duvidosas. A nossa estimada e
conhecida amiga, a verdade, tende a transformar-se na Verdade — com V maiúsculo —, um conceito inflacionado de
verdade que de facto não pode ser defendido.
Eis um dos caminhos que conduzem à dificuldade: suponhamos que o conhecimento não é nada senão acreditar
justificadamente em proposições verdadeiras. Suponhamos, além disso, que as proposições verdadeiras, ao contrário
das falsas, exprimem factos. O que são os factos? Quantos factos existem? (Tom, Dick e Harry estão sentados numa
sala. Eis um facto. Mas para além de Tom, Dick e Harry, da sala onde estão sentados e do que lhes serve de assento,
parece que temos um sem-fim de outros factos: Dick não está de pé, não existe qualquer cavalo que esteja a ser montado por Tom, e assim por diante, ad infinitum. Precisaremos realmente de admitir uma infinidade de outros factos
juntamente com o pouquíssimo equipamento deste pequeno mundo?) Já existiam factos antes de existirem aqueles
que os procuram, ou são antes os factos como as frases verdadeiras (inglesas, francesas, latinas, etc.), cuja existência
teve de aguardar que se criassem as línguas humanas? São os factos independentes das mentes daqueles que acreditam nas proposições que os exprimem? Correspondem as verdades aos factos? A que correspondem então as verdades da matemática, se é que correspondem a algo? As categorias começam a multiplicar-se, não emergindo nenhuma
teoria unificada, óbvia e consensual sobre a verdade. Os cépticos, vendo as armadilhas que parecem rodear qualquer
versão da verdade, absoluta ou transcendental, argumentam a favor de versões mais moderadas, mas os seus adversários contra-argumentam, mostrando as imperfeições das tentativas rivais de chegar a uma teoria aceitável. Reina a
controvérsia sem fim.
Esta investigação modesta, mas por vezes brilhante, do próprio significado da palavra «verdade» tem tido algumas consequências perniciosas. Algumas pessoas pensaram que os argumentos filosóficos que mostram a situação
desesperada das doutrinas inflacionadas da verdade mostraram que, na realidade, a própria verdade não era algo
digno de apreço ou sequer passível de ser alcançado. «Desistam!», parecem essas pessoas dizer. A verdade é um ideal
inalcançável e insensato. Aqueles que buscam uma doutrina da verdade aceitável e defensável parecem estar a agarrar-se a um credo ultrapassado, dando crédito a uma religião que não conseguem fundamentar pelos métodos da
própria ciência. A epistemologia começa a parecer-se com um jogo de idiotas — mas apenas porque os seus observadores esquecem tudo aquilo que ambos os lados aceitam acerca da verdade. Os efeitos desta visão distorcida podem
ser perturbadores.
Quando era um jovem assistente de filosofia, recebi uma vez uma visita de um colega do Departamento de Literatura Comparada, um elegante e eminente teórico literário que precisava de ajuda. Senti-me lisonjeado por ele me ter
procurado e fiz o melhor que pude para corresponder ao pedido, mas fiquei, estranhamente, perplexo com o sentido
geral das suas perguntas acerca de vários tópicos filosóficos. Durante muito tempo não chegámos a lado nenhum, até
45
que ele conseguiu tornar claro o que desejava. Ele queria «uma epistemologia», afirmou. Uma epistemologia. Todos
os teóricos literários dignos desse nome tinham, ao que parece, de exibir uma epistemologia naquela temporada, sem
a qual ele se sentia nu, de maneira que tinha vindo ter comigo em busca de uma epistemologia que pudesse usar —
ele tinha a certeza que isso estava na moda e queria por isso o dernier cri em epistemologia. Não lhe interessava que
fosse sólida, defensável, nem (como se poderia muito bem dizer) verdadeira; só tinha de ser nova e diferente e com
estilo. Usa os acessórios certos, meu caro amigo, senão ninguém vai reparar em ti na festa.
Nesse momento percebi que existia entre nós um abismo que até àquele momento não tinha claramente compreendido. Primeiro pensei tratar-se unicamente do abismo entre a seriedade e a frivolidade. Mas a minha vaga inicial de
orgulho na minha própria integridade era, de facto, uma reacção ingénua. O meu sentimento de ultraje, o meu sentimento de que tinha desperdiçado o meu tempo com o bizarro projecto deste homem, era, à sua própria maneira, tão
pouco sofisticado como a reacção de alguém que, ao assistir pela primeira vez a uma peça de teatro, irrompe pelo
palco para proteger a heroína do vilão. «Não estás a ver?», perguntamos, incrédulos. «É um faz-de-conta. É arte. Não
é suposto ser tomado literalmente!» Neste contexto, a demanda deste homem não era afinal tão vergonhosa quanto
isso. Eu não teria ficado ofendido se um colega do Departamento de Teatro me tivesse pedido alguns metros de livros
para colocar nas prateleiras do cenário para a sua produção da peça Jumpers, de Tom Stoppard, pois não? Que mal
haveria em abastecer este amigo com uma série de vistosas doutrinas epistemológicas escandalosas, com as quais ele
poderia excitar ou confundir os seus colegas?
O que seria errado, uma vez que ele não se dava conta do abismo, não reconhecendo sequer a sua existência, seria
o facto de a minha concordância com a sua pândega consumista contribuir para o aviltamento de um bem precioso e
para a erosão de uma distinção valiosa. Muitas pessoas, incluindo quer os espectadores quer os participantes, não se
dão conta deste abismo, ou negam activamente a sua existência; e é aí que está o problema. O que é triste nisto tudo é
que em alguns círculos intelectuais, habitados por alguns dos nossos pensadores mais avançados nas artes e nas humanidades, esta atitude passa por ser uma sofisticada apreciação da futilidade da demonstração e da relatividade de
todas as afirmações de conhecimento. Na verdade, esta opinião, longe de ser sofisticada, é o cúmulo da ingenuidade
inconsciente, só possível graças à ignorância grosseira dos métodos já demonstrados de procura científica da verdade,
assim como do seu poder. Como muitos outros ingénuos, estes pensadores, ao refletirem na manifesta insuficiência
dos seus métodos de procura da verdade para atingir resultados estáveis e valiosos, generalizam inocentemente a
partir dos seus próprios casos, concluindo que mais ninguém sabe como descobrir a verdade. [...]
A verdade pode magoar. Toda a gente deseja a verdade. Quando o leitor se interroga sobre se o seu vizinho o enganou, ou se há peixes nesta área do lago, ou para que lado deve caminhar para chegar a casa, está interessado na
verdade. Mas então, se a verdade é tão maravilhosa, por que motivo existe tanto antagonismo em relação à ciência?
Toda a gente aprecia a verdade; mas nem toda a gente aprecia os instrumentos científicos de procura da verdade.
Ao que parece, algumas pessoas prefeririam outros métodos mais tradicionais de alcançar a verdade: a astrologia,
a adivinhação, os profetas e gurus e xamãs, o transe e a consulta de vários textos sagrados. Nestes casos, o veredicto
da ciência é tão familiar que quase nem preciso repeti-lo: enquanto diversões ou exercícios de elasticidade mental,
todas estas actividades têm os seus méritos, mas, enquanto métodos para procurar a verdade, nenhum deles pode
competir com a ciência — um facto em geral reconhecido tacitamente pelos que defendem a sua prática alternativa
favorita através do que afirmam ser a base científica (que outra coisa havia de ser?) dos seus poderes. Nunca encontramos um crente na comunicação com o além a procurar o apoio de uma associação de astrólogos ou de um Colégio
dos Cardiais; pelo contrário: exibem-se avidamente todos os farrapos de possíveis indícios estatísticos e qualquer
físico ou matemático extraviado que possa oferecer um testemunho favorável.
Mas então por que motivo há tanto pavor, se mesmo os que procuram passar palavra acerca de alternativas apelam regularmente para a ciência? A resposta é amplamente conhecida: a verdade pode magoar. Sem dúvida que pode.
Isto não é uma ilusão, mas é por vezes negado ou ignorado por cientistas e outras pessoas que fingem acreditar que a
verdade acima de tudo é o bem supremo. Posso facilmente descrever circunstâncias nas quais eu próprio mentiria ou
omitiria a verdade para evitar o sofrimento humano. A uma senhora idosa, no fim dos seus dias, nada resta senão as
histórias dos feitos heróicos do seu filho — vai o leitor dizer-lhe a verdade quando o seu filho for preso, condenado
por um crime terrível e humilhado? Não será para ela melhor deixar este mundo em ignorante serenidade? Claro que
é, afirmo eu. Mas note-se que mesmo aqui temos de compreender estes casos como excepções à regra. Não poderíamos oferecer a esta mulher o conforto das nossas mentiras se mentir fosse a regra geral; ela tem de acreditar em nós
quando falamos com ela.
É um facto que as pessoas não querem muitas vezes saber a verdade. E é um facto mais inquietante que as pessoas não queiram muitas vezes que os outros saibam a verdade. Mas, tentar transformar estes factos de forma a que
apoiem a ideia estúpida de que a própria fé na verdade é uma atitude humana relativa a certas culturas, situada ou em
qualquer caso opcional, é confundir tudo. O pai do acusado que ouve em tribunal os testemunhos contra o seu filho, a
mulher que se pergunta se o marido a anda a enganar — eles podem muito bem não querer saber a verdade, e podem
ter razão em não querer saber a verdade, mas o facto é que acreditam na verdade; isso é claro. Eles sabem que a
verdade está aí, para ser evitada ou abraçada, e sabem que a verdade é importante. É por isso que eles podem muito
bem não querer saber a verdade. Porque a verdade pode magoar. Podem conseguir enganar-se a si mesmos, pensando
que a atitude que têm nestas ocasiões perante a verdade reflecte um defeito da própria verdade, assim como da própria procura e descoberta da verdade — mas se isto acontecer é puro auto-engano. O máximo a que podem aspirar
agarrar-se é à ideia de que podem existir boas razões, as melhores razões — no tribunal da verdade, note-se — para,
por vezes, suprimir ou ignorar a verdade. [...]
DENNETT, Daniel. Fé na Verdade.
46
Cogito, logo aprendo
1. (UFMG 2002) Leia estes trechos:
TRECHO 1. “Considero paradigmas as realizações científicas universalmente reconhecidas que, durante algum
tempo, forneceram problemas e soluções modelares para uma comunidade de praticantes de uma ciência. [...] Homens cuja pesquisa está baseada em paradigmas compartilhados estão comprometidos com as mesmas regras e padrões para a prática científica. Esse comprometimento e o consenso aparente que produz são pré-requisitos para a
ciência normal, isto é, para a gênese e continuação de uma tradição de pesquisa determinada.” [KUHN, Thomas. A
estrutura das revoluções científicas.]
TRECHO 2. “De tempos em tempos, teorias aparentemente sólidas e brilhantes como diamantes são destruídas
por outras teorias emergentes, que mostram as falhas da anterior e apontam novos caminhos. A situação é absolutamente natural. Os problemas começam quando uma velha lei da ciência é derrubada e não se sabe o que colocar no
lugar.” [Pesquisa FAPESP, nov. 2000. p. 38.]
A partir das ideias contidas nesses trechos,
a. COMENTE a noção de paradigma neles presente.
b. DÊ dois exemplos dessa mesma noção retirados da história da ciência.
2. (UFMG 2003) Leia esta afirmação: “... as causas e os efeitos não podem ser descobertos pela razão, mas sim
pela experiência...” [HUME, David. Investigação sobre o entendimento humano.]
REDIJA um texto justificando, do ponto de vista do autor, essa afirmação.
3. (UFMG 2003) Leia estes trechos:
TRECHO 1. “O Projeto Genoma [...] é a resposta final ao mandamento: Conhece-te a ti mesmo.” [...] “Se eu tivesse um computador adequado e a sequência completa do DNA, poderia calcular o organismo de um ser vivo, isto é,
sua anatomia, sua fisiologia e seu comportamento.” [GILBERT, W. Conhecimento proibido.]
TRECHO 2. “Num certo sentido, é necessário, como disse o oráculo grego, conhecer-se a si mesmo. Essa é a
primeira meta do conhecimento. Mas reconhecer que a alma do homem é incognoscível é o objetivo supremo da
sabedoria. O mistério final somos nós mesmos. Quando tivermos conseguido pesar o sol na balança e medido os
degraus da lua e desenhado o mapa dos sete céus, estrela por estrela, ainda restaremos nós. Quem poderá calcular a
órbita da própria alma?” [WILDE, Oscar. De profundis.]
IDENTIFIQUE e ANALISE as duas posições
4. (UFMG 2005) CLASSIFIQUE os argumentos que se seguem em indutivos (I) ou dedutivos (D):
( ) Pedro, Joana, Francisco e Andréia têm ótimas notas em Matemática e são bons jogadores de xadrez. Fernando
tem ótimas notas em Matemática. Fernando é bom jogador de xadrez.
( ) Alunos brilhantes nunca estudam aos domingos. Maria sempre estuda aos domingos. Maria não é uma aluna
brilhante.
( ) Se o determinismo é uma doutrina verdadeira, a vontade humana não é livre. O determinismo é uma doutrina
verdadeira. A vontade humana não é livre.
( ) Insisto em dizer que a infração de que sou acusado se deve a um engano do guarda de trânsito, já que dirijo há
mais de trinta anos e nunca fui multado.
INDIQUE uma característica própria aos
a) argumentos indutivos.
b) argumentos dedutivos.
5. (UFMG 2007) Leia este trecho: “Podemos, por conseguinte, dividir todas as percepções do espírito em duas
classes ou espécies, que se distinguem por seus diferentes graus de força e de vivacidade. As menos fortes e menos
vivas são geralmente denominadas pensamentos ou ideias. A outra espécie não possui um nome em nosso idioma e
na maioria dos outros, porque, suponho, somente com fins filosóficos era necessário compreendê-las sob um termo
ou nomenclatura geral. Deixe-nos, portanto, usar um pouco de liberdade de denominá-las impressões, empregando
essa palavra num sentido de algum modo diferente do usual. Pelo termo impressão, entendo, pois, todas as nossas
percepções mais vivas, quando ouvimos, vemos, sentimos, amamos, odiamos, desejamos ou queremos. [...] Todas as
nossas ideias ou percepções mais fracas são cópias de nossas impressões ou percepções mais vivas.” [HUME, David.
Investigação acerca do entendimento humano.]
Com base na leitura desses trechos e considerando outras informações presentes na obra citada, EXPLIQUE, segundo Hume, a origem da ideia de Deus.
6. (UFMG 2007) Leia estes trechos:
TRECHO 1. “PEIXES: (de 20/2 a 20/3) Regente: Netuno. Dia positivo para as atividades mentais, estudos, meditação ou qualquer tipo de leitura. Sua capacidade de raciocinar e assimilar conhecimentos está num ótimo momento.
Cuidado, entretanto, com reações emocionais irrefletidas e impulsivas, que podem desconcentrá-lo.” [O GLOBO, Rio de
Janeiro, 16 maio 2006. Segundo Caderno.]
TRECHO 2. “Tratava-se de traçar uma linha (da melhor maneira possível) entre as afirmações, ou sistemas de afirmações, das ciências empíricas e todas as outras afirmações, de caráter religioso, metafísico ou simplesmente
pseudocientífico. Anos mais tarde [...] chamei este meu primeiro problema de ‘problema da demarcação’. O critério
da ‘refutabilidade’ [ou falseabilidade] é a solução para o problema da demarcação, pois afirma que, para serem classi-
47
ficadas como científicas, as assertivas ou sistemas de assertivas devem ser capazes de entrar em conflito com observações possíveis ou concebíveis.” [POPPER, Karl. Conjecturas e refutações.]
Com base na leitura desses dois trechos e considerando outros conhecimentos sobre o assunto, EXPLIQUE por
que, segundo Karl Popper, a astrologia não pode ser considerada uma ciência.
7. (UFMG 2006) Leia estes trechos:
TRECHO 1. “Quando Galileu deixou suas esferas rolar sobre a superfície oblíqua com um peso por ele mesmo
escolhido, ou quando Torricelli deixou o ar carregar um peso de antemão pensado como igual ao de uma coluna de
água conhecida por ele [...]: isso foi uma revelação para todos os pesquisadores da natureza. Deram-se conta que a
razão só compreende o que ela mesma produz segundo seu projeto, que ela teria que ir à frente com princípios dos
seus juízos segundo leis constantes e obrigar a natureza a responder às suas perguntas, mas sem se deixar conduzir
por ela como se estivesse presa a um laço; do contrário, observações feitas ao acaso, sem um plano previamente
projetado, não se interconectariam numa lei necessária, coisa que a razão todavia procura e necessita. A razão tem
que ir à natureza [...] não porém na qualidade de um aluno que se deixa ditar tudo o que o professor quer, mas sim na
de um juiz nomeado que obriga as testemunhas a responder às perguntas que lhes propõe. [KANT, Immanuel. Crítica da
razão pura.]
TRECHO 2. “... fazemos ciência com fatos assim como construímos uma casa com pedras, mas uma acumulação
de fatos não é ciência; assim como não é uma casa um monte de pedras.” [POINCARÉ, H. A ciência e a hipótese.]
IDENTIFIQUE e CARACTERIZE, no que diz respeito às origens do conhecimento científico, a posição filosófica expressa nesses trechos.
8. “É decerto possível, aos cidadãos, partilhar crianças, esposas e propriedades, como Platão sugere em A República. Porque nesse trabalho Platão propõe que as crianças, as esposas e a propriedade devam ser de todos em comum. Indagamos, portanto: é melhor agir como agimos agora ou devemos adotar a prática recomendada em A República? A proposta de que as esposas devam ser possuídas em comum apresenta muitas dificuldades, entre as quais as
três mais importantes são: a) Sócrates não deixa claro nenhum motivo pelo qual esse costume deva ser parte do sistema social; b) quando vista como meio para alcançar um fim (para o qual, diz-se no diálogo, o Estado existe), a
proposta é inviável; c) em nenhum lugar é explicada a maneira como a proposta pode ser posta em prática.” [ARISTÓTELES.
Política.]
a) Explique o que é um argumento.
b) O trecho sublinhado é um argumento. Identifique sua(s) premissa(s) e sua conclusão.
9. “Eu estudei muito para esta prova; consequentemente, vou tirar uma boa nota, porque quem estuda bastante
tem resultados bons.”
O trecho acima contém um argumento. Identifique sua(s) premissa(s) e sua conclusão.
10. “Uma vez mais, o melhor é dar exemplos e apontar algumas das características mais salientes dos problemas
filosóficos típicos. Pensemos, por exemplo, em Deus. Os cristãos têm uma dada concepção de Deus, os muçulmanos
outra e os hindus outra ainda. E há muitas mais, tantas quantas as religiões. As religiões partem de certas verdades
reveladas pelos seus profetas e inscritas nos seus livros sagrados; procuram descobrir a verdadeira natureza de Deus e
encontrar o caminho da salvação. Mas nada disso são problemas filosóficos.” [MURCHO, Desidério. O caráter conceitual
da filosofia.]
a) O texto acima menciona uma abordagem não-filosófica ao problema de Deus. Quais as diferenças entre uma
abordagem não-filosófica e uma abordagem filosófica a respeito de algum problema?
b) Sabendo que a argumentação é o método da filosofia, o papel da lógica no estudo filosófico dos problemas torna-se evidente. Um objetivo da lógica é a determinação da validade dos argumentos. Na lógica, qual é a diferença
entre a verdade e a validade?
11. Argumento 1: “Todos os homens que viveram até hoje são mortais. Portanto, é provável que todos os homens
sempre sejam mortais”.
Argumento 2: “Todos os homens que viveram até hoje são mortais. Portanto, é provável que alguns homens sejam imortais no futuro”.
a) Indique se é possível afirmar que algum dos argumentos acima é válido. Justifique sua resposta.
b) A força indutiva não é monotônica . Explique esta afirmação.
12. “A filosofia é uma atividade a priori, pois a filosofia investiga a realidade sem recorrer à experiência e a investigação da realidade sem o recurso da experiência é uma atividade a priori.”
O trecho acima contém um argumento. Identifique sua(s) premissa(s) e sua conclusão.
13. “Como todos os outros estudos, a filosofia aspira essencialmente ao conhecimento. O conhecimento a que aspira é o que unifica e sistematiza o corpo das ciências e o que resulta de um exame crítico dos fundamentos das nossas convicções, dos nossos preconceitos e das nossas crenças. Mas não se pode dizer que a filosofia tenha tido grande
sucesso ao tentar dar respostas exatas às suas questões. Se perguntarmos a um matemático, a um mineralogista, a um
historiador ou a qualquer outro homem de saber, que corpo exato de verdades a sua ciência descobriu, a sua resposta
durará o tempo que estivermos dispostos a escutá-lo. Contudo, se colocarmos a mesma questão a um filósofo, se for
sincero terá de confessar que o seu estudo não chegou a resultados positivos como aqueles a que chegaram outras
ciências. É verdade que isto se explica em parte pelo fato de que assim que se torna possível um conhecimento exato
48
acerca de qualquer assunto, este assunto deixa de se chamar filosofia e passa a ser uma ciência separada. A totalidade
do estudo dos céus, que pertence atualmente à astronomia, esteve em tempos incluído na filosofia; a grande obra de
Newton chamava-se “os princípios matemáticos da filosofia natural”. Analogamente, o estudo da mente humana, que
fazia parte da filosofia, foi agora separado da filosofia e deu origem à ciência da psicologia. Assim, a incerteza da
filosofia é em larga medida mais aparente do que real: as questões às quais já é possível dar uma resposta exata são
colocadas nas ciências, e apenas aquelas às quais não é possível, no presente, dar uma resposta exata, formam o
resíduo a que se chama filosofia.” [RUSSELL, Bertrand. O valor da filosofia.]
a) A oração em itálico é um argumento. Identifique a(s) sua(s) premissa(s) e a sua conclusão.
b) O texto de Bertrand Russel trata da diferença entre a filosofia e a ciência. A filosofia tem um método diferente
do método científico. Explique a diferença entre o método filosófico e o científico.
c) O objeto da filosofia é diferente do objeto da ciência. Cite três exemplos de objetos filosóficos.
14. “Mais tarde, ao analisar com atenção o que eu era, e vendo que podia presumir que não possuía corpo algum e
que não havia mundo algum, ou lugar onde eu existisse, mas que nem por isso podia supor que não existia; e que, ao
contrário, pelo fato mesmo de eu pensar em duvidar da verdade das outras coisas, resultava com bastante evidência e
certeza que eu existia; ao passo que, se somente tivesse parado de pensar, apesar de que tudo o mais que alguma vez
imaginara fosse verdadeiro, já não teria razão alguma de acreditar que eu tivesse existido; compreendi, então, que eu
era uma substância cuja essência ou natureza consiste apenas no pensar, e que, para ser, não necessita de lugar algum,
nem depende de qualquer coisa material. De maneira que esse eu, ou seja, a alma, por causa da qual sou o que sou, é
completamente distinta do corpo e, também, que é mais fácil de conhecer do que ele, e, mesmo que este nada fosse,
ela não deixaria de ser tudo o que é.” [DESCARTES, René. Meditações.]
Responda às questões abaixo.
a) Caracterize a posição de Descartes em relação ao conhecimento do homem sobre o próprio homem.
b) Relacione o texto acima à atitude cética.
c) O trecho selecionado constitui um argumento. Identifique sua(s) premissa(s) e sua conclusão.
d) O trecho selecionado constitui um argumento dedutivo ou indutivo? Responda e justifique.
15. (UEL 2003) “Mas logo em seguida, adverti que, enquanto eu queria assim pensar que tudo era falso, cumpria
necessariamente que eu, que pensava, fosse alguma coisa. E, notando que esta verdade eu penso, logo existo era tão
firme e tão certa que todas as mais extravagantes suposições dos céticos não seriam capazes de a abalar, julguei que
poderia aceitá-la, sem escrúpulo, como o primeiro princípio da Filosofia que procurava.” [DESCARTES, René. Discurso
do método.]
De acordo com o texto e com os conhecimentos sobre o tema, assinale a alternativa correta.
a) Para Descartes, não podemos conhecer nada com certeza, pois tudo quanto pensamos está sujeito à falsidade.
b) O eu penso, logo existo expressa uma verdade instável e incerta, o que fez Descartes ser vencido pelos céticos.
c) A expressão eu penso, logo existo representa a verdade firme e certa com a qual Descartes fundamenta o conhecimento e a ciência.
d) As extravagantes suposições dos céticos impediram Descartes de encontrar uma verdade que servisse como
princípio para a filosofia.
e) Descartes, ao acreditar que tudo era falso, colocava em dúvida sua própria existência.
16. (UFRJ 2008) A disputa entre racionalismo e empirismo se dá no ramo da filosofia destinado ao estudo da natureza, das fontes e dos limites do conhecimento. Essa disputa diz respeito à questão sobre se e em que medida somos
dependentes da experiência sensível para alcançar o conhecimento. Os racionalistas afirmam que nossos conhecimentos têm sua origem independentemente da experiência sensível, isto é, independentemente de qualquer acesso imediato a coisas externas a nós. Os empiristas, por sua vez, consideram que a experiência sensível é a fonte de todos os
nossos conhecimentos. Em relação ao tema, considere a seguinte afirmativa: “Primeiramente, considero haver em nós
certas noções primitivas, as quais são como originais, sob cujo padrão formamos todos os nossos outros conhecimentos.” [DESCARTES, René. Carta a Elizabeth de 21 de maio de 1743.]
Com base no que foi exposto acerca da oposição entre racionalismo e empirismo, responda: a frase de Descartes é
mais representativa da posição racionalista ou da posição empirista? Justifique sua resposta, indicando o(s) elemento(s) da frase que a sustenta(m).
17. (UEL 2003) “Embora nosso pensamento pareça possuir esta liberdade ilimitada, verificaremos, através de um
exame mais minucioso, que ele está realmente confinado dentro de limites muito reduzidos e que todo poder criador
do espírito não ultrapassa a faculdade de combinar, de transpor, aumentar ou de diminuir os materiais que nos foram
fornecidos pelos sentidos e pela experiência.” [HUME, David. Investigação acerca do entendimento humano.]
De acordo com o texto, é correto afirmar que, para Hume:
a) Os sentidos e a experiência estão confinados dentro de limites muito reduzidos.
b) Todo conhecimento depende dos materiais fornecidos pelos sentidos e pela experiência.
c) O espírito pode conhecer as coisas sem a colaboração dos sentidos e da experiência.
d) A possibilidade de conhecimento é determinada pela liberdade ilimitada do pensamento.
e) Para formar as ideias, o pensamento descarta os materiais fornecidos pelos sentidos
49
Capítulo VII
METAFÍSICA
LAERTE. “Piratas do Tietê”. In: Folha de S. Paulo (21 de julho de 2009).
V
ejamos, neste capítulo, outra importante subdivisão da Filosofia: a Metafísica.
Este grande quinhão filosófico trabalha com problemas relacionados aos aspectos mais gerais da estrutura
da realidade; são eles: o sentido da vida, a identidade pessoal, o livre-arbítrio, problemas conceituais do
espaço e do tempo, a natureza da objetividade e da subjetividade, a independência do mundo relativamente à
nossa experiência etc. Dentro da Metafísica encontramos a Ontologia, parte que estuda a existência ou o que
existe.
Se tomarmos como base toda a História da Filosofia, veremos que em determinados períodos os modos de formular as questões e respondê-las modificou-se; ainda assim, uma questão fundamental permaneceu: “o que é?”. Eis,
pois, a questão a partir da qual a investigação metafísica se funda. Se tomarmos o verbo “é” dessa aparentemente
simples questão, podemos dar a ele dois sentidos, a saber, “é” significa “existe” e, com isso, a pergunta se refere à
existência da realidade; “é” significa “natureza própria de algo” e, assim, a pergunta se refere à essência da realidade.
Como se nota, a existência e a essência da realidade em seus diversos aspectos são, assim, os temas centrais da Metafísica, que investiga, como já dissemos, as causas, os fundamentos, o ser íntimo de todas as coisas, investigando por
que existem e por que são o que são.
Como toda matéria complexa, a Metafísica é subdividida em períodos para melhor compreensão; sua história,
com isso, divide-se em três períodos: o “primeiro período” vai de Platão e Aristóteles (séculos IV e III a.e.c.) até
David Hume (século XVIII); o “segundo período” vai de Immanuel Kant (século XVIII) até a fenomenologia de
Husserl (século XX); e o “terceiro período” da Metafísica (nessa fase também chamada de “Ontologia contemporânea”) vai dos anos 20 aos anos 70 do século XX. Vejamos, agora, as principais características de cada período:
No primeiro período, a Metafísica apresentará as seguintes características: conhecimento sistemático em que cada
conceito depende de outros e se relaciona com outros, constituindo um sistema coerente de ideias ligadas entre si;
conhecimento racional que não se baseia nos dados conhecidos diretamente pela experiência sensível, mas nos conceitos puros formulados pelo pensamento; investigação daquilo que é ou existe; exigência da distinção entre “realidade” e “aparência” ou entre “ser” e “parecer” (pois, para certos filósofos dessa época, a aparência poderia ser compreendida e explicada apenas pelo conhecimento da realidade que subjaz a ela.
Esse primeiro período da metafísica termina quando Hume demonstra que os conceitos metafísicos não correspondem a nenhuma realidade externa, existente em si mesma e independente de nós, mas são meros nomes gerais para as coisas, nomes que
nos vêm pelo hábito mental ou psíquico de associar em ideias as sensações, as percepções e as impressões dos sentidos, quando são constantes, frequentes e regulares. [CHAUÍ, Marilena. Convite à filosofia.]
A palavra “Metafísica” surgiu por intermédio de Andrônico de Rodes, quando este classificou as obras de Aristóteles. Com essa palavra — ta meta ta physika —, indicava-se um conjunto de escritos que, em sua classificação,
localizavam-se depois (meta: depois de) dos tratados sobre a física ou sobre a Natureza. Mas, embora a Metafísica
tenha começado com os filósofos Parmênides e Platão, costuma-se atribuir seu nascimento a Aristóteles por três
motivos principais:
1. diferentemente de seus dois predecessores, Aristóteles não julga o mundo das coisas sensíveis, ou a Natureza, um mundo
aparente e ilusório. Pelo contrário, é um mundo real e verdadeiro cuja essência é, justamente, a multiplicidade de seres e a
mudança incessante. Em lugar de afastar a multiplicidade e o devir como ilusões ou sombras do verdadeiro Ser, Aristóteles afirma que o ser da Natureza existe, é real, que seu modo próprio de existir é a mudança e que esta não é uma contradição impensável. É possível uma ciência teorética verdadeira sobre a Natureza e a mudança: a física. Mas é preciso, primeiro, demonstrar que o objeto da física é um ser real e verdadeiro e isso é tarefa da Filosofia Primeira ou da metafísica.
2. diferentemente de seus dois predecessores, Aristóteles considera que a essência verdadeira das coisas naturais e dos seres humanos e de suas ações não está no mundo inteligível, separado do mundo sensível, onde as coisas físicas ou naturais existem e onde vivemos. As essências, diz Aristóteles, estão nas próprias coisas, nos próprios homens, nas próprias ações e é tarefa da Filosofia conhecê-las ali mesmo onde existem e acontecem. Como conhecê-las? Partindo da sensação até alcançar a
intelecção. A essência de um ser ou de uma ação é conhecida pelo pensamento, que capta as propriedades internas desse ser
ou dessa ação, sem as quais ele ou ela não seriam o que são. A metafísica não precisa abandonar este mundo, mas, ao contrário, é o conhecimento da essência do que existe em nosso mundo. O que distingue a ontologia ou metafísica dos outros saberes
(isto é, das ciências e das técnicas) é o fato de que nela as verdades primeiras ou os princípios universais e toda e qualquer realidade são conhecidos direta ou indiretamente pelo pensamento ou por intuição intelectual, sem passar pela sensação, pela
imaginação e pela memória.
50
3. ao se dedicar à Filosofia Primeira ou metafísica, a Filosofia descobre que há diferentes tipos ou modalidades de essências ou de ousiai. Existe a essência dos seres físicos ou naturais (minerais, vegetais, animais, humanos), cujo modo de ser se
caracteriza por nascer, viver, mudar, reproduzir-se e desaparecer — são seres em devir e que existem no devir. Existe a essência dos seres matemáticos, que não existem em si mesmos, mas existem como formas das coisas naturais, podendo, porém, ser
separados delas pelo pensamento e ter suas essências conhecidas; são seres que, por essência, não nascem, não mudam, não se
transformam nem perecem, não estando em devir nem no devir. Existe a essência dos seres humanos, que compartilham com as
coisas físicas o surgir, o mudar e o desaparecer, compartilhando com as plantas e os animais a capacidade para se reproduzir,
mas distinguindo-se de todos os outros seres por serem essencialmente racionais, dotados de vontade e de linguagem. Pela razão, conhecem; pela vontade, agem; pela experiência, criam técnicas e artes. E, finalmente, existe a essência de um ser eterno,
imutável, imperecível, sempre idêntico a si mesmo, perfeito, imaterial, conhecido apenas pelo intelecto, que o conhece como
separado de nosso mundo, superior a tudo que existe, e que é o ser por excelência: o ser divino. [Idem.]
Nesse sentido, Aristóteles averiguou que, se existiam diferentes tipos de essências, e se para cada uma delas havia
uma ciência própria (biologia, física, psicologia, matemática, ética, política etc.), deveria haver uma ciência geral e
mais ampla, universal, que fosse anterior a todas essas, cujo objeto de estudo fosse a essência em geral, que investiga
o que é a essência e aquilo que faz com que haja essências particulares e individualizadas: a Metafísica. Assim, ao
defini-la como o estudo do “ser enquanto ser”, Aristóteles quis dizer que a Filosofia Primeira estuda as essências das
coisas sem diferenciar essências físicas, matemáticas, humanas etc., pois competiria às diferentes ciências estudá-las
enquanto diferentes entre si. À Metafísica pertenceriam três estudos:
1. o do ser divino, a realidade primeira e suprema da qual todo o restante procura aproximar-se, imitando sua perfeição
imutável. As coisas se transformam, diz Aristóteles, porque desejam encontrar sua essência total e perfeita, imutável como a
essência divina. É pela mudança incessante que buscam imitar o que não muda nunca. Por isso, o ser divino é o Primeiro Motor Imóvel do mundo, isto é, aquilo que, sem agir diretamente sobre as coisas, ficando à distância delas, as atrai, é desejado
por elas. Tal desejo as faz mudar para, um dia, não mais mudar (esse desejo, diz Aristóteles, explica por que há o devir e por
que o devir é eterno, pois as coisas naturais nunca poderão alcançar o que desejam, isto é, a perfeição imutável) [...];
2. o dos primeiros princípios e causas primeiras de todos os seres ou essências existentes;
3. o das propriedades ou atributos gerais de todos os seres, sejam eles quais forem, graças aos quais podemos determinar
a essência particular de um ser particular existente. A essência ou ousia é a realidade primeira e última de um ser, aquilo sem
o qual um ser não poderá existir ou sem o qual deixará de ser o que é. À essência, entendida sob essa perspectiva universal,
Aristóteles dá o nome de substância: o substrato ou o suporte permanente de qualidades ou atributos necessários de um ser. A
metafísica estuda a substância em geral. [Idem.]
Dentro desses estudos, Aristóteles esboçou os principais conceitos de sua Metafísica — conceitos que se tornarão
os fundamentos de toda a metafísica ocidental posterior. Para o filósofo grego, eram três os princípios que estudamos
na Lógica (identidade, não-contradição e terceiro excluído), sendo tais princípios lógicos também ontológicos porque
determinariam as condições sem as quais um ser não poderia existir nem ser pensado. Esses princípios garantiam, ao
mesmo tempo, a racionalidade e a realidade das coisas. Nesse âmbito, Aristóteles apontou também as causas primeiras, ou seja, as razões que explicavam o que a essência era, bem como explicavam a origem e o motivo da existência
de uma essência; essas causas primeiras nos diriam “o que é”, “como é”, “por que é” e “para que é” uma essência,
sendo, pois, quatro causas:
Causa material, isto é, aquilo de que uma essência é feita, sua matéria (por exemplo, água, fogo, ar, terra);
Causa formal, isto é, aquilo que explica a forma que uma essência possui (por exemplo, o rio ou o mar são formas da água;
mesa é a forma assumida pela matéria madeira com a ação do carpinteiro; margarida é a forma que a matéria vegetal possui na
essência de uma flor determinada, etc.);
Causa eficiente ou motriz, isto é, aquilo que explica como uma matéria recebeu uma forma para constituir uma essência
(por exemplo, o ato sexual é a causa eficiente que faz a matéria do espermatozóide e do óvulo receber a forma de um novo animal ou de uma criança; o carpinteiro é a causa eficiente que faz a madeira receber a forma da mesa; o fogo é a causa eficiente
que faz os corpos frios tornarem-se quentes, etc.); e
A causa final, isto é, a causa que dá o motivo, a razão ou finalidade para alguma coisa existir e ser tal como ela é (por exemplo, o bem comum é a causa final da política, a felicidade é a causa final da ação ética; a flor é a causa final da semente
transformar-se em árvore; o Primeiro Motor Imóvel é a causa final do movimento dos seres naturais, etc.). [Idem.]
Prosseguindo com sua Metafísica, Aristóteles esboçou as ideias de “matéria” e “forma”. Matéria seria o elemento
constituinte das coisas da Natureza, dos homens, dos outros animais, dos artefatos, e sua principal característica seria
a de possuir virtualidades ou conter em si mesma possibilidades de modificação. Já a forma era o que individualizaria
e determinaria uma matéria, fazendo existir as coisas ou os seres particulares; sua principal característica seria ser
aquilo que uma essência era em um momento determinado, pois a forma seria o que atualizaria as virtualidades contidas na matéria.
Outras importantes definições aristotélicas foram embasadas nas palavras “ato” e “potência”: esta seria o que está
contido em uma matéria, podendo vir a existir caso for atualizado por alguma causa (a criança, por exemplo, seria um
adulto em potência; a semente, uma árvore em potencial); já o ato seria a atualidade de uma matéria, ou melhor, sua
forma em um determinado momento do tempo. O ato, desse modo, seria a forma que atualizou uma potência contida
na matéria (o adulto seria o ato da criança, a árvore seria o ato da semente, o universitário seria o ato do vestibulando
etc.).
Mas até agora tratamos da essência. E o que seria isso? Essência seria a unidade interna e indissolúvel entre uma
forma e uma matéria, ou seja, a unidade que lhe dá um conjunto de características que a fazem ser essencialmente
aquilo que ela é (um ser humano é, por essência, um animal racional, mortal, possuidor de vontade, originado a partir
de outros semelhantes a ele e capaz de gerar outros semelhantes a ele etc.). Por fim, entendamos que a substância ou
o sujeito seria o substrato ou a base na qual se desempenham a matéria-potência, a forma-ato, onde estão as caracte51
rísticas essenciais e acidentais, que obedece aos três princípios lógico-ontológicos (identidade, não-contradição e
terceiro excluído) e sobre o qual agem as quatro causas (material, formal, eficiente e final); em resumo: o Ser.
Desse modo, a Metafísica aristotélica estabelece o estudo da composição geral de todos os seres, bem como as
condições universais e necessárias a partir das quais pode existir um ser que possa ser conhecido pelo pensamento.
Aristóteles assegurava que a realidade, no seu todo, era inteligível e apresentava-se como conhecimento teorético da
realidade sob todos os seus aspectos, devendo, por isso, anteceder as investigações que cada ciência realizaria sobre
uma espécie determinada de ser.
Contudo, a partir do final do século XVI e no início do século XVII, o pensamento ocidental sofre transformações
importantes no pensamento metafísico.
Os filósofos clássicos (século XVII) julgavam-se modernos por terem rompido com a tradição do pensamento platônico, aristotélico e neoplatônico e, por conseguinte, por não mais aceitarem a tradição que havia sido elaborada pelos medievais. Um dos
exemplos mais conhecidos da modernidade é a recusa do geocentrismo e a adoção do heliocentrismo, em astronomia. Um outro exemplo é a nova física ou mecânica, elaborada por Galileu contra a herança aristotélica. [Idem.]
Dentre as características desse segundo período, temos a afirmação da incompatibilidade entre razão e fé, o que
acarretou a separação de ambas; com isso, Filosofia e religião teriam que seguir caminhos próprios (ainda que, na
época, a Filosofia não estivesse publicamente autorizada a expor argumentos que contradissessem as verdades ou
dogmas da fé). Assistiu-se também a uma redefinição do conceito de ser ou substância; ao invés de considerar a
existência de inumeráveis tipos de seres ou substâncias, afirmou-se que existiam apenas três seres ou substâncias, a
saber, a “substância infinita” (que existe absolutamente em si e por si, isto é, o infinito ou Deus), a “substância pensante” (a que existe em si, mas para existir teve que ser criada pelo infinito, qual seja, a alma) e a “substância extensa” (que também existe em si, mas que, para existir, também teve que ser criada pelo infinito: o corpo); nota-se, dessa
maneira, uma grande simplificação do campo de investigação da Metafísica que, com Aristóteles, definia de modo
mais complexo a substância. Houve, ainda, a redefinição do conceito de causa ou causalidade.
Causa é aquilo que produz um efeito. O efeito pode ser produzido por uma ação anterior ou por uma finalidade posterior. Por
exemplo, o fogo realiza uma ação anterior, cujo efeito é o aquecimento e a dilatação de um outro corpo; uma pessoa pode escolher entre fazer ou não fazer alguma coisa, tendo em vista a finalidade que pretende alcançar, de sorte que o fim ou o objetivo é algo posterior à ação e causa da decisão tomada. Causa eficiente é aquela na qual uma ação anterior determina como
consequência necessária a produção de um efeito. Causa final é aquela que determina, para os seres pensantes, a realização
ou não-realização de uma ação. Há duas e somente duas modalidades de causas — a eficiente e a final — e a causa final só
atua na substância pensante, referindo-se às ações de um sujeito. Não há causa final para os corpos ou para a substância extensa, mas apenas causa eficiente. Desaparecendo as noções de causa material e causa formal, desaparecem as de potência e
ato, matéria e forma como explicações da pluralidade dos seres e de suas transformações; a metafísica não se divide em teologia, psicologia racional e cosmologia racional. [Idem.]
Após essas modificações, temos que a substância infinita ou Deus seria a causa da existência e da essência das
substâncias pensante e extensa; e seria, sobretudo, causa das relações entre ambas, no caso do homem (já que este é
uma substância mista). Deus, homem e Natureza seriam os objetos da Metafísica; infinito, finito, causa eficiente e
causa final seriam os primeiros princípios de que se ocuparia a Metafísica. Ideias verdadeiras produzidas pelo intelecto humano, com as quais o sujeito do conhecimento representa e conhece a realidade, seriam os fundamentos da
Metafísica como ciência verdadeira ou como Primeira Filosofia.
Enfim, se a realidade pesquisada pela Metafísica é aquela que pode e deve ser racionalmente fundada pelas ideias
verdadeiras determinadas pela razão humana, que aconteceria se se provasse que tais ideias eram meros hábitos mentais do sujeito do conhecimento, não correspondendo a realidade alguma? Tenhamos em mente, para entender a
questão, que as Metafísicas antiga e medieval baseavam-se na afirmação de que a realidade ou o Ser existiam em si
mesmos, oferecendo-se, tal como eram, ao pensamento; já as Metafísicas clássica ou moderna baseavam-se na afirmação de que o intelecto humano ou o pensamento possuíam a capacidade de conhecer a realidade tal como era em si
mesma e que, em virtude das operações intelectuais ou dos conceitos que representavam as coisas e as transformavam
em objetos de conhecimento, o sujeito do conhecimento tinha acesso ao Ser.
Tanto em um como no outro caso, a Metafísica baseava-se em dois pressupostos: a realidade em si existe e pode
ser conhecida; ideias ou conceitos são um conhecimento verdadeiro da realidade, pois a verdade é a correspondência
entre as coisas e os pensamentos (ou entre o intelecto e a realidade). Esses dois pressupostos assentavam-se, por sua
vez, em um único fundamento: a existência de um Ser Infinito (Deus) que garantiria a realidade e a inteligibilidade de
todas as coisas existentes, dotando os seres humanos de um intelecto capaz de conhecê-las tais como eram em si
mesmas.
Nesse ínterim, o filósofo David Hume dirá que os dois pressupostos da Metafísica não têm fundamento. Argumentara ele que se o sujeito do conhecimento operava associando sensações, percepções e impressões recebidas pelos
órgãos dos sentidos e retidas na memória, as ideias nada mais seriam do que hábitos mentais de associação de impressões semelhantes ou de impressões sucessivas.
Que é a ideia de substância ou de essência? Nada mais do que um nome geral dado para indicar um conjunto de imagens
e de ideias que nossa consciência tem o hábito de associar por causa das semelhanças entre elas. O princípio da identidade e o
da não-contradição são simplesmente o resultado de percebermos repetida e regularmente certas coisas semelhantes e sempre
da mesma maneira, levando-nos a supor que, porque as percebemos como semelhantes e sempre da mesma maneira, isso lhes
daria uma identidade própria, independente de nós.
Que é a ideia de causalidade? O mero hábito que nossa mente adquire de estabelecer relações de causa e efeito entre percepções e impressões sucessivas, chamando as anteriores de causas e as posteriores de efeitos. A repetição constante e regular
52
de imagens ou impressões sucessivas nos leva à crença de que há uma causalidade real, externa, própria das coisas e independente de nós. [Idem.]
Para Hume, se a Metafísica foi sempre alimentada por controvérsias infindáveis era porque ela não se referia a
nenhuma realidade externa existente em si e por si, mas sim a hábitos mentais dos sujeitos; hábitos variáveis que
originaram inúmeras doutrinas filosóficas sem qualquer fundamento real. Com Hume, a Metafísica mostrou-se impossível de ser continuada tal como existira desde o século IV a.e.c.
Após essa crise, no segundo período a Metafísica encontrou seu centro na filosofia de Immanuel Kant, filósofo
que demonstrou a impossibilidade dos conceitos metafísicos clássicos para alcançar e conhecer a realidade em si das
coisas, propondo, em lugar disso, que a Metafísica fosse o conhecimento de nossa própria capacidade de conhecer,
adotando a realidade como aquilo que existe para nós à medida em que somos os sujeitos do conhecimento.
Ao observamos os conceitos de verdade e razão, notaremos que Kant realizou uma “revolução copernicana” na
Filosofia; isso porque determinou que, antes de qualquer afirmação sobre as ideias, existisse um estudo aprofundado
da própria capacidade de conhecer, (ou seja, da razão). Kant iniciou tal revolução distinguindo duas grandes modalidades de conhecimento: os “conhecimentos empíricos” (baseados nos dados da experiência psicológica de cada ser
humano) e os “conhecimentos apriorísticos” (baseados unicamente na estrutura interna da própria razão, independentemente da experiência individual de cada ser humano). Após o advento das ideias de Hume, Kant reagiu aos novos
problemas ao declarar que, graças ao filósofo inglês, foi capaz de “acordar do sono dogmático” — dogmático seria,
pois, aquele que aceita, sem exame e sem crítica, afirmações sobre as coisas e sobre as ideias, sem questioná-las. Para
Kant, Hume despertou a Metafísica do “sono dogmático” porque a forçou a investigar sobre sua própria validade e
sua pretensão ao conhecimento verdadeiro. Com isso, Kant pôde “abrir os olhos” e indagar, antes de qualquer coisa,
se a Metafísica era possível e, se o fosse, em que condições era possível. Elaborou, assim, uma crítica da razão teórica, ou melhor, um estudo sobre a estrutura e o poder da razão em se tratando de determinar o que ela poderia e o que
não poderia conhecer verdadeiramente.
Em seguida, Kant distinguiu as duas maneiras pelas quais esses dois tipos de conhecimentos (empíricos e apriorísticos) se exprimem: os “juízos analíticos” (aqueles em que o predicado não é senão a explicitação do conteúdo do
sujeito: “o triângulo é uma figura de três lados”) e os “juízos sintéticos” (aqueles nos quais o predicado acrescenta
dados novos sobre o sujeito: “Aristóteles é filósofo”).
Antes de prosseguirmos, vejamos o que são exatamente os juízos:
Um juízo é uma afirmação ou uma negação referente a propriedades de um sujeito, isto é, a maneira como o conhecimento
afirma ou nega o que uma coisa é ou não é. Como a realidade ou o objeto é aquilo que pode ser conhecido através das formas
a priori da sensibilidade e dos conceitos a priori do entendimento, um juízo é a afirmação ou a negação da realidade de um
objeto pela afirmação ou negação de suas propriedades. [...] Um juízo, portanto, nos dá a conhecer alguma coisa, desde que
esta possa ser apreendida sob as formas do espaço e do tempo e sob os conceitos do entendimento. Uma coisa passa a existir
quando se torna objeto de um juízo. Isso não significa que o juízo cria a própria coisa, mas sim que a faz existir para nós. O juízo põe a realidade de alguma coisa ao colocá-la como sujeito de uma proposição, isto é, ao colocá-la como objeto de um conhecimento. É, portanto, o juízo que põe a qualidade, a quantidade, a causalidade, a substância, a matéria, a forma, a essência
das coisas, na medida em que estas existem apenas enquanto são objetos de conhecimento postos pelas formas do espaço, do
tempo e pelos conceitos do entendimento.
Um juízo, para ter valor científico e filosófico ou valor teórico, deve preencher duas condições: 1. deve ser universal e necessário; 2. deve ser verdadeiro, isto é, corresponder à realidade que enuncia. Os juízos analíticos, diz Kant, preenchem as duas condições, mas não os juízos sintéticos. Por quê? Porque um juízo sintético se baseia nos dados da experiência psicológica
individual e, como bem mostrou Hume, tal experiência nos dá sensações e impressões que associamos em ideias, mas estas não
são universais e necessárias, nem correspondem à realidade. Ora, um juízo analítico não nos traz conhecimentos, pois simplesmente repete, no predicado, o conteúdo do sujeito. Somente juízos sintéticos são fonte de conhecimento. Portanto, se quisermos realizar metafísica e ciência, temos, primeiro, que provar que são possíveis juízos sintéticos universais, necessários e
verdadeiros e, portanto, demonstrar que tais juízos não podem ser empíricos. Dizer que um juízo sintético é universal, necessário e verdadeiro e dizer que não pode ser empírico significa dizer que o juízo sintético filosófico e científico tem que ser um juízo sintético apriorístico ou a priori, isto é, tem que depender de alguma coisa que não seja a experiência. [Idem.]
Dessarte, a pergunta “seria possível a Metafísica?” poderia ser respondida apenas se, antes, fosse confirmado se
haveria ou não juízos sintéticos a priori (ou seja, Deus, alma, mundo, substância, matéria, infinito, finito, causalidade
etc.). Kant, por conseguinte, demonstrou a existência e a validade dos juízos sintéticos a priori nas ciências, evidenciando que o conhecimento da realidade nada mais era do que o modo pelo qual a razão, por meio de sua estrutura
universal, organizava de modo universal e necessário os dados da experiência: graças às formas a priori da sensibilidade (espaço e tempo) e dos conceitos a priori do entendimento (as categorias de substância, causalidade, quantidade,
qualidade etc.), o ser humano possuía uma capacidade inata, universal e necessária de conhecimento, que não dependeria da experiência, mas se realizaria em virtude da experiência sobre os objetos que esta nos oferece.
Em outras palavras, uma coisa existe quando pode ser posta pelo sujeito do conhecimento, entendido não como um sujeito individual e psicológico (João, Pedro, Maria, Ana), mas como o sujeito universal ou estrutura a priori universal da razão humana, aquilo que Kant denomina de Sujeito Transcendental. Quando o juízo for sintético e a priori, o conhecimento obtido é universal, necessário e verdadeiro. No entanto, a demonstração de que, graças às formas a priori da sensibilidade e graças aos
conceitos a priori do entendimento, os juízos sintéticos a priori são possíveis, é uma demonstração que não ajuda em nada a
pergunta sobre a possibilidade da metafísica. Por quê? Kant distinguiu duas modalidades de realidade. A realidade que se oferece a nós na experiência e a realidade que não se oferece à experiência. A primeira foi chamada por ele de fenômeno, isto é,
aquilo que se apresenta ao sujeito do conhecimento na experiência, é estruturado pelo sujeito com as formas do espaço e do
tempo e com os conceitos do entendimento, é sujeito de um juízo e objeto de um conhecimento. A segunda foi chamada por ele
de nôumeno, isto é, aquilo que não é dado à sensibilidade nem ao entendimento, mas é afirmado pela razão sem base na experiência e no entendimento. O fenômeno é a coisa para nós ou o objeto do conhecimento propriamente dito, é o objeto enquanto
53
sujeito do juízo. O nôumeno é a coisa em si ou o objeto da metafísica, isto é, o que é dado para um pensamento puro, sem relação com a experiência. Ora, só há conhecimento universal e necessário daquilo que é organizado pelo sujeito do conhecimento
nas formas do espaço e do tempo e de acordo com os conceitos do entendimento. Se o nôumeno é aquilo que nunca se apresenta à sensibilidade, nem ao entendimento, mas é afirmado pelo pensamento puro, não pode ser conhecido. E se o nôumeno é o
objeto da metafísica, esta não é um conhecimento possível. [Idem.]
A partir dessa perspectiva, Kant entendeu que a Metafísica havia sido, desde então, uma “insensatez dogmática”,
uma pretensão em conhecer seres (tais como Deus) que escapam de toda possibilidade humana de conhecimento,
posto que eram seres aos quais não se aplicam as condições universais e necessárias dos juízos (espaço, tempo, causalidade, qualidade, quantidade etc.). Essa metafísica, para Kant, não é possível; no entanto, não significava que toda
Metafísica fosse impossível.
Qual é a metafísica possível? É aquela que tem como objeto a investigação dos conceitos usados pelas ciências — espaço,
tempo, quantidade, qualidade, causalidade, substancialidade, universalidade, necessidade, etc. —, isto é, que tem como objeto
o estudo das condições de possibilidade de todo conhecimento humano e de toda a experiência humana possíveis. A metafísica
estuda, portanto, as condições universais e necessárias da objetividade em geral e não o “Ser enquanto Ser”, nem Deus, alma
e mundo, nem substância infinita, pensante e extensa. Estuda as maneiras pelas quais o sujeito do conhecimento, ou a razão
teórica, põe a realidade, isto é, estabelece os objetos do conhecimento e da experiência. A metafísica é o conhecimento do conhecimento humano e da experiência humana, ou, em outras palavras, do modo como os seres humanos, enquanto expressões
do Sujeito Transcendental, definem e estabelecem realidades. [Idem.]
Não obstante, Kant apontou outro objeto para a Metafísica; um objeto prático, e não teórico: a ação humana enquanto ação moral (a ser estudada no próximo capítulo). A moral, no entender de Kant, poderia se tornar objeto da
metafísica em virtude da liberdade.
A razão teórica mostra que todos os seres, incluindo os homens, são seres naturais. Isso significa que são seres submetidos a
relações necessárias de causa e efeito. A Natureza é o reino das leis naturais de causalidade. Nela, tudo acontece de modo necessário ou causal, não havendo lugar para escolhas livres. No entanto, os seres humanos são capazes de agir por escolha livre, por determinação racional de sua vontade e são capazes de agir em nome de fins ou finalidades humanas, e não apenas
condicionados por causas naturais necessárias. A ação livre ou por escolha voluntária ou racional é uma ação por finalidade e
não por causalidade. Nesse sentido, a ação moral mostra que, além do reino causal da Natureza, existe o reino ético da liberdade e da finalidade. Cabe à metafísica o estudo dessa outra modalidade de realidade, que não é natural nem teórica, mas prática. [Idem.]
Por fim, no terceiro período a Metafísica (ou Ontologia) procura ir além da antiga Metafísica (ou seja, conhecimento da realidade em si, independente de nós), bem como da compreensão kantiana (a saber, conhecimento da
realidade como aquilo que é para nós). Considera-se, assim, o objeto da Metafísica como sendo a relação originária
mundo-homem. Como características dessa fase, temos: investigação dos diferentes modos como os entes ou os seres
existem, além de indagar sobre a essência (ou o sentido) e a estrutura desses entes ou seres; averigua, também, a
relação necessária entre a existência e a essência dos entes e a maneira pela qual surgem para nossa consciência
(imaginação, memória, percepção, linguagem, ação política, ação moral, prática artística etc.). Contudo, esse período
não busca oferecer uma explicação da realidade, e sim uma interpretação racional da lógica dessa realidade, de maneira a descrever as estruturas do mundo e as do pensamento humano.
Depois do surgimento da solução kantiana para o problema da Metafísica, esta não mais voltou à antiga concepção de conhecimento da realidade em si; pelo contrário, seguiu no sentido inaugurado por Kant, conhecido como
“Idealismo”. Entendamos, pois, por quê. Antes, a Filosofia era o que chamamos de “realista” (ou seja, partia da afirmação de que o Ser ou a realidade existem em si mesmos e que, enquanto tais, podem ser conhecidos pela razão
humana). Com o realismo clássico ou moderno, introduziu-se uma alteração no realismo antigo e medieval, pois
estabelecera que, antes de iniciar uma investigação metafísica da realidade, fosse respondida a questão “podemos
conhecer a realidade?” (isto é, exigira que a teoria do conhecimento antecedesse a Metafísica). A seguir, o resultado
dessa exigência surgiu com Hume, demonstrando que o conteúdo da Metafísica são apenas nossas ideias e que estas
são nomes gerais atribuídos aos hábitos psicológicos de associar as informações da sensação e da percepção.
Completando a trajetória moderna, Kant trouxe duas inovações fundamentais: transformou a própria teoria do conhecimento em Metafísica e, também, demonstrou que o sujeito do conhecimento não é, como pensou Hume, o sujeito psicológico individual, e sim uma estrutura universal, idêntica para todos os seres humanos em todos os tempos e
lugares, a saber, a razão (também chamada de “Sujeito transcendental”), como faculdade a priori de conhecer. Podese notar que, com isso, a história da Metafísica foi sempre o trabalho filosófico para responder a duas perguntas: “o
que é aquilo que existe?” e “como podemos conhecer aquilo que existe?”. Duas respostas surgiram: a “realista”, cujo
mais proeminente exemplo é a metafísica de Aristóteles, e a “idealista”, cujo melhor exemplo é a crítica da razão
teórica e prática de Kant.
Mantendo-se fiel à tradição moderna e kantiana, Edmund Husserl privilegiou a consciência reflexiva ou o sujeito
do conhecimento, ou seja, afirmou
que as essências descritas pela Filosofia são produzidas ou constituídas pela consciência, enquanto um poder para dar significação à realidade. A consciência de que fala o filósofo não é, evidentemente, aquela de que fala o psicólogo. Para este, a consciência é o nome dado a um conjunto de fatos externos e internos observáveis e explicados causalmente. A consciência a que se
refere o filósofo é o sujeito do conhecimento, como estrutura e atividade universal e necessária do saber. É a Consciência
Transcendental ou o Sujeito Transcendental. Qual é o poder da consciência reflexiva? O de constituir ou criar as essências,
pois estas nada mais são do que as significações produzidas pela consciência, enquanto um poder universal de doação de sentido ao mundo. A consciência não é uma coisa entre as coisas, não é um fato observável, nem é, como imaginava a metafísica,
54
uma substância pensante ou uma alma, entidade espiritual. A consciência é uma pura atividade, o ato de constituir essências
ou significações, dando sentido ao mundo das coisas. Estas — ou o mundo como significação — são o correlato da consciência, aquilo que é visado por ela e dela recebe sentido. [Idem.]
Desde Kant, o que era denominado de “fenômeno” indicava aquilo que, do mundo externo, se oferecia ao sujeito
do conhecimento. Contra Kant, Husserl afirmou que não havia nôumeno (a “coisa em si”, incognoscível): tudo o que
existia era fenômeno e só existiriam fenômenos, ou seja, presenças reais de coisas reais diante da consciência. A
consciência, assim, não se encarnaria nas coisas, não se tornaria as próprias coisas; pelo contrário, daria significação a
elas, permanecendo diferente delas.
Ao ampliar o conceito de fenômeno, Husserl propôs que a Filosofia distinguisse diferentes tipos de essências ou fenômenos e
que considerasse cada um deles como manifestando um tipo de realidade diferente, um tipo de ser diferente. Falou, assim, em
regiões do ser: a região Consciência, a região Natureza, a região Matemática, a região Arte, a região História, a região Religião, a região Política, a região Ética, etc. Propôs que a Filosofia investigasse as essências próprias desses seres ou desses entes, criando ontologias regionais. Com essa proposta, Husserl fazia com que a metafísica do “Ser enquanto Ser” e a metafísica
das substâncias (Deus, alma, mundo; infinito, pensante, extensa) cedessem lugar ao estudo do ser diferenciado em entes dotados de essências próprias e irredutíveis uns aos outros. Esse estudo seria a ontologia sob a forma de ontologias regionais. [Idem.]
Os filósofos que vieram após Husserl adotaram suas ideias e desenvolveram uma nova ontologia. Entre eles, Martin Heidegger e Maurice Merleau-Ponty. Ambos se esforçaram para liberar a Ontologia do antigo problema deixado
pela Metafísica, a saber, o dilema do realismo e do idealismo (dilema que Husserl resolveu em favor do idealismo).
A nova ontologia parte da afirmação de que estamos no mundo e de que o mundo é mais velho do que nós (isto é, não esperou
o sujeito do conhecimento para existir), mas, simultaneamente, de que somos capazes de dar sentido ao mundo, conhecê-lo e
transformá-lo. Não somos uma consciência reflexiva pura, mas uma consciência encarnada num corpo. Nosso corpo não é apenas uma coisa natural, tal como a física, a biologia e a psicologia o estudam, mas é um corpo humano, isto é, habitado e animado por uma consciência. Não somos pensamento puro, pois somos um corpo. Não somos uma coisa natural, pois somos
uma consciência. O mundo não é um conjunto de coisas e fatos estudados pelas ciências segundo relações de causa e efeito e
leis naturais. Além do mundo como conjunto racional de fatos científicos, há o mundo como lugar onde vivemos com os outros
e rodeados pelas coisas, um mundo qualitativo de cores, sons, odores, figuras, fisionomias, obstáculos, um mundo afetivo de
pessoas, lugares, lembranças, promessas, esperanças, conflitos, lutas. Somos seres temporais — nascemos e temos consciência
da morte. Somos seres intersubjetivos — vivemos na companhia dos outros. Somos seres culturais — criamos a linguagem, o
trabalho, a sociedade, a religião, a política, a ética, as artes e as técnicas, a filosofia e as ciências. O que é, pois, a realidade?
É justamente a existência do mundo material, natural, ideal, cultural e a nossa existência nele. A realidade é o campo formado
por seres ou entes diferenciados e relacionados entre si, que possuem sentido em si mesmos e que também recebem de nós outros e novos sentidos. [Idem.]
O que é o mundo? O que é o eu ou a consciência? O que é o corpo? O que é o outro? O que é o espaço-tempo?
O que é a linguagem? O que é o trabalho? A religião? A arte? A sociedade? A história? A morte? O infinito? As
questões abordadas pela Ontologia recuperam, assim, a velha questão filosófica (“o que é isto que é?”), mas com uma
nova questão adicionada (“para quem é isto que é?”). Volta-se, com isso, a buscar o Ser ou a essência das coisas, dos
valores humanos, dos atos, da vida, da morte etc. A Ontologia (como parte da Metafísica) investiga a essência ou
sentido do ente natural ou físico, do ente psíquico, lógico, estético, ético, temporal, espacial etc.; inquire as diferenças
e as relações entre eles, seu modo próprio de existir, sua origem, sua finalidade.
Do que expomos até aqui, pode-se notar o valor da Metafísica que, como uma fração medular da Filosofia, mostra-se como a ciência filosófica fundamental, posto que governa todos os domínios da Filosofia em seus mais íntimos
fundamentos, dando ao homem a possibilidade de distinguir e entender as questões mais fundamentais sobre si mesmo e sobre o mundo, de modo que é capaz de permitir ao ser humano o esclarecimento de sua situação existencial e
de seu significado enquanto ser.
Leituras complementares
É costume dizer-se que cada um tem a sua filosofia e até que todos os homens têm opiniões metafísicas. Nada
poderia ser mais tolo. É verdade que todos os homens têm opiniões, e que algumas delas — tais como as opiniões
sobre religião, moral e o sentido da vida — confinam com a filosofia e a metafísica, mas raros são os homens que
possuem qualquer concepção de filosofia e ainda menos os que têm qualquer noção de metafísica.
William James definiu algures a metafísica como “apenas um esforço extraordinariamente obstinado para pensar
com clareza”. Não são muitas as pessoas que assim pensam, excepto quando os seus interesses práticos estão envolvidos. Não têm necessidade de assim pensar e, daí, não sentem qualquer propensão para o fazer. Exceptuando algumas raras almas meditativas, os homens percorrem a vida aceitando como axiomas, simplesmente, aquelas questões
da existência, propósito e sentido que aos metafísicos parecem sumamente intrigantes. O que sobretudo exige a atenção de todas as criaturas, e de todos os homens, é a necessidade de sobreviver e, uma vez isso razoavelmente assegurado, a necessidade de existir com toda a segurança possível. Todo o pensamento começa aí, e a sua maior parte cessa
aí. Sentimo-nos mais à vontade para pensar como fazer isto ou aquilo. Por isso a engenharia, a política e a indústria
são muito naturais aos homens. Mas a metafísica não se interessa, de modo algum, pelos “comos” da vida mas antes
apenas pelos “porquês”, pelas questões que é perfeitamente fácil jamais formular durante uma vida inteira.
Pensar metafisicamente é pensar, sem arbitrariedade nem dogmatismo, nos mais básicos problemas da existência.
Os problemas são básicos no sentido em que são fundamentais e muita coisa depende deles. A religião, por exemplo,
55
não é metafísica; e, entretanto, se a teoria metafísica do materialismo fosse verdadeira, e assim fosse um facto que os
homens não têm alma, então grande parte da religião soçobraria diante desse facto. Também a filosofia moral não é
metafísica e, contudo, se as teorias metafísicas do determinismo ou do fatalismo fossem verdadeiras, então muitos
dos nossos pressupostos tradicionais seriam refutados por essas verdades. Similarmente, a lógica não é metafísica e,
contudo, se se apurasse que, em virtude da natureza do tempo, algumas asserções não são verdadeiras nem falsas, isso
acarretaria sérias implicações para a lógica tradicional.
Isto sugere, contrariamente ao que em geral se supõe, que a metafísica é um alicerce da filosofia e não o seu coroamento. Se for longamente exercido, o pensamento filosófico tende a resolver-se em problemas metafísicos básicos. Por isso o pensamento metafísico é difícil. Com efeito, seria provavelmente correcto afirmar que o fruto do
pensamento metafísico não é o conhecimento, mas a compreensão. As interrogações metafísicas têm respostas e,
entre as várias respostas concorrentes, nem todas poderão ser verdadeiras, por certo. Se um homem defende uma
teoria materialista e outro a nega, então um desses homens está errado; e o mesmo acontece a todas as outras teorias
metafísicas. Contudo, só muito raramente é possível provar e conhecer qual das teorias é a verdadeira. A compreensão, porém — e, por vezes, a parte mais profunda dela — resulta de se ver dificuldades persistentes em opiniões que
frequentemente parecem, com outras bases, ser muito obviamente verdadeiras. É por essa razão que um homem pode
ser um sábio metafísico sem que, não obstante, sustente suas opiniões e juízos em conceitos metafísicos. Tal homem
pode ver tudo o que um dogmático metafísico vê, e pode entender todas as razões para afirmar o que outro homem
afirma com tamanha confiança. Mas, ao invés do outro, também vê algumas razões para duvidar e, assim, ele é, como
Sócrates, o mais sábio, mesmo em sua profissão de ignorância. Advirta-se o leitor, neste particular, de que quando
ouvir um filósofo proclamar qualquer opinião metafísica com grande confiança, ou o ouvir afirmar que determinada
coisa, em metafísica, é óbvia, ou que algum problema metafísico gravita apenas em torno de confusões de conceitos
ou de significados de palavras, então poderá estar inteiramente certo de que esse homem está infinitamente distante
do entendimento filosófico. Suas opiniões parecem isentas de dificuldades apenas porque ele se recusa obstinadamente a ver as dificuldades.
Um problema metafísico é inseparável dos seus dados, pois são estes que, em primeiro lugar, dão origem ao problema. Ora o datum, ou dado, significa literalmente algo que nos é oferecido, posto à nossa disposição. Assim, tomamos como dado de um problema certas convicções elementares do senso comum que todos ou a maioria dos homens estão aptos a sustentar com alguma persuasão íntima, antes da reflexão filosófica, e teriam relutância em abandonar. Não são teorias filosóficas, pois estas são o produto da reflexão filosófica e, usualmente, resultam da tentativa
de conciliar certos dados entre si. São, pelo contrário, pontos de partida para teorias, as coisas por onde se começa,
visto que, para que se consiga alguma coisa, devemos começar por alguma coisa, e não se pode gastar o tempo todo
apenas começando. Observou Aristóteles: “Procurar a prova de assuntos que já possuem evidência mais clara do que
qualquer prova pode fornecer é confundir o melhor com o pior, o plausível com o implausível e o básico com o derivativo”. Exemplos de dados metafísicos são as crenças que todos os homens possuem, independentemente da filosofia, de que existem, de que têm um corpo, de que lhes cabe algumas vezes uma opção entre cursos alternativos de
ação, de que por vezes deliberam sobre tais cursos, de que envelhecem e morrerão algum dia, etc. Um problema
metafísico surge quando se verifica que tais dados não parecem concordar entre si, que têm, aparentemente, implicações inconsistentes entre si. A tarefa, então, é encontrar uma teoria adequada à eliminação desses conflitos.
Talvez convenha observar que os dados, como os considero, não são coisas necessariamente verdadeiras nem evidentes em si. De fato, se o conflito entre certas convicções do senso comum não for tão-só aparente, mas real, então
algumas dessas convicções estão fadadas a ser falsas, embora possam, não obstante, ser tidas na conta de dados até
que sua falsidade se descubra. É isso o que torna emocionante, por vezes, a metafísica; nomeadamente, o fato de
sermos coagidos, algumas vezes, a abandonar certas opiniões que sempre havíamos considerado óbvias. Contudo, a
metafísica tem de começar por alguma coisa e, como não pode começar, obviamente, pelas coisas que já estão provadas, deve começar pelas coisas em que as pessoas acreditam; e a confiança com que uma pessoa sustenta as suas
teorias metafísicas não pode ser maior do que a confiança que deposita nos dados em que aquelas repousam.
Ora, o intelecto do homem não é tão forte quanto a sua vontade, e os homens, geralmente, acreditam no que querem acreditar, particularmente quando essas crenças parecem dar valor a si mesmos e às suas actividades. A sabedoria
não é, pois, o que os homens buscam em primeiro lugar. Procuram, outrossim, uma justificação para aquilo de que
por acaso gostam. Não surpreende, portanto, que os principiantes em filosofia, e mesmo os que já não são principiantes, tenham uma acentuada inclinação para se apegarem a uma teoria que os atraia, em face de dados conflituantes, e
neguem por vezes a veracidade dos dados, apenas por aquela razão. Tal atitude dificilmente se pode considerar propícia à sabedoria. Assim, não é incomum encontrarmos pessoas que, dizem, querem ardentemente acreditar na teoria do
determinismo e que, partindo desse desejo, negam, simplesmente, a verdade de quaisquer dados que com ela colidam.
Os dados, por outras palavras, são meramente ajustados à teoria, em vez de a teoria aos dados. Mas deve-se insistir
ainda que é pelos dados, e não pela teoria, que se terá de começar; pois se não partirmos de pressupostos razoavelmente plausíveis, onde irmos obter a teoria, senão no que os nossos corações desejam? Mais cedo ou mais tarde
poderemos ter de abandonar alguns dos dados do nosso senso comum, mas, ao fazê-lo, será em consideração a certas
outras crenças do senso comum que temos ainda maior relutância em abandonar e não em deferência pelas teorias
filosóficas que nos atraem.
TAYLOR, Richard. O que é a metafísica?
56
Cogito, logo aprendo
1. (UFMG 2004) Leia estes trechos:
TRECHO 1. “Afinal, que é o homem dentro da natureza? Um nada em relação ao infinito; um tudo em relação ao
nada; um meio entre nada e tudo. Infinitamente incapaz de compreender os extremos, tanto o fim das coisas como
seus princípios permanecem invencivelmente ocultos num segredo impenetrável, e é-lhe igualmente impossível ver o
nada de onde saiu e o infinito que o envolve. [...] Eis o nosso estado verdadeiro, que nos torna incapazes de saber com
segurança e de ignorar totalmente. Vagamos em um meio vasto, sempre incertos e flutuantes, empurrados de um
extremo ao outro. [...] É o estado que nos é natural e, no entanto, nenhum será mais contrário à nossa inclinação.”
TRECHO 2. “A grandeza do homem é grande na medida em que ele se conhece miserável. Uma árvore não se
conhece miserável. É, pois, ser miserável conhecer-se miserável, mas é ser grande conhecer que se é miserável.”
[PASCAL, Blaise. Pensamentos.]
Com base na leitura desses trechos e em outras ideias contidas nesta obra de Pascal, REDIJA um texto, desenvolvendo dois aspectos concernentes à ideia de condição humana.
2. Leia o texto a seguir: “A razão humana, num determinado domínio dos seus conhecimentos, possui o singular
destino de se ver atormentada por questões, que não pode evitar, pois lhe são impostas pela sua natureza, mas às quais
também não pode dar respostas por ultrapassarem completamente as suas possibilidades.” [KANT, Immanuel. Crítica da
Razão Pura.]
Com base no texto e nos conhecimentos sobre Kant, o domínio destas intermináveis disputas chama-se
a) experiência.
b) natureza.
c) entendimento.
d) metafísica.
e) sensibilidade.
3. Leia o texto a seguir.
“Como o costume nos determina a transferir o passado para o futuro em todas as nossas inferências, esperamos
— se o passado tem sido inteiramente regular e uniforme — o mesmo evento com a máxima segurança e não toleramos qualquer suposição contrária. Mas, se temos encontrado que diferentes efeitos acompanham causas que em
aparência são exatamente similares, todos estes efeitos variados devem apresentar-se ao espírito ao transferir o passado para o futuro, e devemos considerá-los quando determinamos a probabilidade do evento.” [HUME, David. Investigações acerca do entendimento humano.]
Com base no texto e nos conhecimentos sobre Hume, é correto afirmar:
a) Hume procura demonstrar o cálculo matemático de probabilidades.
b) Hume procura mostrar o mecanismo psicológico pelo qual a crença se fixa na imaginação.
c) Para Hume, há uma conexão necessária entre causa e efeito.
d) Para Hume, as inferências causais são a priori.
e) Hume procura mostrar que crença e ficção produzem o mesmo efeito na imaginação humana.
57
Capítulo VIII
ÉTICA
ITURRUSGARAI, Adão. “Mundo monstro”. In: Folha de S. Paulo (1 de abril de 2009).
ser humano vive em sociedade e, por isso, coexiste com outros seres humanos; consequentemente, cabe-lhe
refletir e (ao menos tentar) responder à seguinte interrogação: como devo agir diante dos outros? Ainda que
seja uma questão fácil de ser formulada, é difícil de ser respondida. Essa é, pois, a questão fundamental da
Ética (ou Filosofia moral), subdivisão da Filosofia que examina os problemas relacionados com o que devemos valorizar e com a maneira como devemos viver. Vale dizer que, como doutrina filosófica, a Ética é
basicamente especulativa (ou seja, tem caráter teórico, não prático). Não obstante, deve-se distinguir o que é Ética e o
que é Moral. A Filosofia define a Moral como um conjunto de princípios, normas, valores e costumes que orientam o
comportamento dos indivíduos em um meio social, sendo, portanto, normativa; já a Ética é teórica, tendo como função apontar, explicar e justificar os costumes próprios de uma determinada sociedade, além de auxiliar na solução dos
dilemas que podem surgir.
“Célula-tronco sem embrião é aprovada em teste final”. “Coreia do Norte zomba de Hillary em nota oficial”. “Índios invadem prédio da Funai e fazem 6 reféns”. “Presa quadrilha acusada de pelo menos 30 roubos de carga”. “Falsários ‘roubam’ nomes de famosos para espalhar boatos em perfis do Twitter” (Folha de S. Paulo, 24 de julho de
2009). Manchetes como essas (ou temas como a eutanásia, o aborto, o meio ambiente etc.), sejam elas mais ou menos
dramáticas, mais ou menos sérias, mais ou menos importantes, surgem diariamente no mundo e, sem muito esforço,
em nossas vidas. Em nós surgem, por conseguinte, dúvidas relacionadas a qual decisão tomar, a qual opinião ter, a
qual atitude seguir diante de certos acontecimentos. Tais episódios põem à prova nossa consciência moral e fazem
com que manifestemos nosso senso moral, exigindo que resolvamos o que fazer, que justifiquemos para nós e para os
outros a razão (ou as razões) de nossas decisões, assim assumindo também as consequências delas — já que somos
responsáveis por nossas escolhas.
Os exemplos citados apontam que “senso moral” e “consciência moral” referem-se a valores (tais como a justiça,
a honradez, a integridade etc.), bem como a sentimentos provocados por esses valores (vergonha, raiva, remorso,
dúvida, amor, receio etc.) e também a decisões que conduzem a atos com resultados que dizem respeito a nós e aos
outros. Apesar dos conteúdos dos valores variarem, pode-se notar que referem-se a um valor mais profundo, por
vezes implícito: o bom ou o bem.
O
Os sentimentos e as ações, nascidos de uma opção entre o bom e o mau ou entre o bem e o mal, também estão referidos a algo
mais profundo e subentendido: nosso desejo de afastar a dor e o sofrimento e de alcançar a felicidade, seja por ficarmos contentes conosco mesmos, seja por recebermos a aprovação dos outros. O senso e a consciência moral dizem respeito a valores,
sentimentos, intenções, decisões e ações referidos ao bem e ao mal e ao desejo de felicidade. Dizem respeito às relações que
mantemos com os outros e, portanto, nascem e existem como parte de nossa vida intersubjetiva. [CHAUÍ, Marilena. Convite à filosofia.]
Em meio a qualquer situação podemos, enquanto seres racionais, enunciar juízos — ou seja, podemos contar com
a “faculdade intelectual que permite julgar, avaliar com correção, discernimento, bom senso; capacidade de ponderação, siso; equilíbrio mental” [HOUAISS, Antonio. Dicionário eletrônico Houaiss da língua portuguesa.]. Tais juízos
podem ser de duas espécies: “juízo de fato” e “juízo de valor”. Os juízos de fato exprimem o que as coisas são, como
são e por que são; os juízos de valor manifestam avaliações sobre pessoas, experiências, acontecimentos, situações,
decisões (e são, por isso, comuns na moral, nas artes, na política, na religião). Se alguém disser, por exemplo, “hoje é
sábado e há aula de Filosofia”, estará pronunciado um fato constatado, e o juízo é, portanto, um juízo de fato; se,
contudo, esse mesmo alguém disser “aula de Filosofia aos sábados é algo que me aborrece”, ele (ou ela) estará interpretando e analisando o mesmo fato; há, nesse caso, a declaração de um juízo de valor.
Se levarmos essa ideia de juízo para a Ética, veremos que os chamados “juízos éticos de valor” são também normativos, ou melhor, exprimem normas que determinam o “dever ser” de nossas atitudes, de nossos sentimentos, de
nossos comportamentos, pois se tratam de juízos que enunciam obrigações e avaliam ações e finalidades de acordo
com o critério do adequado/inadequado, do correto/incorreto. Assim, temos que os juízos éticos de valor nos apontam
o que é o mal, o bem, a felicidade etc.; e os juízos éticos normativos (que explicitam normas, isto é, regras) apontam
quais atos, intenções e comportamentos devemos apresentar para alcançarmos o bem e a felicidade. Vale lembrar que
os juízos éticos normativos também podem apontar sentimentos, atos, intenções e comportamentos que sejam condenáveis, incorretos na perspectiva moral.
Com isso, fica claro que o senso moral e a consciência moral são próprios da vida cultural, pois é a vida cultural
que determina para seus membros os valores negativos ou positivos a serem respeitados ou evitados.
58
Qual a origem da diferença entre os dois tipos de juízos? A diferença entre a Natureza e a Cultura. A primeira, como vimos, é
constituída por estruturas e processos necessários, que existem em si e por si mesmos, independentemente de nós [...]. Por sua
vez, a Cultura nasce da maneira como os seres humanos interpretam a si mesmos e suas relações com a Natureza, acrescentando-lhe sentidos novos, intervindo nela, alterando-a através do trabalho e da técnica, dando-lhe valores. [Idem.]
É importante notar que às vezes não distinguimos a origem cultural do senso moral, da consciência moral e dos
valores éticos porque somos educados e formados neles e para eles, de maneira que, para nós (que estamos imersos
nesse contexto cultural), nos parecem naturais, existentes em si e por si mesmos. Isso se deve à necessidade de se
garantir a conservação dos padrões morais ao longo do tempo, sua continuação de geração a geração e, para isso, as
sociedades acabam por torná-los naturais. Todavia, essa naturalização da existência moral oculta uma característica
importante da Ética: ela é uma invenção histórico-cultural.
LAERTE. “Piratas do Tietê”. In: Folha de S. Paulo (27 de março de 2009).
Vejamos, agora, os constituintes do campo ético.
A conduta ética, para que exista, necessita da existência daquele que conhece e saiba diferenciar certo e errado,
permitido e proibido, bem e mal; em outras palavras, a conduta ética precisa do denominado “agente consciente”.
Esse agente, por possuir consciência moral, é capaz de não apenas conhecer tais diferenças, mas sobretudo reconhece-se como apto para julgar o valor das condutas e dos atos, seus e dos demais, e de agir em harmonia com os valores
morais da sociedade da qual faz parte. O agente consciente tem, por isso, responsabilidade em se tratando de suas
ações e de seus sentimentos, bem como é responsável pelas consequências do que sente e faz. Nesse ínterim, a consciência moral manifesta-se na competência para decidir, entre várias (ou, no mínimo, duas) alternativas possíveis,
uma delas antes de lançar-se na ação.
O agente consciente, em posse da consciência moral, tem competência para avaliar as exigências feitas pela situação, as motivações pessoais, as consequências para si e para os demais, a obrigação de respeitar o que está estabelecido ou mesmo de transgredi-lo (na situação em que o estabelecido for injusto ou imoral). Pode-se ver que o campo
ético é, assim, composto pelos valores e pelas obrigações que constituem o conteúdo das condutas morais: as virtudes.
Tais virtudes são realizadas pelo “sujeito moral”, ou seja, a pessoa, que só pode existir enquanto sujeito moral se
preencher as seguintes condições:
- ser consciente de si e dos outros, isto é, ser capaz de reflexão e de reconhecer a existência dos outros como sujeitos éticos iguais a ele;
- ser dotado de vontade, isto é, de capacidade para controlar e orientar desejos, impulsos, tendências, sentimentos (para que
estejam em conformidade com a consciência) e de capacidade para deliberar e decidir entre várias alternativas possíveis;
- ser responsável, isto é, reconhecer-se como autor da ação, avaliar os efeitos e consequências dela sobre si e sobre os outros,
assumi-la bem como às suas consequências, respondendo por elas;
- ser livre, isto é, ser capaz de oferecer-se como causa interna de seus sentimentos, atitudes e ações, por não estar submetido a
poderes externos que o forcem e o constranjam a sentir, a querer e a fazer alguma coisa. A liberdade não é tanto o poder para
escolher entre vários possíveis, mas o poder para autodeterminar-se, dando a si mesmo as regras de conduta. [Idem.]
Levando-se em conta os valores, a Ética traduz a maneira pela qual a sociedade define, para si mesma, o que julga ser o mal, a violência, o vício, o crime e, por outro lado, o que considera ser a virtude e o bem. A Ética, por não ser
indiferente às condições políticas, históricas, econômicas e culturais da ação moral, está em relação com o tempo e
com a História; isso significa que ela se transforma com o passar do tempo para responder às novas exigências da
sociedade e da Cultura, pois, além de sermos seres históricos e culturais e nossas ações se desenvolverem no tempo,
não nos esqueçamos de que, como já salientado, a Ética é uma invenção, e não algo acabado, pronto e enlatado para
consumo em qualquer época e lugar. Nesse sentido, cada sociedade institui uma moral; em outras palavras, define
valores relacionados com o bem e o mal, o permitido e o proibido, válidos para todos os seus membros.
Em meio ao que esclarecemos até aqui, surge, porém, uma pergunta: se somos seres racionais e teoricamente livres, por que motivo os valores, os fins e as leis morais não são espontâneos, instintivos em nós, mas precisam assumir a forma de um dever? Immanuel Kant respondeu a essa questão da seguinte maneira:
Porque não somos seres morais apenas. Também somos seres naturais, submetidos à causalidade necessária da Natureza.
Nosso corpo e nossa psique são feitos de apetites, impulsos, desejos e paixões. Nossos sentimentos, nossas emoções e nossos
comportamentos são a parte da Natureza em nós, exercendo domínio sobre nós, submetendo-se à causalidade natural inexorável. Quem se submete a eles não pode possuir a autonomia ética. A Natureza nos impele a agir por interesse. Este é a forma
natural do egoísmo que nos leva a usar coisas e pessoas como meios e instrumentos para o que desejamos. Além disso, o inte-
59
resse nos faz viver na ilusão de que somos livres e racionais por realizarmos ações que julgamos terem sido decididas livremente por nós, quando, na verdade, são um impulso cego determinado pela causalidade natural. [Idem.]
Assim, agir por interesse é atuar a partir de motivações físicas, psíquicas, vitais. Se tivermos em mente que os
impulsos, os instintos, os desejos, enfim, os comportamentos naturais costumam ser muito mais intensos, fortes (e
mesmo violentos) em relação à razão, a razão prática e a liberdade verdadeira precisam reprimir nossa parte natural e
nos impor nosso ser moral. Com isso, para sermos livres é necessário que sejamos obrigados pelo dever de sermos
livres. Afirma Kant que o dever não se apresenta por meio de um conjunto de conteúdos estáveis, que o dever não é
um catálogo fixo de virtudes ou uma lista de “faça isso” ou “não faça aquilo”; o dever é uma forma que deve valer
para toda e qualquer ação moral, e essa forma não é indicativa, mas imperativa (“tu deves”).
Não admitindo hipóteses (“se... então”) nem condições que o fariam valer em algumas situações mas não em outras, aquilo que é imperativo vale, sem exceções, para todas e quaisquer circunstâncias de todas as ações morais; o
dever torna-se, assim, um imperativo categórico, ou seja, ordena incondicionalmente. Tal imperativo categórico não
se trata de uma motivação psicológica, mas de uma lei moral interior que pode ser anunciada com uma fórmula geral:
“age em conformidade apenas com a máxima que possas querer que se torne uma lei universal”. Tal fórmula permitiu
a Kant deduzir três máximas morais que revelam a incondicionalidade dos atos realizados por dever.
A primeira máxima (“age como se a máxima de tua ação devesse ser erigida por tua vontade em lei universal da
Natureza”) afirma a universalidade da conduta ética, ou seja, aquilo que todo e qualquer ser humano, desde que racional, deve fazer como se fosse uma lei inquestionável, apropriada para todos, em todo tempo e lugar. A segunda
máxima (“age de tal maneira que trates a humanidade, tanto na tua pessoa como na pessoa de outrem, sempre como
um fim e nunca como um meio”) afirma a dignidade dos seres humanos enquanto pessoas e, desse modo, a exigência
de que sejam tratados como fim da ação e nunca como meio ou como instrumento para interesses alheios. A terceira
máxima (“age como se a máxima de tua ação devesse servir de lei universal para todos os seres racionais”) afirma
que a vontade que age por dever estabelece um reino humano de seres morais porque racionais e, portanto, portadores
de uma vontade legisladora livre.
O imperativo categórico não enuncia o conteúdo particular de uma ação, mas a forma geral das ações morais. As máximas
deixam clara a interiorização do dever, pois este nasce da razão e da vontade legisladora universal do agente moral. O acordo
entre vontade e dever é o que Kant designa como vontade boa que quer o bem. O motivo moral da vontade boa é agir por dever. O móvel moral da vontade boa é o respeito pelo dever, produzido em nós pela razão. Obediência à lei moral, respeito pelo
dever e pelos outros constituem a bondade da vontade ética. O imperativo categórico não nos diz para sermos honestos, oferecendo-nos a essência da honestidade; nem para sermos justos, verazes, generosos ou corajosos a partir da definição da essência da justiça, da verdade, da generosidade ou da coragem. Não nos diz para praticarmos esta ou aquela ação determinada,
mas nos diz para sermos éticos cumprindo o dever (as três máximas morais). É este que determina por que uma ação moral deverá ser sempre honesta, justa, veraz, generosa ou corajosa. Ao agir, devemos indagar se nossa ação está em conformidade
com os fins morais, isto é, com as máximas do dever. [Idem.]
Visto isso, podemos questionar, por exemplo, por que mentir é imoral? Diria Kant: por que o mentiroso infringe
as três máximas morais: quando mente, não respeita em sua pessoa e na do outro a consciência, a racionalidade e a
liberdade (em suma: a humanidade), comete uma violência escondendo de um outro ser humano uma informação
verdadeira; além disso, não respeita a segunda máxima, pois se a mentira se tornasse uma ação universal, o gênero
humano deveria abandonar a razão e o conhecimento, a capacidade para decidir e escolher, vivendo no erro e na
ilusão. E assim podemos trazer outras questões: por que um político que rouba dinheiro público é imoral? Por que um
estuprador é imoral? Porque, como vimos, tanto o político corrupto quanto o estuprador transgridem as três máximas
kantianas.
Concluindo, e resumindo o que vimos até aqui, a Ética é uma característica inerente a toda e qualquer ação humana e, por este motivo, é um item fundamental na produção e na manutenção da realidade social. Relacionada à
liberdade, às escolhas, ao desejo de realizar a vida mantendo com os demais relações justas e aceitáveis, a Ética funda-se nas ideias de bem e de virtude enquanto valores buscados pelos homens a fim de alcançarem uma existência
social ordenada e o menos negativa possível.
Leituras complementares
Alguns moralistas recomendam que a vida “autêntica” não significa simplesmente lembrarmo-nos de que um dia
morreremos, mas de alguma maneira viver permanentemente com a consciência desse facto, “viver-para-a-morte”. O
poeta John Donne tinha mesmo o seu próprio retrato em que aparecia pintado de mortalha, antecipando cheio de
esperança a maneira como iria encarar o Juízo Final. Todavia, a maioria das pessoas não acha a preocupação de
Donne particularmente saudável. De facto, tal estado de espírito prevalece apenas em condições de instabilidade
social e impotência política, correspondendo a uma tendência para o pessimismo e o suicídio entre a intelligentsia.
Ora, não é fácil argumentar com um estado de espírito. Se o poeta se sente atraído pela serenidade da morte, ou nauseado pela desordem humana, talvez precise de uma mudança de governo, ou de um tónico, ou de umas férias, de
preferência a um argumento.
A obsessão pela morte pode correr o perigo de inconsistência. É inconsistente a veemência de que a morte é uma
coisa boa, e até um luxo em si, e que ao mesmo tempo o que torna a vida absurda e ilusória é que ela tem um fim.
Mas por que razão é isto um problema se a morte é estimável?
Apesar de argumentarem que a morte não é para ser temida, os estoicos não promoveram qualquer preocupação
mórbida com a morte. Em vez disso, tal como o uso moderno do seu nome sugere, a sua mensagem era de força de
60
espírito e resignação perante a morte, ou de fatalismo em face do inevitável desenrolar dos acontecimentos. A atitude
estóica está enraizada numa das conotações populares do próprio termo “filosofia”, como se pode ver no comentário
de uma pessoa sobre a desgraça de outra: “encara as coisas filosoficamente — simplesmente não penses nisso.” P. G.
Wodehouse provavelmente tem a última palavra sobre este aspecto dos Estóicos. Jeeves consola Bertie:
“Não sei se posso chamar a sua atenção para uma observação do imperador Marco Aurélio”. Disse ele: “Acontece-te o que
quer que seja? Isso é bom. Faz parte do destino de Universo determinado para ti desde o início. Tudo o que te acontece faz parte
da grande rede”.
Respirei com um estertor.
“Ele disse realmente isso?”
“Sim, Sir.”
“Bem, podes dizer-lhe da minha parte que ele é um idiota. As minhas malas estão prontas?
Como Bertie observou judiciosamente mais tarde: “Duvido, como questão de facto, se o que Marco Aurélio afirmou poderá alguma vez dar moral às tropas quando elas se deparam com um destino terrível: terás de querer esperar
que a agonia se abata sobre ti.”
Os filósofos e poetas que tentam reconciliar-nos com a morte não o fazem habitualmente com argumentos tão incisivos como os Estóicos, nem com o fatalismo Estóico, mas lamentando-se da própria vida. Já todos ouvimos o
funesto refrão: o mundo não é senão discórdia, instabilidade e desordem. A vida é tédio e fardo. As suas esperanças
ilusórias, os seus prazeres insignificantes. O desejo é infinito e insaciável; a satisfação não traz apaziguamento. Carpe
Diem (aproveita o dia) — mas não podes aproveitar o dia porque o dia se desvanece enquanto tentas fazê-lo. Tudo
mergulha no abismo, nada é estável; palácios e impérios desfazem-se em pó; friamente, o universo move-se e tudo
será no fim esquecido.
Vaidade das vaidades, diz o pregador, vaidade das vaidades, tudo é vaidade. Que benefício retira o homem de todo o seu
labor na terra?
Acima de tudo, os mortos têm de ser invejados. A morte é preciosa. Melhor seria não ter nascido, mas uma vez nascido, o
melhor será morrer depressa.
O perigo aqui consiste naquilo a que o filósofo George Berkeley (1685-1753) chamou o vício da abstracção, “a
rede subtil e elegante de ideias abstractas que deixam os espíritos dos homens tão miseravelmente perplexos e enredados”. É mais fácil lamentar a natureza irrisória do desejo e as suas inconsistências se não nos centrarmos em nada,
mantendo a discussão em termos abstractos. Desse modo, parece desolador se a satisfação do desejo é efémera e o
próprio desejo é instável e susceptível de dar origem apenas a mais insatisfação. Mas será que isto é de lamentar?
Pensando em termos concretos, supõe que nos apetece um bom jantar e que ele nos deu prazer. Deverá envenenar o
prazer que tivemos a reflexão de que o prazer é efémero (o prazer deste jantar não subsistirá para sempre), de que o
desejo de um bom jantar não perdura (mais tarde não sentiremos fome) ou é apenas temporariamente satisfeito (iremos querer jantar amanhã outra vez)? A verdade é que a vida não se tornaria melhor se quiséssemos sempre um
jantar, Ou se, tendo jantado, não quiséssemos mais nenhum jantar, ou se aquele jantar bastasse para toda a vida.
Nenhuma destas coisas parece remotamente desejável; logo, porquê lamentarmo-nos se as coisas não são assim?
Se a disposição pessimista abandonar a abstracção de certas ideias, tenderá a concentrar-se em desejos problemáticos, tais como o desejo de riqueza, ou o desejo erótico. É fácil argumentar que estes desejos são intrinsecamente
insaciáveis, pelo menos para algumas pessoas durante algum tempo. Da aquisição de riqueza resulta frequentemente
ou a exigência de mais riqueza, ou a incapacidade de gozar o que se tem. O nosso bem-estar pode ser destruído pela
pobreza, mas a mais breve consideração das vidas dos ricos não sugere que o bem-estar aumenta indefinidamente
com mais riqueza. Muitas pessoas são hoje mais ricas do que qualquer pessoa alguma vez foi, mas serão mais felizes?
Estatísticas sociais relevantes, como taxas de suicídio, não o sugerem. Os fechados e vigiados guetos dos ricos, como
o American Governor's Club, dificilmente permitem testemunhar vidas felizes e invejáveis. E, seguindo Veblen,
podemos esperar que o aumento de riqueza simplesmente fará subir a fronteira a partir da qual a vaidade exigirá que
os ricos se distingam. Esta é uma das coisas desencorajadoras acerca da desencorajadora ciência económica.
O desejo erótico, outra carta de trunfo dos pessimistas, é notoriamente insatisfeito e incerto, além de que oferece
apenas realização parcial. Provavelmente nunca chegaremos a possuir tanto outra pessoa quanto realmente desejaríamos. A arte não tem tido dificuldade em ligar o desejo erótico ao desejo de morte e aniquilação. O próprio amor é
uma espécie de morte — o amante é penetrado ou atacado. Nesta tradição, os delírios do amor, especialmente o orgasmo (em francês une petite mort, “uma pequena morte”), são símbolos da morte real. Argumenta-se que as mortes
em Tristão e Isolda ou em Romeu e Julieta indicam o desejo oculto dos amantes de extinção conjunta. Na arte é
extraordinariamente perigoso ser uma mulher apaixonada, como nos lembra a interminável procissão de Ofélias,
Violetas, Toscas e Mimis.
É bastante deprimente supor que mesmo Eros (desejo) é contaminado por Thanatos (morte). Mas talvez o vício da
abstracção esteja em funcionamento mais uma vez. Se prestarmos atenção a algumas obras de arte, concluímos que o
desejo erótico tem a morte no seu centro. Não paramos para reflectir que era o artista que sentia a necessidade do
tema dos amantes fatais, suprimindo a referência a quaisquer prazeres e alegrias comuns. O artista tem boas razões
para construir Jack e Jill como Romeu e Julieta. Mas provavelmente Jack e Jill são em si bastante mais alegres. A
tragédia não é inevitável nem habitualmente desejada.
Do mesmo modo, deslizamos para a abstracção quando perguntamos se a vida em bloco, como se de uma massa
informe se tratasse, “tem um sentido”, talvez imaginando algum observador externo, o qual até podemos ser nós
próprios para além do túmulo, olhando para trás. Preocupa-nos que o observador domine com o seu olhar a totalidade
do espaço e do tempo e que a nossa vida encolha até não ser nada, apenas um insignificante e infinitesimal fragmento
do todo. “O silêncio desses espaços infinitos aterroriza-me”, disse Blaise Pascal (1623-62).
Mas Frank Ramsey (1903-30), o filósofo de Cambridge, respondeu:
Onde eu pareço divergir de alguns dos meus amigos é em conceder pouca importância ao tamanho das coisas. Não me sinto
nem um pouco humilhado perante a vastidão dos céus. As estrelas podem ser enormes, mas não podem pensar ou amar; e estas
61
são qualidades que me impressionam bastante mais do que o tamanho. Não me acrescenta qualquer importância o facto de eu
pesar 108 quilos.
A minha descrição do mundo é elaborada em perspectiva e não como um modelo em escala. O solo é ocupado por seres humanos e as estrelas são todas tão pequenas como moedas de três dinheiros.
Quando perguntamos se a vida tem sentido, a primeira questão terá de ser: para quem? Para um observador com a
totalidade do espaço e do tempo no seu olhar, nada a uma escala humana terá sentido (é difícil imaginar como é que
ela pode ser de todo visível — há uma medonha quantidade de espaço lá longe). Mas por que razão a nossa insignificância no interior dessa perspectiva deverá esmagar-nos? Suponha-se em vez disso que temos em mente uma audiência que desceu à nossa medida. Alguém que dedica a sua vida a algum objectivo, como a cura do cancro, pode perguntar se a sua vida tem sentido, e a sua preocupação será se tem sentido para aqueles para quem trabalha. Se o seu
trabalho for bem sucedido, ou se a geração seguinte o tiver em conta, a sua vida terá tido sentido. Para algumas pessoas, é dolorosa a ideia de que o seu trabalho possa falhar e não deixar memória. Outros arranjam-se muito bem com
isso: afinal, muito, muito poucos mesmo, deixam atrás de si realizações que suscitam a admiração contínua da próxima geração, para não falar das gerações que vierem a seguir. Isto é tristemente verdade mesmo em departamentos de
filosofia.
Talvez nos vejamos a nós próprios na posição de juiz: cada um de nós pergunta se a vida tem sentido para si, aqui
e agora. E nesse caso a resposta depende. A vida é uma corrente de acontecimentos vividos no interior da qual há
frequentemente bastante sentido — para nós próprios e os que nos rodeiam. O arquitecto Mies van der Rohe disse
que Deus está nos pormenores e o mesmo é verdade acerca do sentido da vida para nós, aqui e agora. O sorriso do
filho significa tudo para a mãe, a carícia significa beatitude para o amante, a mudança de frase significa felicidade
para o escritor. O sentido vem da entrega e do prazer, da corrente de pormenores que são importantes para nós. O
problema que há com a vida é então o de ela ter demasiado sentido. Todavia, se o estado de espírito é outro, tudo é
penoso. Como Hamlet, estamos determinados a esquivar-nos do carnaval humano, não vendo senão a caveira debaixo
da pele. É triste quando nos tornamos seres humanos assim e mais uma vez precisamos de um tónico de preferência a
um argumento. Ou talvez neste caso, o único bom argumento, numa famosa frase de Hume, é que não há nenhuma
maneira de fazer de ti uma pessoa útil ou agradável para ti próprio ou para os outros.
BLACKBURN, Simon. Desejo e sentido da vida.
φ
O objectivo destas páginas é duplo. Por um lado, mostrar a plausibilidade da seguinte hipótese: quando o sentido
da vida é encarado como um problema pessoal, é mal compreendido. Não disponho de estudos científicos empíricos
que possam sustentar esta hipótese. Por isso defendo apenas que é uma hipótese plausível; talvez as coisas se passem
como esta hipótese diz que são, ou talvez não.
Por outro lado, quero mostrar que, porque o problema pessoal do sentido da vida é mal compreendido, nenhuma
resposta razoável que possamos dar é satisfatória. Mas isto acontece não porque as respostas são más, mas porque o
problema pessoal do sentido da vida é um falso problema.
Esta reflexão é um complemento ao artigo “O Sentido da Vida”, publicado na Intelecto. Nesse artigo, passo em
revista algumas das respostas religiosas ao problema do sentido da vida e defendo que a vida faz sentido na exacta
medida em que houver valores objectivos. Verifiquei, todavia, que este artigo era demasiado “académico”, no sentido
em que não era o tipo de artigo que pudesse responder às ansiedades de quem se defronta realmente na sua vida com
o problema do sentido da vida. Acho esta situação irónica e típica de um certo tipo de academismo estéril, do qual eu
no entanto sempre me esforcei por me afastar.
Típica porque a regra de ouro do academismo estéril é nunca tratar de tema algum que possa ser compreendido
pelo público culto que ainda não se deixou hipnotizar pela prosa soporífera dos universitários; e caso esta regra seja
quebrada, é então necessário tratar do tema de tal modo que o público culto não consiga entender, nem encontre as
respostas de que estava à espera de encontrar. E irónica porque o tema do sentido da vida é visto com maus olhos
precisamente por ser demasiado popular, algo que toda a gente pensa constituir a preocupação principal da filosofia.
E ali estava eu a escrever um texto que não agradava nem a gregos nem a troianos: nem aos académicos por ser demasiado popular, nem ao público em geral, a quem a Intelecto se destina, por não ser suficientemente popular.
Há boas razões para a melhor tradição filosófica procurar evitar a imagem do filósofo como um guru. Os gurus
são figuras tudo menos filosóficas, pois as suas sentenças oraculares não são para ser discutidas e avaliadas, mas
antes seguidas cegamente pelos acólitos. Mas penso que é preciso mostrar que é possível falar ao público culto e
responder às suas preocupações sem cair no dogmatismo dos gurus. Pessoas como Bertrand Russell fizeram-no, de
forma admirável — valendo-lhe até um prémio Nobel — e, mais recentemente, Peter Singer fê-lo também, na obra
“How Are We to Live?”. Mas penso que é preciso insistir.
Penso que é preciso insistir em grande parte por causa da infeliz divisão entre duas formas radicalmente diferentes de fazer e ensinar filosofia: a filosofia analítica e a filosofia continental. Demasiadas vezes as pessoas pensam,
erradamente, que a filosofia analítica está muito mais do lado do academismo estéril — “escolástica” é a palavra que
se ouve e que já me deitaram à cara tantas vezes — do que a filosofia continental. Penso que isto é falso. A ilusão,
todavia, é compreensível. Os textos iniciáticos de Heidegger, por exemplo, ou os textos trágicos de Kierkegaard ou os
textos panfletários e angustiados de Nietzsche, podem responder a algumas das inquietações mais imediatas do grande público. Mas este início promissor — promissor para o grande público que espera resposta às suas inquietações —
é rapidamente atraiçoado, por dois motivos.
Em primeiro lugar, os textos destes autores são extraordinariamente opacos, difíceis de compreender e muito situados. Heidegger é um representante típico do académico que escreve para outros académicos. Nietzsche e Kierkegaard, não — mas escrevem com a cabeça de tal modo metida no seu próprio tempo e nas suas próprias preocupações
62
pessoais, que sem uma preparação adequada, o leitor comum pouco ou nada compreende destes escritores, apesar de
ficar com a sensação de que eram pessoas muito angustiadas, figuras trágicas com um interesse humano inegável.
Em segundo lugar, o trabalho académico que se exige aos estudantes e que os académicos realizam com base nestes autores é regra geral impenetrável ao grande público. E, regra geral, o batalhão de intérpretes e exegetas da obra
destes grandes filósofos pouco mais fazem do que repetir o que eles disseram por outras palavras ou dissecar os
pormenores mais irrelevantes aos olhos do grande público, deixando sempre para depois as ideias centrais que são tão
apelativas nestes autores. É um pouco o mesmo do que acontece com o estudo de “Os Maias” de Eça de Queirós
entre os académicos e na escola secundária: à força de tornarem sério e académico o estudo da obra, acabam por
estripá-la do seu interesse principal, domesticando uma das mais poderosas e bem dirigidas críticas à sociedade portuguesa — críticas que infelizmente continuam actuais.
Penso que há uma alternativa ao guru, e ao texto trágico e grandioso, que apresenta o seu próprio autor como uma
figura teatral. A alternativa é responder com clareza e frontalidade às preocupações do grande público e mostrar
como o pensamento sistemático e consequente nos pode ajudar a compreender melhor o que é para nós importante
compreender. E procurar oferecer o resultado do nosso melhor esforço de compreensão ao grande público é justo;
afinal, é o grande público que nos paga o ordenado. [...]
O problema intuitivo do sentido da vida é um problema pessoal. Não é um problema parecido com a questão de
saber se há vida noutros planetas, ou com a questão de saber qual é a composição da atmosfera de Júpiter. O que se
procura não é realmente uma resposta ao problema do sentido da vida humana em geral, mas sim ao problema do
sentido da minha vida. É o tipo de inquietação que me assaltava quando eu tinha 15 anos, e que inquieta muitos jovens; ou que inquieta pessoas que enfrentam problemas difíceis na sua vida de adultos. Algumas pessoas que passaram anos a ganhar cada vez mais dinheiro parece terem a tendência para se preocupar com o sentido da vida delas
quando os negócios correm mal. Outras, quando morre um ente querido, ou quando descobrem que têm uma doença
terminal. Não tenho qualquer base científica para sustentar estas afirmações; provavelmente, algumas das pessoas que
se preocupam com o problema, terão um interesse genuíno no verdadeiro problema, e não um interesse meramente
interesseiro no sentido da sua própria vida. Mas também não penso que estas ideias, que tenho em resultado da minha
observação assistemática, sejam completamente descabidas.
Quando olhamos para o problema do sentido da vida como um problema pessoal, é difícil ter a lucidez necessária
para compreendermos o problema geral. O que queremos é responder à angústia que estamos a viver. Uma pessoa
nestas circunstâncias dificilmente precisa de ler o artigo que publiquei na Intelecto, ou uma tentativa académica para
tratar o tema. O que uma pessoa nestas circunstâncias precisa é de ser apoiada e encorajada; precisa de amizade e de
um olhar de compreensão, e não de uma lição de filosofia. Uma pessoa nestas circunstâncias não compreende o verdadeiro problema do sentido da vida — esta é a minha convicção. As razões pelas quais eu penso isso são as que
desenvolvo de seguida.
Se eu disser a alguém muito preocupado com o sentido da vida que uma vida que acrescente valor ao mundo faz
sentido, desde que os valores sejam objectivos, esta resposta é insatisfatória. Essa pessoa quer uma resposta para a
vida dela e uma resposta que ela possa compreender. A distinção entre valor objectivo e subjectivo de pouco lhe vale,
se nem sequer compreende bem a distinção. A pessoa quer uma resposta pessoal a um problema pessoal — e não uma
resposta universal que por inerência devia responder também ao seu problema.
É por este motivo que as respostas religiosas são em geral mais apelativas. Porque nas respostas religiosas a própria pessoa é importante; há um Deus que a vê e a escolhe, que lhe concede a vida eterna e a bem-aventurança, e não
apenas um conceito impessoal de valor objectivo. Claro que se perguntarmos a essa pessoa, entretanto convertida, por
que razão há-de Deus dar sentido à nossa vida, já que também isso não se compreende bem, obtemos uma resposta
evasiva: Deus é o mistério, é o incompreensível, nós somos demasiado limitados para compreender os seus desígnios
e a sua natureza. De facto, parece que somos até demasiado limitados, pelo menos alguns de nós, para compreendermos um artigo da Intelecto sobre o sentido da vida, quanto mais Deus. E é agora que a questão interessante se levanta: incompreensão por incompreensão, por que razão é a resposta religiosa melhor do que a filosófica?
Quero deixar para já esta pergunta por responder. Vejamos outra rota de incompreensão e de insatisfação. A resposta ao sentido da vida que evoca valores objectivos é realmente sofisticada e compreende-se que não seja por isso
muito apelativa, apesar de também a resposta religiosa ser incompreensível. Mas também uma resposta como a de
Peter Singer — que nem sequer acredita na objectividade do valor — é incapaz de satisfazer o nosso interlocutor
angustiado com o sentido da vida. A resposta de Singer (e a minha própria, mas a um nível mais térreo e menos abstracto) consiste em dizer a essa pessoa para olhar para o mundo à sua volta com olhos de ver. E que medite no sofrimento dos que morrem à fome, dos que labutam diariamente para sobreviver, dos que sofrem as mais vis injustiças,
do que são estropiados, privados dos seus direitos, etc. Mas quando se diz que uma vida terá sentido desde que possa
servir para trazer algum bem a este mundo, desde que possa servir para ajudar alguns dos que estão em circunstâncias
muito piores do que nós — quando se diz isto, o nosso interlocutor não se deixa impressionar. Tudo isso é verdade,
há imenso sofrimento no mundo, mas a regra básica da psicologia humana é esta: um ardor no meu dedo mindinho é
mais importante para mim do que um milhar de pessoas a morrer à fome a 5 km de distância. Uma inquietação na
minha alminha é mais importante do que as pessoas todas que estão à minha volta e que eu podia ajudar de várias
maneiras. É assim que somos. O que não quer dizer que tenhamos de ser assim.
De modo que esta resposta, a um nível mais térreo, também não satisfaz nada o nosso interlocutor angustiado
com o sentido da sua vida, com as suas dores de alma. O que ele quer é o impossível. Quer uma resposta simples e
rápida, que dê sentido universal à sua vida, mas que ao mesmo tempo lhe fale a ele em particular e não a todos os
seres humanos em geral e — sobretudo — não quer dúvidas nem ter de fazer grande coisa. Se tiver de fazer alguma
coisa, terá de ser algo como um contacto directo com Deus. Não será capaz de dar 200 contos para ajudar uma vítima
da fome ou da miséria, ou para ajudar um estudante pobre inteligente que de outro modo terá de deixar de estudar.
63
Mas será capaz de gastar 200 contos numa peregrinação algures, ou num quadro com uma boa representação de
Deus, ou numa viagem para ir ouvir e ver o Papa.
Esta atitude compreende-se. O que o nosso interlocutor quer é resolver a sua vida; não quer resolver a vida dos
outros. E por mais que lhe possamos dizer que ajudar os outros é a melhor maneira de dar sentido à sua vida, isso é
demasiado rebuscado para ele. Ele quer falar directamente com a gerência para ter a certeza que é promovido, não
quer apostar na possibilidade de promoção sendo um trabalhador honesto e altruísta, que ajuda os seus colegas.
Compreende-se agora por que razão o facto de a resposta religiosa ser incompreensível não é grave para quem se
preocupa com o problema pessoal do sentido da vida, ao passo que o facto de a resposta filosófica ser dificilmente
compreensível é devastador. No primeiro caso, a incompreensão vem depois da garantia da vida eterna de bemaventurança; que não se compreenda o sentido de uma vida eterna de bem-aventurança, é irrelevante. Mas a resposta
filosófica é inquietante; exige um esforço de compreensão, coloca demasiadas coisas nas nossas mãos, não nos dá um
lugar de destaque com um Deus omnipotente e criador e ainda por cima nem sequer nos dá grandes garantias de ser
verdadeiras, excepto na exacta medida da força e plausibilidade dos seus argumentos.
É por estes motivos que penso que o problema do sentido da vida é mal compreendido quando é um problema
pessoal. Este problema não é pessoal, de facto. É universal. Mas se fazemos disto um problema pessoal, perdemos de
vista o essencial do problema. Estamos dispostos a tudo para “resolver” o problema, desde que isso seja uma solução
rápida e garantida que nos dê imensa importância. Não estamos dispostos a relativizar a nossa dor de alma, percebendo que é uma em 6 bilhões. Queremos que a nossa alma seja importante, e não uma em 6 bilhões. Mas não pode ser
importante, precisamente porque enquanto não a encararmos como uma em 6 bilhões, não conseguiremos compreender a importância que eventualmente terá.
Quero terminar com um comentário a um problema que surge muitas vezes ao discutir o problema do sentido da
vida. Curiosamente, é um padrão de pensamento que ocorre às pessoas que estão pessoalmente muito preocupadas
com a sua alma, mas do qual conseguem rapidamente afastar-se, quando isso lhes convém.
Uma maneira intuitiva de formular o problema do sentido da vida é começar a perguntar “Porquê?”. Podemos
perguntar: “Vou levantar-me porquê?” E temos muitas respostas: porque preciso de ir tomar banho e tomar o pequeno-almoço, e apanhar o metro para ir para a faculdade. E porquê? Bom, para poder estudar e poder tirar o curso. E
porquê? Bom, para ter um emprego e comprar uma casa e ter uma família. E porquê? Hum. Isto começa a perder o
sentido. No artigo da Intelecto expliquei que há dois tipos de finalidades e que o problema do sentido da vida é encontrar finalidades últimas. Mas este é o problema filosófico do sentido da vida.
No problema pessoal do sentido da vida não queremos saber de finalidades últimas. O que queremos é garantir
que a nossa vida terá sentido para nós, que seremos eternamente felizes, que nada nos poderá perturbar as tardes
felizes da eternidade abençoada. Que se lixe se isso é ou não uma finalidade última e que isso seja apenas prolongar
tolamente um estado de satisfação para todo o sempre, como se tal fosse sequer concebível. O que importa é que essa
fantasia é reconfortante; é como regressar ao ventre materno e congelar o tempo. Sabe bem.
O padrão de pensamento errado que estou a tentar caracterizar é subtil. Começa por exigir respostas a múltiplos
“porquês”. Não se contenta com resposta alguma, mesmo que sejam finalidades em si, como tomar chá, que é algo
que faço pelo prazer de o beber, ou estudar, que é algo que faço pelo prazer de estudar. Não. Queremos mais do que o
que nos dá prazer. Queremos uma finalidade última. Esta insistência torna-nos cegos para o facto de que grande parte
da nossa vida consiste em alcançar finalidades que não serão últimas, mas que são perfeitamente respeitáveis. Desde
que estudar me dê prazer, qual é o problema? Desde que a vida me corra bem, que eu seja feliz com a minha mulher e
os meus filhos e os meus amigos, qual é o problema? Desde que possa usufruir de uma boa vida, qual é o problema?
A insistência nas finalidades últimas pode cegar-nos para o que é de facto o mais importante para a vida da maior
parte das pessoas.
Mas depois este padrão de pensamento cessa subitamente. Quando atingimos o céu, deixamos de perguntar por
que razão faz tal coisa sentido. E isto cega-nos para o verdadeiro problema do sentido da vida, que é precisamente o
de saber que sentido haverá numa vida humana eterna de felicidade. Mas para o nosso interlocutor esta pergunta já
não faz sentido. O que ele procura é a resposta à inquietação da sua alma e uma garantia de que não voltará a ter tal
inquietação. E para isso nada melhor do que a eterna bem-aventurança, sentado ao lado de Deus.
Nenhuma das 3 respostas que podemos dar ao problema do sentido da vida responde ao problema pessoal do sentido da vida. Isto não é uma deficiência dessas respostas. Resulta antes do facto de o problema pessoal do sentido da
vida ser um falso problema, um problema que não tem realmente solução. É apenas uma inquietação emocional vaga
e pouco lúcida, que costuma atacar os seres humanos quando estão numa situação emocional difícil — mas que é
tranquilamente esquecida quando tudo corre bem na vida.
As 3 respostas que podemos dar, e as razões pelas quais são sempre insatisfatórias para o angustiado em busca da
salvação, são as que se seguem.
Em primeiro lugar, podemos procurar mostrar que acrescentar valor ao mundo dá sentido à nossa vida porque há
valores objectivos. Esta resposta não é satisfatória porque é demasiado abstracta. É difícil de compreender. Implicaria
estudar filosofia seriamente pelo menos durante alguns meses.
Em segundo lugar, podemos mostrar que as finalidades últimas não são muito importantes para a nossa felicidade. Afinal, se a minha vida for feliz, tudo o que faço para preservar essa felicidade, e para a aprofundar e talvez partilhar, faz todo o sentido — tal como faz sentido dar-me a um trabalho imenso para ir passar férias para muito longe.
Toda a viagem e despesa faz sentido porque aquelas férias naquele sítio são importantes para mim, e não há mais
nada a dizer que lhe possa dar realmente valor. E isso chega. Mas esta resposta não satisfaz quem se defronta com o
problema pessoal do sentido da vida porque essa pessoa está emocionalmente num estado que não lhe permite fruir a
amizade, as férias, o amor, uma tarde chuvosa de leitura empolgante. Essa pessoa quer algo tão importante que lhe
devolva o gosto pela vida. Mas:
64
Em terceiro lugar, podemos mostrar que, independentemente de os valores serem ou não objectivos, e independentemente de termos ou não gosto pela vida, há infelizmente sempre muitas pessoas há nossa volta que podemos
ajudar decisivamente, apenas com algum esforço da nossa parte. Mesmo que isso não nos dê satisfação a nós, pelo
menos dá satisfação a alguém. Esta resposta também não interessa nada ao nosso trágico angustiado, claro. Porque o
que ele quer é retomar o gosto pela vida. E não quer obter tal coisa através da via inesperada do altruísmo. Não. Quer
que Deus lhe fale directamente ao ouvido e lhe diga que a vida dela é muito, muito, muito importante, só porque é
uma vida humana e só porque é uma pessoa única. Acho que tenho más notícias. Nenhuma vida humana é em si algo
que tenha mais valor do que tem a vida de um símio — excepto se fizermos coisas que nenhum símio pode fazer,
como trazer algum valor ao mundo e fazer bem a alguém no mundo.
Não quero deixar o leitor com uma nota de pessimismo. Quero dizer-lhe que há imensas coisas que estão nas suas
mãos. Nomeadamente, as vidas das pessoas que estão à sua volta. E a criação de algo que seja bom para alguém. E
que se por acaso enfrenta o problema pessoal do sentido da vida, do ponto de vista emocional o melhor a fazer talvez
seja pensar um pouco menos em si e um pouco mais nos que estão à sua volta. Poderá descobrir, com algum espanto,
que não tinha gosto pela vida porque a vida não tem gosto quando só olhamos para nós próprios e nos esquecemos
dos nossos semelhantes.
MURCHO, Desidério. O problema pessoal do sentido da vida.
φ
É possível dar à expressão “ser humano” um significado preciso. Podemos usá-la como equivalente a “membro
da espécie Homo sapiens”. A questão de saber se um ser pertence a determinada espécie pode ser cientificamente
determinada por meio de um estudo da natureza dos cromossomas das células dos organismos vivos. Neste sentido,
não há dúvida que, desde os primeiros momentos da sua existência, um embrião concebido a partir de esperma e
óvulo humanos é um ser humano; e o mesmo é verdade do ser humano com a mais profunda e irreparável deficiência
mental — até mesmo de um bebé anencefálico (literalmente sem cérebro).
Há outra definição do termo “humano”, proposta por Joseph Fletcher, teólogo protestante e autor prolífico de escritos sobre temas éticos. Fletcher compilou uma lista daquilo a que chamou “indicadores de humanidade”, que inclui
o seguinte:
* Autoconsciência
* Autodomínio
* Sentido do futuro
* Sentido do passado
* Capacidade de se relacionar com outros
* Preocupação pelos outros
* Comunicação
* Curiosidade
É este o sentido do termo que temos em mente quando elogiamos alguém dizendo que “é muito humano” ou que
tem “qualidades verdadeiramente humanas”. Quando dizemos tal coisa não estamos, é claro, a referir-nos ao facto de
a pessoa pertencer à espécie Homo sapiens que, como facto biológico, raramente é posto em dúvida; estamos a querer
dizer que os seres humanos possuem tipicamente certas qualidades e que a pessoa em causa as possui em elevado
grau.
Estes dois sentidos de “ser humano” sobrepõem-se mas não coincidem. O embrião, o feto subsequente, a criança
gravemente deficiente mental e até mesmo o recém-nascido, todos são indiscutivelmente membros da espécie Homo
sapiens, mas nenhum deles é autoconsciente nem tem um sentido do futuro ou a capacidade de se relacionar com os
outros. Logo, a escolha entre os dois sentidos pode ter implicações importantes para a forma como respondemos a
perguntas como “Será que o feto é um ser humano?”
Quando escolhemos as palavras que usamos em situações como esta, devemos empregar os termos que permitam
exprimir o que queremos dizer com clareza e que não introduzam antecipadamente juízos sobre a resposta a questões
substanciais. Estipular que usamos o termo “ser humano”, digamos, no primeiro sentido e que, portanto, o feto é um
ser humano e o aborto é imoral não ajudaria em nada. Tão-pouco seria melhor escolher o segundo sentido e defender
nesta base que o aborto é aceitável. A moral do aborto é uma questão substancial, cuja resposta não pode depender do
sentido que estipularmos para as palavras que usamos. Para evitar fazer petições de princípio e para tornar o meu
sentido claro, porei de lado, por agora, o ambíguo termo “ser humano” e substitui-lo-ei por dois termos diferentes,
correspondentes aos dois sentidos diferentes de “ser humano”. Para o primeiro sentido, o biológico, usarei simplesmente a expressão extensa mas precisa “membro da espécie Homo sapiens”, enquanto para o segundo sentido usarei
o termo “pessoa”.
Este uso da palavra “pessoa” é, ele mesmo, infeliz, susceptível de criar confusões, dado que a palavra “pessoa” é
muitas vezes usada como sinónimo de “ser humano”. No entanto, os termos não são equivalentes; poderia haver uma
pessoa que não fosse membro da nossa espécie. Também poderia haver membros da nossa espécie que não fossem
pessoas. A palavra “pessoa” tem a sua origem no termo latino para uma máscara usada por um actor no teatro clássico. Ao porem máscaras, os actores pretendiam mostrar que desempenhavam uma personagem. Mais tarde “pessoa”
passou a designar aquele que desempenha um papel na vida, que é um agente. De acordo com o Oxford Dictionary,
um dos sentidos actuais do termo é “ser autoconsciente ou racional”. Este sentido tem precedentes filosóficos irrepreensíveis. John Locke define uma pessoa como
“um ser inteligente e pensante dotado de razão e reflexão e que pode considerar-se a si mesmo como aquilo que é, a mesma coisa pensante, em diferentes momentos e lugares.”
65
Esta definição aproxima a “pessoa” do sentido que Fletcher deu a “ser humano”, com a diferença que escolhe duas características cruciais — a racionalidade e a autoconsciência — para cerne do conceito. É muito possível que
Fletcher concordasse que estas duas características são centrais e que as restantes decorrem mais ou menos delas. Em
todo o caso, proponho-me usar o termo “pessoa” no sentido de um ser racional e autoconsciente, para captar os elementos do sentido popular de “ser humano” que não são abrangidos pelo termo “membro da espécie Homo sapiens”.
SINGER, Peter. O que é uma pessoa?
Cogito, logo aprendo
1. (UFMG 2007) Como narrado no diálogo “Críton, ou do dever”, Sócrates, após ser condenado, aguarda, na prisão, a execução de sua sentença, que consiste em tomar cicuta, pena de morte adotada entre os atenienses. Críton,
discípulo do filósofo, propõe-lhe a fuga e o exílio, sob a alegação de que a condenação teria sido injusta. Sócrates
recusa a oferta e, entre outras razões, apresenta a seguinte: “... viver não é o que mais deve importar, mas viver bem.”
[PLATÃO. Críton, ou do dever.]
Com base na leitura desse trecho e considerando outros elementos contidos na obra citada, APRESENTE dois argumentos que justifiquem por que, na opinião de Sócrates, no caso, viver bem implica aceitar a morte.
2. (UFMG 2007) “Se Deus não existisse, tudo seria permitido.” Essa frase é um comentário de Sartre a uma passagem do romance Os irmãos Karamázovi, de Dostoievski, e diz respeito ao problema filosófico da relação entre
moral e religião.
REDIJA um texto argumentando a favor de ou contra a ideia expressa na frase destacada de Sartre.
3. (UFMG ?) “Por tudo isto, evidencia-se também que nenhuma das virtudes morais surge em nós por natureza;
com efeito, nada do que existe naturalmente pode formar um hábito contrário à sua natureza. Por exemplo, à pedra
que por natureza se move para baixo não se pode imprimir o hábito de ir para cima, ainda que tentemos adestrá-la
jogando-a dez mil vezes no ar; nem se pode habituar o fogo a dirigir-se para baixo, nem qualquer coisa que por natureza se comporte de certa maneira a comportar-se de outra. [...] Da mesma forma, tornamo-nos justos praticando atos
justos, moderados praticando ações moderadas, e corajosos praticando ações corajosas.” [ARISTÓTELES. Ética a Nicômaco.]
A partir dessa leitura, EXPLIQUE a diferença entre virtude e aptidões naturais.
4. (UFMG ?) Leia estas passagens:
Passagem 1. “... a virtude diz respeito às paixões e ações, nas quais o excesso é uma forma de erro tanto quanto a
falta, enquanto o meio termo é louvado como acerto...”
Passagem 2. “Mas nem toda ação ou paixão admite um meio termo, pois algumas delas têm nomes nos quais já
está implícita a maldade...” [ARISTÓTELES. Ética a Nicômaco.]
COMENTE essas duas passagens, comparando-as e exemplificando-as.
5. (UFMG 2006) Leia este trecho: “Voltemos novamente ao bem que estamos procurando e indaguemos o que ele
é, pois não se afigura igual nas distintas ações e artes; é diferente na medicina, na estratégia e em todas as demais
artes do mesmo modo. Que é, pois, o bem de cada uma delas? Evidentemente, aquilo em cujo interesse se fazem
todas as outras coisas. Na medicina é a saúde, na estratégia a vitória, na arquitetura uma casa, em qualquer outra
esfera uma coisa diferente, e em todas as ações e propósitos é ele a finalidade; pois é tendo-o em vista que os homens
realizam o resto. Por conseguinte, se existe uma finalidade para tudo que fazemos, essa será o bem realizável mediante a ação [...] Mas procuremos expressar isto com mais clareza ainda. Já que, evidentemente, os fins são vários e nós
escolhemos alguns entre eles [...], segue-se que nem todos os fins são absolutos; mas o sumo bem é claramente algo
de absoluto. Portanto, se só existe um fim absoluto, será o que estamos procurando [...] Ora, nós chamamos aquilo
que merece ser buscado por si mesmo mais absoluto do que aquilo que merece ser buscado com vistas em outra coisa
[...] Ora, esse é o conceito que preeminentemente fazemos da felicidade.” [ARISTÓTELES. Ética a Nicômaco.]
Com base na leitura desse trecho e considerando outras ideias presentes nessa obra de Aristóteles, JUSTIFIQUE
esta afirmativa: “É absurdo perguntar para que queremos ser felizes.”
66
Download